不等式への招待 第5章

このエントリーをはてなブックマークに追加
1不等式ヲタ
ある人は蝶を集め、ある人は切手を収集し、ある人は不等式を集める…
          ___          ----- 参考文献〔3〕 P.65 -----
    |┃三 ./  ≧ \   
    |┃   |::::  \ ./ | 
    |┃ ≡|::::: (● (● |  不等式と聞ゐちゃぁ
____.|ミ\_ヽ::::... .ワ......ノ     黙っちゃゐられねゑ…
    |┃=__    \           ハァハァ
    |┃ ≡ )  人 \ ガラッ

過去スレ
・不等式スレッド (Part1)  http://science3.2ch.net/test/read.cgi/math/1072510082/
・不等式への招待 第2章 http://science6.2ch.net/test/read.cgi/math/1105911616/
・不等式への招待 第3章 http://science6.2ch.net/test/read.cgi/math/1179000000/
・不等式への招待 第4章 http://kamome.2ch.net/test/read.cgi/math/1245060000/

過去スレのミラー置き場:http://cid-d357afbb34f5b26f.skydrive.live.com/browse.aspx/.Public/

まとめWiki http://wiki.livedoor.jp/loveinequality/

姉妹サイト(?)
Yahoo! 掲示板 「出題 不等式」 http://messages.yahoo.co.jp/bbs?.mm=GN&action=l&board=1835554&tid=bdpbja1jiteybc0a1k&sid=1835554&mid=10000
2不等式ヲタ:2010/10/24(日) 23:58:46
[1] 不等式,ハーディ・リトルウッド・ポリヤ,シュプリンガー,2003年
   http://amazon.co.jp/o/ASIN/4431710566
[2] 不等式,大関信雄・青木雅計,槇書店,1967年(絶版)
[3] 不等式への招待,大関信雄・大関清太,近代科学社,1987年(絶版)
[4] 不等式入門,渡部隆一,森北出版,2005年
   http://amazon.co.jp/o/ASIN/4627010494
[5] 不等式の工学への応用、海津聰、森北出版,2004年
   http://amazon.co.jp/o/ASIN/4627075812
[6] 不等式(モノグラフ4),染取弘,科学新興新社,1990年
   http://amazon.co.jp/o/ASIN/4894281740
[7] 数理科学 No.386 特集「現代の不等式」,サイエンス社,1995年8月号(絶版)
[8] 数学トレッキングツアー第3章「相加平均≧相乗平均」,東京理科大学数学教育研究所,教育出版,2006年
   http://amazon.co.jp/o/ASIN/4316801988
[9] 数学オリンピック事典,数学オリンピック財団,朝倉書店、2001年
   http://amazon.co.jp/o/ASIN/4254110871
[10] The Cauchy-Schwarz Master Class: An Introduction to the Art of Mathematical Inequalities,J. M. Steele,Cambridge Univ. Pr.,2004年
   http://amazon.co.jp/o/ASIN/052154677X
[11] 数学セミナー 2009年 02月号,日本評論社,2009年
   http://www.amazon.co.jp/o/ASIN/B001O9UIZ8
[12] 大学への数学 2009年4月号-2010年3月号,東京出版
    連載 「不等式の骨組み」 、栗田哲也、全12回、各4ページ
[13] Inequalities: A Mathematical Olympiad Approach,Birkhaeuser Basel,2009年
   http://amazon.jp/dp/3034600496
3不等式ヲタ:2010/10/24(日) 23:59:53
不等式の埋蔵地
[1] RGMIA http://rgmia.vu.edu.au/
[2] Crux Mathematicorum Synopses http://www.journals.cms.math.ca/CRUX/synopses/
[3] Maths problems http://www.kalva.demon.co.uk/
[4] Mathematical Inequalities & Applications http://www.ele-math.com/
[5] American Mathematical Monthly http://www.maa.org/pubs/monthly.html
[6] Problems in the points contest of KoMaL http://www.komal.hu/verseny/feladatok.e.shtml
[7] IMO リンク集 http://imo.math.ca/
[9] Mathematical Olympiads Correspondence Program http://www.cms.math.ca/Competitions/MOCP/
[10] Mathematical Excalibur http://www.math.ust.hk/excalibur/
[11] MathLinks Contest http://www.mathlinks.ro/Forum/contest.html
[12] MATH PROBLEM SOLVING WEB PAGE http://www.math.northwestern.edu/~mlerma/problem_s... (要自動登録)
[13] Wolfram MathWorld http://mathworld.wolfram.com/
[14] GRA20 Problem Solving Group http://www.mat.uniroma2.it/~tauraso/GRA20/main.htm...
[15] American Mathematical Monthly Problems http://www.mat.uniroma2.it/~tauraso/AMM/amm.html
[16] Journal of Inequalities and Applications http://www.hindawi.com/journals/jia/

海外不等式ヲタの生息地
[1] Journal of Inequalities in Pure and Applied Mathematics http://jipam.vu.edu.au/
[2] MIA Journal http://www.mia-journal.com/
[3] MathLinks Math Forum http://www.mathlinks.ro/Forum/forum-55.html
4不等式ヲタ:2010/10/25(月) 00:03:24
>>2
姉妹サイト(?)に付け忘れました

キャスフィ 高校数学板 不等式スレ http://www.casphy.com/bbs/test/read.cgi/highmath/1169210077/


         ∧_∧
         (´Д` )   死んでお詫びを…
         /  y/  ヽ      
    Σ(m)二フ ⊂[_ノ
        (ノノノ | | | l )
    ̄ ̄ ̄ ̄ ̄ ̄ ̄ ̄ ̄ ̄
5132人目の素数さん:2010/10/25(月) 00:11:20
http://www.mat.uniroma2.it/~tauraso/GRA20/main.html

M1854、M1852、C934、C932 に(*゚∀゚)=3 ハァハァ…
6132人目の素数さん:2010/10/25(月) 04:04:00
>>5 読めぬぅ…

〔M1852.〕
f∈C^1([0,1]) で f(0) = f(1) = -1/6 とする。次を示せ。
 ∫[0,1] {f '(x)}^2 dx ≧ 2∫[0,1] f(x)dx + (1/4),
等号は f(x) = (1/2)x(1-x) - 1/6. (Cezar Lupu e Tudorel Lupu (Romania))


〔C934.〕
n個の辺からなる多角形を考え、その半周をsとおく。次を示せ。
 農[1≦i<j≦n] (a_i)^2・(a_j)^2/{(a_i)^2 + (a_j)^2} ≦ {(n-1)/(n-2)^2}納k=1,n] (s-a_k)^2,
ここに、a_1, a_2, ……, a_n はn個の辺の長さ。(Jose Luis Diaz-Barrero (Spagna))


〔C932.〕
f:[0,1]→R は連続函数で ∫[0,1] {f(x)}^3 dx =0 とする。次を示せ。
 ∫[0,1] {f(x)}^4 dx ≧ (27/4){∫[0,1] f(x)dx}^4,
            (Cezar Lupu e Tudorel Lupu (Romania))

http://www.mat.uniroma2.it/~tauraso/GRA20/main.html
7132人目の素数さん:2010/10/25(月) 10:01:52
>>6
文献[3] 不等式への招待 のP.145 あたりを参考に考えたが…
8132人目の素数さん:2010/10/27(水) 10:31:36
猫に小判、まで読んだ。
9猫は火病 ◆MuKUnGPXAY :2010/10/27(水) 16:03:54

10132人目の素数さん:2010/10/27(水) 23:33:42
等差数列a[i],等比数列b[i]を考える

0<a[1]<a[2]<…<a[n]
0<b[1]<b[2]<…<b[n]
a[1]=b[1],a[n]=b[n]

を満たすとき
Σ[i=1~n]a[i]≧Σ[i=1~n]b[i]
11132人目の素数さん:2010/10/28(木) 00:06:26
bk=(bn)^(k-1)/nb1
ak=(bn-b1)(k-1)/n+b1
12132人目の素数さん:2010/10/31(日) 14:42:03
>>10

.     (n-k)個、   (k-1)個
a[k] =(a[1],…,a[1], a[n],…,a[n] の相加平均)
b[k] =(b[1],…,b[1], b[n],…,b[n] の相乗平均)

∴ a[k] ≧ b[k],

ぬるぽ
13132人目の素数さん:2010/11/01(月) 10:58:38
>>6の積分の不等式を見ると
それを証明できそうになくても
フリーザやセルにボコられる前の
悟空のようにワクテカしてしまいます
14132人目の素数さん:2010/11/02(火) 21:15:33
今年の東工大AO

a,b,cを三角形の三辺の長さとするとき
(a^2+b^2+c^2)/(ab+bc+ca)
のとりうる値の範囲を求めよ
15132人目の素数さん:2010/11/02(火) 21:39:24
大数の宿題

Aはxy平面上のx^2+y^2=1を動き
Bはyz平面上のy^2+z^2=1を動き
Cはzx平面上のz^2+x^2=1を動くとき
AB^2+BC^2+CA^2のとりうる値の範囲を求めよ
16132人目の素数さん:2010/11/02(火) 21:41:56
三角関数を知っていれば
どちらも解けそうですな。
17132人目の素数さん:2010/11/02(火) 21:57:04
A=(x,(1-x^2)^.5,0)
B=(0,y,(1-y^2)^.5)
C=((1-z^2)^.5,0,z)
f=AB^2+BC^2+CA^2
fx=fy=fz=0
18132人目の素数さん:2010/11/02(火) 22:26:34
fx=Ax=(1,-2x(1-x^2)^-.5,0)=0,x=0,+/-1
(x,y,z)=(0+/-1,0+/-1,0+/-1)
19132人目の素数さん:2010/11/02(火) 22:29:58
0,3
20132人目の素数さん:2010/11/02(火) 22:41:37
A、Bの焦点の楕円からCにMAXをとる。以下同文
21132人目の素数さん:2010/11/03(水) 02:36:01
>>14
 1 ≦ (与式) < 2,

 (a^2 +b^2 +c^2) - (ab+bc+ca) = (1/2){(a-b)^2 + (b-c)^2 + (c-a)^2} ≧ 0,
 等号成立は a=b=c (正三角形)。

 2(ab+bc+ca) - (a^2 +b^2 +c^2) = (2ab -a^2 -b^2 +c^2) + (2bc -b^2 -c^2 +a^2) + (2ca - a^2 -c^2 +b^2)
   = { -(a-b)^2 + c^2} + { -(b-c)^2 + a^2) + { -(c-a)^2 + b^2}
   = (-a+b+c)(a-b+c) + (-b+c+a)(b-c+a) + (-c+a+b)(c-a+b)
   > 0,

あるいは >>16 にしたがって
 2(ab+bc+ca) - (a^2 +b^2 +c^2) = (2ab -a^2 -b^2 +c^2) + (2bc -b^2 -c^2 +a^2) + (2ca - a^2 -c^2 +b^2)
   > {2ab・cos(C) -a^2 -b^2 +c^2} + {2bc・cos(A) -b^2 -c^2 +a^2} + {2ca・cos(B) -a^2 -c^2 +b^2}
   = 0,
   等号成立は一辺が → 0 のとき。
22132人目の素数さん:2010/11/03(水) 04:19:14
>>6

〔C934.〕
 (a_i)^2 + (a_j)^2 - 2a_i・a_j = (a_i-a_j)^2 ≧ 0,
より
(左辺) ≦ (1/2)納1≦i<j≦n] a_i・a_j ≦ {(n-1)/n}s^2,  (*)

 納k=1,n] (s-a_k)^2 = 納k=1,n] {s^2 -2s・a_k + (a_k)^2}
   = (n-4)s^2 + 納k=1,n] (a_k)^2
   ≧ (n-4)s^2 + (4/n)s^2           (**)
   = (1/n)(n-2)^2・s^2,

∴ (右辺) ≦ {(n-1)/n}s^2,

n辺多角形でなくても成立つ希ガス。
---------------------------------------------------
〔補題〕  = 納1≦i,j≦n] (a_i-a_j)^2 ≧ 0, より

(*) 納1≦i<j≦n] a_i・a_j = {2(n-1)/n}s^2 - (1/2n) ≦ {2(n-1)/n}s^2,

(**) 納k=1,n] (a_k)^2 = (4/n)s^2 + (1/n) ≧ (4/n)s^2,
23132人目の素数さん:2010/11/04(木) 23:59:21
f (0) = f(1)= 0 のとき
∫[0,1] {f'(x)}^2・dx ≧∫[0,1] {f(x)}^2 を示せ

〔M1852.〕と似てる
24132人目の素数さん:2010/11/08(月) 01:08:45
>>23

〔Wirtingerの不等式〕
 f(x), f '(x) が [0,1] で連続のとき
 ∫[0,1] {f'(x)}^2・dx ≧ (π^2)∫[0,1] {f(x)}^2 dx,
 等号成立は f(x) = c・sin(πx) のとき。


f(x) を sine級数でフーリエ展開して
 f(x) = 納k=1,∞) b_k sin(kπx),
とおく。
 (右辺) = (π^2)納k=1,∞) (1/2)(b_k)^2,

 f '(x) = π納k=1,∞) k・b_k cos(kπx),
 (左辺) = (π^2) 納k=1,∞) (1/2)(k・b_k)^2,
25132人目の素数さん:2010/11/08(月) 18:20:11
>>23
初等的証明

f(0)=0 より f(x)=∫_[0,x] f'(t) dt と書ける.
Schwarzの不等式とHo"lderの不等式より,
|f(x)|≦ ∫_[0,x] |f'(t) | dt
   ≦ { ∫_[0,x] 1^2 dt }^{1/2}・{∫_[0,x] |f' (t)|^2 dt}^{1/2}.
両辺自乗すれば,
|f(x)|^2 ≦ x ∫_[0,x] |f' (t)|^2 dt.
よって,x について 0 →1まで積分すれば,
∫_[0,1] |f(x)|^2 dx ≦ ∫_[0,1] x { ∫_[0,x] |f' (t)|^2 dt } dx.
ここで、右辺を部分積分すれば,
右辺 = [ x^2/2・∫_[0,x] |f' (t)|^2 dt ]_[x=0]^1 - ∫_[0,1] x^2/2・|f' (x)|^2 dx
   = 1/2 ・∫_[0,1] |f' (t)|^2 dt - - ∫_[0,1] x^2/2・|f' (x)|^2 dx
   = 1/2 ・∫_[0,1] (1-x^2) |f' (x)|^2 dx
   ≦1/2 ・∫_[0,1] |f' (x)|^2 dx
   ≦∫_[0,1] |f' (x)|^2 dx

よって、  
 ∫_[0,1] |f(x)|^2 dx ≦ ∫_[0,1] |f' (x)|^2 dx
が証明できた。
26132人目の素数さん:2010/11/08(月) 19:08:21
>>25
すっげー、ネ申!
27じゅー:2010/11/19(金) 00:53:54
キャスフィ―修羅の刻―より。
正の実数a,b,cに対し次を満たす最大の実数kを求めよ。
{(a+2b)(b+2c)(c+2a)}^2
≧k(ab+bc+ca)^3
できれば解き方も書いてほしいです。
非対称な部分の処理の仕方がわからないので教えて下さいって意味も込めて。
28132人目の素数さん:2010/11/19(金) 13:47:12
シュワルツの不等式サイコウ
29132人目の素数さん:2010/11/21(日) 19:41:21
>>27

基本対称式を a+b+c=s, ab+bc+ca=t, とおくと、
 (a+2b)(b+2c)(c+2a) = 3st + (a-b)(b-c)(c-a) = 3st + ,
非対称な部分 (a-b)(b-c)(c-a) = は正にも負にもなり得るので、
絶対値を押さえます。

〔補題〕 |處 ≦ (2/√3)(t/s)(s^2 -3t),

詳細は [第3章.727, 737-739] へ。ミーラ置き場にあります...
30132人目の素数さん:2010/11/21(日) 19:48:23
>>27

序でに解き方も書いておこう。
 (a+2b)(b+2c)(c+2a) - (3t)^(3/2)
 = 3st + - 3t√(3t)
 = 3t{s-√(3t)} +
 ≧ (3t/2s)(s^2 -3t) - |處   (← *)
 ≧ (3/2 - 2/√3)(s^2 -3t)   (←補題)
 ≧ 0,             (← **)

*) s-√(3t) = (s^2 -3t)/{s+√(3t)} ≧ (s^2 -3t)/2s,

〔補題〕 |處 ≦ (2/√3)(t/s)(s^2 -3t),
(略証)
min(a,b,c) = m とおき、{a,b,c} = {m, m+x, m+x+y} とする。(x,y≧0)
然らば、 |處 = xy(x+y), s = 3m+2x+y, t = 3m^2 + 2m(2x+y) + x(x+y), s^2 -3t = x^2 +xy +y^2,
∴ t(s^2 -3t) - ((√3)/2)s|處 = 3m^2・(x^2 +xy +y^2) + m・{4x^3 + 3(1-(√3)/2)xy(x+y) +2y^3} + x(x+y){x - ((√3 -1)/2)y}^2 ≧0,
等号成立は m=0 かつ x/y = (√3 -1)/2 のとき。

**) s^2 -3t = (1/2){(a-b)^2 + (b-c)^2 + (c-a)^2} ≧ 0,
31じゅー:2010/11/22(月) 00:16:47
巧妙ですね。
まず(a-b)(b-c)(c-a)を作るんですか??
あとは最小値とかを与えて計算??
|處=xy(x+y)となるのは面白いですね。
32132人目の素数さん:2010/11/27(土) 23:50:59
[問題]
f は区間 (a,b) (a<b) で C^2 級関数であるとき、

 ∫_[a,b] |f′(x) |dx ≦ 54{ 1/(b-a)^2 ∫_[a,b] |f(x)|^2 dx
                + (b-a)^2∫_[a,b] |f" (x)|^2 dx }

が成立することを示せ。
33132人目の素数さん:2010/12/02(木) 22:54:21
キャスフィー からもう一題。(Σ計算-180)

〔問題〕
 a[1],a[2],・・・,a[n]≧0
 納k=1,n] a[k] = S のとき 次を示せ。
(1) 納k=1,n] a[k]^2 ≧ (1/n)S^2,
(2) 納k=1,n-1] a[k]a[k+1] ≦ (1/4)S^2,
34132人目の素数さん:2010/12/06(月) 20:57:07
>>33 しょうがねぇなぁ…

(3) 納k=1,n-2] a[k]a[k+1]a[k+2] ≦ (S/3)^3,
(4) 納k=1,n-3] a[k]a[k+1]a[k+2]a[k+3] ≦ (S/4)^4,
 … …

35132人目の素数さん:2010/12/06(月) 23:51:02
>>33
(1)シュワルツの不等式より、

S=納k=1,n] 1・a[k] ≦ { 納k=1,n] 1^2 }^{1/2}・{ 納k=1,n] a[k]^2 }^{1/2} = {n}^{1/2}・{ 納k=1,n] a[k]^2 }^{1/2}.

よって,
   S^2 ≦ n 納k=1,n] a[k]^2
が証明された。
36132人目の素数さん:2010/12/09(木) 03:54:32
不等式の『未解決』問題集ってどこかに無いかな?

フェルマー予想やポアカレ予想のような、不等式界における大予想、大問題
というのを知りたいね。

ついでに、このスレでチャレンジしてみるのも面白そう。

不等式の大予想、2ちゃんねるスレで解ける!

みたいに…

どこかの本や問題集からの問題はやる気が失せる。
37132人目の素数さん:2010/12/09(木) 03:58:53
例えば、ラマヌジャン系の不思議な不等式に証明を与える(ないし、反例を与える)作業とか

なんか良い文献ないかなあ
38132人目の素数さん:2010/12/09(木) 05:43:22
Bateman

[Ramanujan Book]

ha dou??

39132人目の素数さん:2010/12/09(木) 10:04:33
nesbitの不等式って未解決の部分があるんじゃなかったっけ
40132人目の素数さん:2010/12/09(木) 22:03:35
>>36
巡回和関係の不等式は見かけは簡単でも、未解決なのが結構あるそうだ。

Shapiroの巡回不等式もこの10年数年に解けたそうな。
(大関本ではまだ未解決になっている)
41132人目の素数さん:2010/12/09(木) 22:07:06
>>39
Nsbittの不等式の一般化が Shapiroの巡回不等式。
だから現在では解決済み。
42132人目の素数さん:2010/12/09(木) 22:18:31
43132人目の素数さん:2010/12/11(土) 15:07:40
>>42
思わずフルボッキしてしまった!
44132人目の素数さん:2010/12/12(日) 05:14:43
>>29-31

〔補題’〕a,b,c≧0 のとき |處 ≦ 2{s-√(3t)}t,
 等号成立は m=0 かつ x/y = 1/2 のとき。
45132人目の素数さん:2010/12/12(日) 06:08:01
Shapiro's cyclic sum

http://mathworld.wolfram.com/ShapirosCyclicSumConstant.html

なお、λ = 1/3 としたものが [初代スレ.501] にある。
46132人目の素数さん:2010/12/25(土) 18:03:31
 
47132人目の素数さん:2010/12/30(木) 19:09:27
[問題]
A=(a_[ij]) を複素n次正方行列とし α_1,,, α_n を A の固有値(重複度込み)とする。
このとき、次の不等式を示せ。
  農[k=1,,n] |α_k|^2 ≦ 農[i,j=1,,,n] |a_[ij]|^2

また等号が成立するための必要十分条件は A が正規行列(A^* A = A A^*)である。
48132人目の素数さん:2010/12/30(木) 19:13:47
>>47
この不等式は「Shurの不等式」と呼ばれ線型代数の本などに載っていますが、
不等式の部分だけでも線型代数を使わない証明をして欲しいです。
49132人目の素数さん:2011/01/03(月) 02:32:47
〔問題961〕
自然数nに対して、
 I_n = ∫[0,(2n+1)π] {x・sin(x)/[n + sin(x)^2]} dx とおく。
 (2n+1)π/(n+1) < I_n < (2n+1)π/n を示せ。


http://www.casphy.com/bbs/test/read.cgi/highmath/1089455158/961-965
 casphy - 高校数学 −修羅の刻−【難問】
50132人目の素数さん:2011/01/06(木) 23:40:15
a、b、c、dを正の定数とする。
不等式 s(1-a)-tb>0
-sc+t(1-d)>0
を同時に満たす正の数s、tがあるとき、
2次方程式x^2-(a+d)x+(ad-bc)=0は-1<x<1の範囲に異なる2つの実数解をもつことを示せ。
51132人目の素数さん:2011/01/09(日) 06:31:06
>>50
 f(x) = x^2 -(a+d)x +(ad-bc), とおく。

 判別式 D = (a+d)^2 -4(ad-bc) = (a-d)^2 +4bc > 0,
より、異なる2つの実数解をもつ。

 f((a+d)/2) = −(1/4)(a-d)^2 -bc < 0,
また、題意より
 0 < a,d < 1,
 1-a > (t/s)b > 0,
 1-d > (s/t)c > 0,
∴ f(1) = (1-a)(1-d) -bc > 0,
  f(a+d-1) = f(1) > 0,

∴ 2実数解は a+d-1<x<(a+d)/2, および (a+d)/2<x<1 の範囲にある。
52132人目の素数さん:2011/01/09(日) 14:17:45
http://www.math.ust.hk/excalibur/v15_n3.pdf


P352など

 二 `丶、`丶、_\__\〉ノノへ!
`‐-、 二. `ヽ、 ミ ̄ /⌒シ′)
二‐/,ィ┐|=ミ=┘ ,r‐'_二ニ....イ
‐ニ| i<  i ,..-=ニ‐''\  /彡}    http://www.math.ust.hk/excalibur/v15_n3.pdf
二‐ヽ ┘ |     lヾ. } } / /リ
ニ ‐'"/   /    |_{;)} レ' /((   Problem352などを見ると・・・・・
'  /   /     '" ` `゙ / ソ
  /    ,      F'′/    なんていうか・・・・・・その・・・
  ヽ.    \、 L`___l       
 _\    ヽ._>┘         下品なんですが・・・・・・フフ・・・・・
 /了\_ノ
 ◆(                 勃起・・・・・・しちゃいましてね・・・・・・・・・
 門|
53132人目の素数さん:2011/01/10(月) 11:49:13
>>52

〔Problem 352.〕
a,b,c>0, abc≧1 のとき 
 a/{√(bc)+1} + b/{√(ca)+1} + c/{√(ab)+1} ≧ 3/2,
               (P.H.O.Pantoja による)
 
Math. Excalibur, Vol.15, No.3, 2010/Oct.-Dec.
54132人目の素数さん:2011/01/10(月) 11:53:48
>>53

 abc =k とおく。
 (左辺) = a^(3/2)/(√k + √a) + b^(3/2)/(√k + √b) + c^(3/2)/(√k + √c)
    = F(a) + F(b) + F(c),
ここに
 F(x) = x^(3/2)/(√k + √x) = x - √(kx) +k -(k√k)/(√k + √x),
 F '(x) = {3√(kx) + 2x}/{2(√k +√x)^2} > 0,  (単調増加)
 F "(x) = {√k + (3/√x)}/{4(√k + √x)]^3} > 0, (下に凸)
よって
 (左辺) > 3F((a+b+c)/3)   (← Fは下に凸)
    > 3F(k^(1/3))     (← 相加・相乗平均)
    = 3(√k)/{√k + k^(1/6)}
    > 3/2,            (← k≧1)
55132人目の素数さん:2011/01/10(月) 12:16:29

〔Problem 357.〕
正の整数nに対し、次を満たす4整数 a,b,c,d が存在しないことを示せ。
 ad = bc,
 n^2 < a < b < c < d < (n+1)^2,


〔Problem 359.〕
次を満たすすべての実数(x,y,z)を求む。
 x+y+z ≧ 3,
 x^3+y^3+z^3 + x^4+y^4+z^4 ≦ 2(x^2+y^2+z^2),
             (M.Bataille による)

http://www.math.ust.hk/excalibur/v15_n3.pdf
 Excalibur, Vol.15, No.3, 2010/Oct-Dec
56132人目の素数さん:2011/01/11(火) 21:32:52
>>55

P.359.
 x^4 + x^3 - 2x^2 = (x^2 +3x +3)(x-1)^2 + 3(x-1) ≧ 3(x-1),
等号成立は x=1 のとき。
∴ (左辺) - (右辺) ≧ 3(x+y+z-3) ≧ 0,

∴ 題意の不等式が成立するのは (x,y,z)=(1,1,1) のみ。
57132人目の素数さん:2011/01/12(水) 21:00:00
1<m<n。
0≦a。
b=(n−m)a/(n−1)。
c=(m−1)a/(n−1)。
a=b+c。
ma=b+nc。
a/m≦b+c/n。

 (x+n(1−x))(x+(1−x)/n)^2
=(2n−2(n−1)x)(1+(n−1)x)^2/2n^2
≦((2n+2)/3)^3/2n^2
=4(n+1)^3/27n^2。
58132人目の素数さん:2011/01/14(金) 00:12:11
>>57

〔問題〕
nを自然数、 0≦x≦1 を実数とするとき、
 {x + n(1-x)}{x + (1-x)/n}^2 の最大値を求めよ。


hint:
 {x+n(1-x), [nx+(1-x)]/2, [nx+(1-x)]/2} の相加平均は (n+1)/3,
59132人目の素数さん:2011/01/14(金) 05:00:00
1≦i≦n。
(i−1)(i−n)≦0。
n≦i(n+1−i)。
1/i≦(n+1−i)/n。

 Σ(ia(i))Σ(a(i)/i)^2
≦Σ(ia(i))Σ(((n+1−i)/n)a(i))^2
=Σ(ia(i))(((n+1)/n)Σ(a(i))−(1/n)Σ(ia(i)))^2
≦(4(n+1)^3/27n^2)Σ(a(i))^3。
60132人目の素数さん:2011/01/16(日) 05:00:00
 Σ(ia(i))Σ(a(i)/i)^2
≦Σ((n+1−n/i)a(i))Σ(a(i)/i)^2
=((n+1)Σ(a(i))−nΣ(a(i)/i))Σ(a(i)/i)^2
≦(4(n+1)^3/27n^2)Σ(a(i))^3。
61132人目の素数さん:2011/01/16(日) 08:21:59
62132人目の素数さん:2011/01/18(火) 22:52:05
x > 0 、 y > 0 、 0 < p < 1 のとき、 (x+y)^p < x^p + y^p を示せ
63132人目の素数さん:2011/01/18(火) 23:01:47
>>62
成り立たねーぞボケ
64132人目の素数さん:2011/01/19(水) 01:21:33
>>63
ニヤニヤ…
65132人目の素数さん:2011/01/19(水) 02:34:17
x^p+y^p-(x+y)^p
x^p+t^p*x^p-(1+t)^p*x^p
=x^p*(1+t^p-(1+t)^p)
>0
66132人目の素数さん:2011/01/19(水) 02:41:07
もっとエレガントに解かんかい
67132人目の素数さん:2011/01/19(水) 05:52:11
>>62
私の粗末なコレクションを検索したところ、似たようなものがあった。

[1997早稲田大]-----------------------------------------------------------

x、yを任意の正の数とし、p、qを 1/p + 1/q = 1 かつ p>1、q>1 をみたす有理数とする。

(1) (x+y)^2 ≦ px^2 + qy^2 を示せ

(2) (x+y)^(1/p) < x^(1/p) + y^(1/p) を示せ

-------------------------------------------------------------------------

これって、有理数という縛りは必要なのかな? 不等式( ゚∀゚)ハァハァ…
68132人目の素数さん:2011/01/19(水) 05:54:57
>>65がわからん
69132人目の素数さん:2011/01/19(水) 07:18:39
俺も>>63>>65がわからん
70132人目の素数さん:2011/01/19(水) 07:57:18
>>63
>>66
口が悪いな、直したほうがいい
71 ◆LANDAUL/nY :2011/01/19(水) 09:46:33
y=txとおいてるのかな
1+t^p-(1+t)^p>0の証明は
f(t)とおいてp<1なのでf'(t)<0だから
lim_{t→∞}f(t)>0
ということかな?



(x^p+y^p)^(1/p)>x+y
を示す
x≧yとする
(x^p+y^p)^(1/p)≧(x^p+x^p)^(1/p)=2^(1/p)*x>2x=x+x≧x+y
72132人目の素数さん:2011/01/19(水) 10:07:52
コーシーか相加相乗しか認めんぞ
73 ◆LANDAUL/nY :2011/01/19(水) 10:13:11
普通に間違ってた
74132人目の素数さん:2011/01/19(水) 11:02:41
>>71
x^p+y^p≦2*((x+y)/2)^p
(x^p+y^p)^(1/p)≦2^(p-1)*(x+y)
(x^p+y^p)^(1/p)<x+y
75132人目の素数さん:2011/01/19(水) 22:23:25
不等式に魅せられた高校生なんですけど[13]の書籍は体系だって不等式を学ぶのに有効ですか?
76132人目の素数さん:2011/01/19(水) 22:54:19
>>62, >>66, >>68-69,
 1-p > 0,
 x^p = x/{x^(1-p)} > x/(x+y)^(1-p),
 y^p = y/{y^(1-p)} > y/(x+y)^(1-p),
辺々たす。 (終)

>>67

(1) 題意より (p-1)(q-1) = 1,
  px^2 + qy^2 - (x+y)^2 = {x√(p-1) - y√(q-1)}^2 ≧ 0,

(2) 上と同様。ただし p ⇔ 1/p
77132人目の素数さん:2011/01/23(日) 06:27:07
>>67
(x+y)^(1/p)*(x+y)^(1/q)
= (x+y)^(1/p + 1/q)
= x+y
= x^(1/p + 1/q) + y^(1/p + 1/q)
= x^(1/p)*x^(1/q) + y^(1/p)*y^(1/q)
< x^(1/p)*(x+y)^(1/q) + y^(1/p)*(x+y)^(1/q)
= {x^(1/p) + y^(1/p)}*(x+y)^(1/q)

両辺を (x+y)^(1/q) で割って (*゚∀゚)=3 ハァハァ、ハァハァ、ハァハァ…
78132人目の素数さん:2011/01/28(金) 09:21:02
a、b、c が三角形の3辺の長さをなしながら変化するとき、
(a^2 + b^2 + c^2)/(ab+bc+ca) のとりうる値の範囲を求めよ
79132人目の素数さん:2011/01/28(金) 11:10:19
>>78

 1 ≦ (a^2 + b^2 + c^2)/(ab+bc+ca) < 2,

(右側)
 (a^2 + b^2 + c^2) - (ab+bc+ca) = (1/2)(a-b)^2 + (1/2)(b-c)^2 + (1/2)(c-a)^2 ≧ 0,
 等号成立は a=b=c (正三角形)

(左側)
 2(ab+bc+ca) - (a^2 + b^2 + c^2) = a(b+c-a) + b(c+a-b) + c(a+b-c) > 0,
80132人目の素数さん:2011/01/28(金) 11:56:40
>>79
すっきりした証明ですね、(*゚∀゚)=3 ハァハァ…
81132人目の素数さん:2011/01/28(金) 12:16:14
>>78
2011年度 東工大特別入試 第2問 [大学への数学2011年1月号P.P.56-57]

雑誌の模範解答は、3通り

(解1)
a+b+c = 2k を固定し、ab+bc+ca = t とおくと、
 (a^2 + b^2 + c^2)/(ab+bc+ca) = 4k^2/t - 2
だから、tのとりうる値を考える

三角形の成立条件から、a<k、b<k、a+b<k なので、
aを 0<a<s の範囲で固定して、k-a < b < s の範囲で、
 t = - { b - (2k-a)/2 }^2 -3a^2/4 + ka + k^2
のとりうる値の範囲を求める

(解2)
x=b+c-a、y=c+a-b、z=a+b-c とおくと、x、y、z>0で
 (a^2 + b^2 + c^2)/(ab+bc+ca) = (2/3){1 + 2/(1+3u)}
ただし、u = (xy+yz+zx)/(x^2 + y^2 + z^2)
uのとりうる値の範囲を考える

(解3)
a≦b≦cと設定して、cの関数とみて微分

(;´д`) ハァハァ…
82132人目の素数さん:2011/01/28(金) 19:29:45
本番でa=1固定で
b+c=k固定で動かした記憶
83132人目の素数さん:2011/01/28(金) 22:46:10
数学板で一番の良スレ
久しぶりに(気のせいか)上に上がってきたな
sageなかったのか?
84132人目の素数さん:2011/01/28(金) 23:10:08
>>83
お前と、82が上げたんだろうが!
85132人目の素数さん:2011/01/28(金) 23:18:33
age、sage言ってる時点でじじいだから、頭がボケていても仕方ない。
86Fランク受験生:2011/01/29(土) 03:13:19
>>79
きれいなやりかたですね。

すこしきになるのは(a^2 + b^2 + c^2)/(ab+bc+ca)のとりうる範囲が[1,2]
の中間の値を全部とるというのはどこで証明しているのでしょうか?
87132人目の素数さん:2011/01/29(土) 03:49:19
投稿確認
・投稿者は、投稿に関して発生する責任が全て投稿者に帰すことを承諾します。

・投稿者は、話題と無関係な広告の投稿に関して、
相応の費用を支払うことを承諾します

・投稿者は、投稿された内容及びこれに含まれる知的財産権、
(著作権法第21条ないし第28条に規定される権利も含む)その他の権利につき
(第三者に対して再許諾する権利を含みます。)、掲示板運営者に対し、
無償で譲渡することを承諾します。ただし、投稿が別に定める削除ガイドラインに該当する場合、
投稿に関する知的財産権その他の権利、義務は一定期間投稿者に留保されます。

・掲示板運営者は、投稿者に対して日本国内外において無償で非独占的に複製、
公衆送信、頒布及び翻訳する権利を投稿者に許諾します。
また、投稿者は掲示板運営者が指定する第三者に対して、
一切の権利(第三者に対して再許諾する権利を含みます)を許諾しないことを承諾します。

・投稿者は、掲示板運営者あるいはその指定する者に対して、
著作者人格権を一切行使しないことを承諾します。
8879:2011/01/29(土) 04:17:32
>>86
 (a,b,c) = (1,1,c) とし、cを (0,1] で連続的に変化させてみる。

比の値がr (1≦r<2) となるのは c=r-√{(r-1)(r+2)} のとき。
89132人目の素数さん:2011/01/29(土) 04:24:56
f(a,b,c)=(a^2 + b^2 + c^2)/(ab+bc+ca)は領域a>0,b>0,c>0 において連続である。
を証明すればいいんだけど、。。。
とちゅうでギャップが無いという保証がいる。
あたりまえであるようであまり意識しないもんだね。 一応多変数だからね
90Frank:2011/01/29(土) 04:34:05
>>88
わかりました。 ありがとうございます。
91132人目の素数さん:2011/01/29(土) 07:43:42
>>87
何が言いたい?
92132人目の素数さん:2011/01/29(土) 12:32:18
変なのがいろいろ湧きだしたな
無能は黙ってROMってろ!
93132人目の素数さん:2011/01/29(土) 16:15:09
>>88-89

 r = (1+1+c^2)/(1+c+c) = 1 + (3/4){(1+2c)/3 + 3/(1+2c) - 2}
どう見ても連続・・・・
94132人目の素数さん:2011/01/29(土) 16:25:14
79+88 で満点というわけか?
平均点はいくらぐらいなの?
95132人目の素数さん:2011/01/29(土) 17:20:31
79+88のように優雅ではないが、力ごなしにやると
Let define f(a,b,c)=(a^2 + b^2 + c^2)/(ab+bc+ca)
let define ff(a,b,t)=f(a,b,(a^2+b^2-2 ab cos(t))^(1/2))
let define fff(s,t)=ff(1,s,t) for 0<t<Pi, 1>= s >=0:from symmetry
and d(fff(s,t)z)/ds=(s-1){ ..positive...}==>monotone decreasing for s in [0,1] (for every t)
so
fff[1,t]={2+2-2cos(t)}/{1+2(2-2cos(t))^(1/2)}=(2+u^2)/(1+2u) here u=(2-2cos(t))^(1/2)

g[u]=(2+u^2)/(1+2u) for 0<u<4

Easily we get 2=g[4]=g[0]>g[u]>=g[1]=1

And fff[0,t]=2

The answer is [1,2]
96132人目の素数さん:2011/01/29(土) 17:26:45
[1, 2)
97132人目の素数さん:2011/01/29(土) 18:24:05
>>88-89

 r = (1+1+c^2)/(1+c+c) = 1 + (1-c)^2 /(1+2c),
でも同じだが・・・・・
98132人目の素数さん:2011/01/29(土) 19:12:30
>>81 2011年度 東工大特別入試

って普通の入試と違うの?
99132人目の素数さん:2011/01/29(土) 20:04:17
>>98
よう知らんが、センター試験より前に試験があったようだ
100132人目の素数さん:2011/01/29(土) 20:48:59
うかれば東工大合格というわけ?
101132人目の素数さん:2011/01/29(土) 21:09:56
ググレカス
102132人目の素数さん:2011/01/29(土) 21:14:02
[問題]
A, B を実 n 次の正定値対称行列とするとき、次の不等式を示せ。

det ( (A+B)/2 ) ≧ { detA ・det B }^{1/2}
103132人目の素数さん:2011/01/29(土) 22:29:52
>>102
こういう線形代数と微積分が合わさったような話題ってどんな本が詳しいですか?
線形代数も微積分も教養でやるけど、この手の話題って面白そうだけど意外と講義や演習でもやらない気がします。
行列の先の話題としてリー群の本は多いんですけど、書いて無いですよね。
104132人目の素数さん:2011/01/29(土) 23:46:50
微積?
102って相加相乗の拡張ぽいけど。
105132人目の素数さん:2011/01/30(日) 00:13:22
>>103
斎藤の線形代数演習
106132人目の素数さん:2011/01/30(日) 01:22:49
>>102
Q1=tx.A.x>0
Q2=tx.B. x>0 xはn次元ヴェクター

A または Bのいづれかが、正定あれば、(Aとする)
適当な T 正則マトリクス,が存在して
線形変換x=Tyにより

Q1=ty.E.ty
Q2=ty.L.y :L=対角マトリクス:Bも正定だからL>0


((Q1+Q2)/2)^2=(ty((E+L)/2)y)^2 =Sigma{i}(yi^2(1+Li))^2

Q1Q2=ty.y.ty.L.y=Sigma{i}yiLi

 Sigma{i}(yi^2(1+Li))^2 >= Sigma{i}yi^2Li
これから 
(Det(E+L)/2)^2>=Det(E)Det(L) 
つまり 
Det(A+B)/2>=(Det(A)Det(B))^(1/2)
107132人目の素数さん:2011/01/30(日) 01:41:14
x,y,z≧0のとき
(x^3+y^3+z^3)^4≧(x^4+y^4+z^4)^3
108132人目の素数さん:2011/01/30(日) 04:42:21
>>107

x^4 =X, y^4 =Y, z^4 =Z, 3/4 =p とおくと、与式は
 X^p + Y^p + Z^p ≧ (X+Y+Z)^p,
これは >>67 (2) の形だから、>>76 と同様にして示せる。
109132人目の素数さん:2011/01/30(日) 04:50:57
>>103
下らんごくごく普通の関数解析の本に、そのような話題は嫌というほど載っている。
もはやそれらは楽しいリー群(位相群)やフーリエ解析などに昇華されている。
110132人目の素数さん:2011/01/30(日) 05:07:33
>>103
ハーディー・リトルウッドの「不等式」がおすすめだ。
これがモデルになった関数解析(フーリエ解析)の本がある位だ。
111108:2011/01/30(日) 05:35:42
>>107 (訂正)

 これは、>>62 の形だから・・・・
112え(⌒▽⌒)?:2011/01/30(日) 10:34:05
y=e^x
113132人目の素数さん:2011/01/31(月) 00:26:04
〔問題〕
A, Bを2次の実対称行列とするとき、次の不等式を示せ。
 tr(exp(A+B)) ≦ tr(exp(A))・tr(exp(B)),


ただし、exp(X) = E + Σ[n=1,∞) (1/n!)X^n, (Eは単位行列)


(注意) 
A,Bが交換可能ならば等式になりますが、AB≠BA のときには一般に不等式になります。
この結果は一般にA,Bがn次対称行列のときにも正しいのですが、2次のときなら、腕づくで計算してもできます。


数セミ増刊「数学の問題 第2集」No.96, 日本評論社 (1978)
114132人目の素数さん:2011/01/31(月) 00:59:02
>>113

荒木不二洋 先生(元・京大、RIMS)の名作だな。

(;´д`) ハァハァ…
115132人目の素数さん:2011/01/31(月) 15:43:55
>>106
>適当な T 正則マトリクス,が存在して
>線形変換x=Tyにより
>
>Q1=ty.E.ty
>Q2=ty.L.y :L=対角マトリクス:Bも正定だからL>0

A,B は非可換だから、一般に同時対角化は出来ないのでは?
116132人目の素数さん:2011/01/31(月) 16:51:00
>>115
> A,B は非可換だから、一般に同時対角化は出来ないのでは?

はい。
A,B が同時対角化できるための必要十分条件は AB=BA(可換)なので、
>>106の証明は誤りです。
これが出来てしまうと>>113は等号になってしまいますから・・・
117132人目の素数さん:2011/01/31(月) 17:00:47
>>103
一般的には、作用素環のノルム不等式と言われる分野ですかね。
もちろん、行列環だけでなくもっと一般的な物を扱っています。
一般論はコンヌなどの非可換幾何などとも関係して難しいです。
118仙石60:2011/01/31(月) 17:22:17
>>113 
AB=BA ならば
Exp(A+B)=Exp(A)Exp(B) だから Tr(Exp(A+B))=Tr(Exp(A)Exp(B))=Tr(Exp(B)Exp(A))

Tr(Exp(A)Exp(B))=<Tr(Exp(A))Tr(Exp(B)) ですか?

 つまりTr(A.B)=<Tr(A)Tr(B) というわけですね?
119仙石60:2011/01/31(月) 17:33:14
>>116
 >>106の証明は誤りです。

実の対称マトリクスについては同時対角は可能ではないのですか?
(証明を見たようなきがするのだが。。。。自信ない)
A,Bがエルミートで一方が正定であれば同時対角化(Lamda,En)が可能であるというのは
よく使ったような気がする。(正定の定義が違うのかな?)
120132人目の素数さん:2011/01/31(月) 20:10:22
>>119
> 実の対称マトリクスについては同時対角は可能ではないのですか?

違います。

実際、AとBが同時対角化されたとします。
つまり、ある1つの直交行列 T が存在して、T^t A T, T^t B T が対角行列になる。

対角行列同士は交換可能なので、T^t A T と T^t B T は交換可能です。
つまり、
(T^t A T)・(T^t B T) = (T^t B T)・(T^t A T)
が成り立ちますが、Tは直交行列なので T^t T = Eより、
T^t AB T = T^t BA T
つまり,AB=BA となります。

しかし、任意の2つの正定値対称行列は交換可能とは限らないので、これは矛盾です。
121仙石60:2011/01/31(月) 20:28:27
対角化の意味ですが、
 E、と Lamda=固有値マトリクス のふたつを意味しているのですが。

122132人目の素数さん:2011/01/31(月) 20:43:42
>>121
何を言いたいのか全く分かりません。

>>103の証明を書いた >>106は以下の事実が間違っています。

A,Bを一般の正定値対称行列に対して

> Q1=tx.A.x>0
> Q2=tx.B. x>0 xはn次元ヴェクター
>
> A または Bのいづれかが、正定あれば、(Aとする)
> 適当な T 正則マトリクス,が存在して
> 線形変換x=Tyにより
>
> Q1=ty.E.ty
> Q2=ty.L.y :L=対角マトリクス:Bも正定だからL>0
123仙石60:2011/01/31(月) 20:46:31
正定マトリクスの定義 (x*)Ax>0 for all x not zero
2個の実対称マトリクスA、Bについての2次形式
Q1=TxAx,Q2=TXBX とする。
Aが正定ならば、適当な正則マトリクスによる線形変換x=Tyにより新変数yについて
2次形式がyの各成分の2乗項だけをふくむ
Q1=ty y、Q2=ty L y L:対角まとりくす
とあらわせる

証明 略
124仙石60:2011/01/31(月) 20:47:54
>何を言いたいのか全く分かりません。

チョット時間をおきませう
125132人目の素数さん:2011/01/31(月) 22:16:26
>>123
その主張が間違っているんですよ。

具体的には、同じ線形変換 x=Ty で、
Q1=ty y、Q2=ty L y 

と表される(つまり、Q_1=<y,y> )のは良いのですが、
「Lが対角行列」というのが間違いです。

これが出来るためには AB=BA でなくてはなりません。
理由は>>120です。
126仙石60:2011/01/31(月) 22:30:16
CLAIM:
BがC^mxn ならばA:=B*B:C^nxnは準正定マトリクスである。
逆に
エルミートマトリクスA:C^nxnが準正定ならば rank(B)=m (mはAの固有地の数)なるマトリクスB:C^mxn
をもちいてA=B*Bと表すことができる。

を証明せよ

A*=Aが成立するようなマトリクスをエルミートマトリクスという
A*はAの共役転置
127仙石60:2011/01/31(月) 22:39:06
Claim2
 2個の実対称マトリクス:R^nxn、B:R^nxnについての2次形式
Q1=txAx,Q2=txBx  x:R^n t:transpose

A ガ正定ならば、適当な正則マトリクスT:R^nxn による線形変換x=Ty
によって、新変数yについては2次形式がyの各成分の二乗項だけをふくむ
Q1=ty y、Q2=ty L y、 L:対角マトリクス
と表せる

を証明せよ
128仙石60:2011/01/31(月) 22:44:10
↑なお 一般にLの対角要素はBの固有地ではない。
129132人目の素数さん:2011/01/31(月) 23:07:07
仙石60さんよ、これ以上書くと恥晒しになるから止めとけや

130Fランク受験生:2011/01/31(月) 23:18:13
システム制御工学で使われるマトリクス演算手法ですね。
証明を期待しています。
131132人目の素数さん:2011/01/31(月) 23:23:47
こんなやつは、30年ぐらい前いっぱい流行したよねえ。
恥を晒して墓までよ 気にすることは無い。
132132人目の素数さん:2011/02/01(火) 00:09:59
正則行列と直交行列を混同しているような…
133仙石60:2011/02/01(火) 00:22:36
そうだね
134132人目の素数さん:2011/02/01(火) 01:33:17
>>128
糞コテめ!
さっさと消えろ!

我々のスレを荒らすな!
ここは不等式のスレだ、日本語読めるか? あん?

質問は質問スレでやれ!
135132人目の素数さん:2011/02/01(火) 02:16:43
120と123は何ら矛盾してない。この件については仙石が正しい。
136132人目の素数さん:2011/02/01(火) 20:11:43
【問題】
n次の実行列 A,B,C,D に対して,2n次の行列 X を
  X = [A B]
     [C D]
とおく.X が正定値対称行列のとき,

  det (X) ≦ det (A)・det(D)

を示せ.
137132人目の素数さん:2011/02/02(水) 13:21:13
Det(A) !=0 としてもよい。 A=tF.F
 Det(X)=Det(A)Det[D-B.A^-1.C]

X X が正定値対称行列-=> C=tB ,A=tF.F-->B.A^-1.C=B.A^-1.tB=B.(tF.F)^-1.tB
=B.(F)^-1.tF^-1.tB=B.F^-1.t(B.F^-1)
=Y.Ty : y:=B.F^-1
Det[D-B.A^-1.C]=Det[D-Y.tY]<=Det(D)
 det (X) ≦ det (A)・det(D)

q.e.d
138Fランク受験生:2011/02/02(水) 15:36:31
(1) A1>=B1,A2>=B2==>A1+A1>=B1+B2
(2) A>=B>=C ==> A >=C
(3)A1>=B1,A2>=B2 --->A1A2>=B1B2

間違っているのは(1)、(2)、(3)のどれでしょうか?

139132人目の素数さん:2011/02/02(水) 15:57:27
>>138
オコチャマは消えろ!
カーッ(゚Д゚≡゚д゚)、ペッ
140132人目の素数さん:2011/02/02(水) 16:14:29
↑ わからないとすぐそうやってごまかすんだから
141132人目の素数さん:2011/02/02(水) 16:19:29
なんかアホの巣窟になっちまったなw
142132人目の素数さん:2011/02/02(水) 16:24:41
猫がいなくなったから、猫に構ってた連中が持て余してんだろw
猫も必要悪ということだ。
143132人目の素数さん:2011/02/02(水) 16:34:19
(1),(2) OK
(3) マちが日
144132人目の素数さん:2011/02/02(水) 17:14:49
大学は試験シーズンだからな

ところで猫って大学の教員なの?
145132人目の素数さん:2011/02/02(水) 17:21:50
>>144
無職のルンペンだよ
146132人目の素数さん:2011/02/02(水) 18:14:26
一応非常勤をやっているんじゃなかったか?
147132人目の素数さん:2011/02/03(木) 01:09:18
>>113
まづ、exp( ) の定義から、
 exp(xE) = (e^x)E,
 AB = BA ⇒ exp(A+B) = exp(A)・exp(B),
 exp(A'+xE) = (e^x)・exp(A'),
 exp(B'+yE) = (e^y)・exp(B'),
ところで、
 A = (tr{A}/n)E + A',
 B = (tr{B}/n)E + B',
とおくと tr{A'} = tr{B'} =0 となる。
これらを上記に代入すると、本問は
tr{A'} = tr{B'} =0 ⇒ tr{exp(A'+B')} ≦ tr{exp(A')}・tr{exp(B')},
に帰着する。
148132人目の素数さん:2011/02/03(木) 01:27:19
>>113
A',B'は跡なしの2次行列だから、Cayley-Hamilton より
 (A')^2 = (a^2)E,  a = √(-det{A'}),
 (B')^2 = (b^2)E,  b = √(-det{B'}),
よって exp( ) の定義から
 exp(A') = cosh(a)・E + {sinh(a)/a}・A',
 exp(B') = cosh(b)・E + {sinh(b)/b}・B',
となる。また、
 tr{E} = n = 2,
 tr{A'} = tr{B'} = 0,
 tr{A'B'} = det{A'} + det{B'} - det{A'+B'} = 2ab・cosθ,
よって、
 tr{exp(A')exp(B')} = cosh(a)cosh(b)tr{E} + (・・・・)*tr{A'} + (・・・・)*tr{B'} + {sinh(a)sinh(b)/(ab)}tr{A'B'}
  = 2cosh(a)cosh(b) + 2sinh(a)sinh(b)・cosθ
  = (1+cosθ)cosh(a+b) + (1-cosθ)cosh(a-b)
  = (1+cosθ)f((a+b)^2) + (1-cosθ)f((a-b)^2)   (← *)
  ≧ 2f(a^2 + b^2 +2ab・cosθ)
  = 2f(-det{A'} -det{B'} +tr{A'B'})
  = 2f(-det{A'+B'})
  = tr{exp(A'+B')},

*)  f(t) = cosh(√t) = Σ[k=0,∞) (1/(2n)!)t^n, は t≧0 で下に凸。
149103:2011/02/03(木) 19:02:06
うわっ、凄い勢いでレスが伸びていてびっくり!!

>>104-105>>109-110>>117
文献の案内ありがとうございます。
作用素環へ繋がっていくのですね。
それは奥が深いわけだ。
150132人目の素数さん:2011/02/03(木) 23:28:08
>>138 >>143

(3)A1>=B1,A2>=B2 --->A1A2>=B1B2
ガ誤りであるような 実例を教えてください。
151132人目の素数さん:2011/02/04(金) 00:11:48
A1=A2=[1,1],B1=B2=[-10,-10]

A1,A2,B1,B2>0 の例をもとむ
152147-148:2011/02/04(金) 00:53:49
>>113
 ・・・・つまり、本題は次の補題に帰着した。

〔補題〕
2次の正方行列 A,B について 
 det(A) + det(B) - tr(AB) + tr(A)・tr(B) = det(A+B),

(略証) 成分を使って計算するだけ。
153132人目の素数さん:2011/02/04(金) 01:19:06
√2+√3>πを示せ
エレガントに頼むよ
154132人目の素数さん:2011/02/04(金) 13:18:51
√2+√3=3.14626...>π
155132人目の素数さん:2011/02/04(金) 13:21:32
エレガントを 求めるのは厨房
156Fランク受験生:2011/02/04(金) 13:43:44
>>152
〔補題〕
2次の正方行列 A,B について 
 det(A) + det(B) - tr(AB) + tr(A)・tr(B) = det(A+B),

計算しても等号がせいりつしません。
数値計算しても等号が成立しません。

どこがまちがっているのでしょうか
 det(A) + det(B) - tr(AB) + tr(A)・tr(B) ー det(A+B)=
a22 b11 + a23 a32 b11 + a33 b11 - a22 a33 b11 - a21 b12 - a23 a31 b12 +
a21 a33 b12 - a31 b13 + a22 a31 b13 - a21 a32 b13 - a12 b21 - a13 a32 b21 +
a12 a33 b21 + a33 b12 b21 - a32 b13 b21 + a11 b22 + a13 a31 b22 + a33 b22 -
a11 a33 b22 - a33 b11 b22 + a31 b13 b22 - a12 a31 b23 - a32 b23 +
a11 a32 b23 + a32 b11 b23 - a31 b12 b23 - a13 b31 + a13 a22 b31 -
a12 a23 b31 - a23 b12 b31 + a22 b13 b31 + a13 b22 b31 - a12 b23 b31 -
a13 a21 b32 - a23 b32 + a11 a23 b32 + a23 b11 b32 - a21 b13 b32 -
a13 b21 b32 + a11 b23 b32 + a11 b33 + a12 a21 b33 + a22 b33 - a11 a22 b33 -
a22 b11 b33 + a21 b12 b33 + a12 b21 b33 - a11 b22 b33
157132人目の素数さん:2011/02/04(金) 13:51:59
>>156
お前、受験板に帰れよ!
カーッ(゚Д゚≡゚д゚)、ペッ
158132人目の素数さん:2011/02/04(金) 13:54:04
147-148 は似非かはったりか?
159132人目の素数さん:2011/02/04(金) 14:51:36
新着レスがあると思ってみたら、便所の落書き未満だったり…
160132人目の素数さん:2011/02/04(金) 18:28:28
>>158
ということになります。
161132人目の素数さん:2011/02/04(金) 18:44:13
(π-x)^3=31 実数xをもとむ
 エレガントに頼むよ
162132人目の素数さん:2011/02/04(金) 19:14:14
156は何を計算してるのやら
2次の正方行列つってんだろが
163132人目の素数さん:2011/02/04(金) 20:39:09
[問題]
正定値対称行列 A,B に対して tA + (1-t)B (0≦t≦1) に対して、
t の関数 f(t) = det (tA + (1-t)B)^{-1} は下に凸であることを示せ。

   t det (A) + (1-t) det(B) ≦ det (tA + (1-t)B).
164132人目の素数さん:2011/02/04(金) 21:11:26
ばかばかし>>162
165132人目の素数さん:2011/02/04(金) 21:13:14
えらそうにしている割に程度低すぎ>>162
166132人目の素数さん:2011/02/04(金) 21:42:20
>>163
det (tA + (1-t)B). はn次の多項式
t det (A) + (1-t) det(B) はnの一次多項式

主張にムリガあるような気がするのだが?

たとえば t=〜0
(1-t) det(B) >=det ((1-t)B).kjなぜなら(1−t)>(1−t)^n

167132人目の素数さん:2011/02/04(金) 22:04:27
>>166
> たとえば t=〜0

すまんが、t=〜0 の意味を教えてもらえまいか?
168132人目の素数さん:2011/02/04(金) 22:12:53



          〜0



169132人目の素数さん:2011/02/04(金) 22:18:54
精子かYO!
170132人目の素数さん:2011/02/05(土) 00:51:07
>>163
det (tA + (1-t)B). はtのn次の多項式
t det (A) + (1-t) det(B) はtの一次多項式

主張にムリガあるような気がするのだが?

171132人目の素数さん:2011/02/05(土) 01:42:23
>>170
多項式の次数が違うことが、不等式が成立しないという根拠にはならんだろ。

例:実数 0< t <1 と自然数 n に対して、不等式 0< t^n < t は常に成立します。
172132人目の素数さん:2011/02/05(土) 01:45:16
というか凸函数におけるイェンセンの不等式を知らんのか?
173132人目の素数さん:2011/02/05(土) 02:48:24
>>170
多項式の次数が違うからといって、不等式が成立しないとは言ぇんぜん。
だな。
174163:2011/02/05(土) 03:46:34
>>170
確かに主張は間違っていました。
(多項式の次数が違うからというのは理由にはなりません)

[>>163の訂正]
関数 f(t) = det (tA + (1-t)B )^{-1} は逆行列を取っているので、
凸性を表す不等式は(この f が凸であることを示すのが問い)
  t f(1) + (1 -t) f(0) ≧ f(t)
これを書き換えると、
  t det (A^{-1}) + (1-t) det(B^{-1}) ≧ det (tA + (1-t)B)^{-1}
と逆数を取った式でした。

あるいは、最初から A, B を逆行列 A^{-1}, B^{-1} (これらも正定値対称行列)にして
 t det (A) + (1-t) det(B) ≦ det (t^{-1} A + (1-t)^{-1} B)
という不等式を得ます。
175132人目の素数さん:2011/02/05(土) 04:02:00
>>166 >>170

t=0,1 のとき 左辺は0ゆえ、因数定理から
 t・det(A) + (1-t)・det(B) - det{t・A + (1-t)・B} = t(1-t)f(t;A,B),


(例) n=2 の場合、
 t・det(A) + (1-t)・det(B) - det{t・A + (1-t)・B} = t(1-t)det(A-B),
176132人目の素数さん:2011/02/05(土) 06:13:39
>>166 >>170

t=0,1 のとき 左辺は0ゆえ、因数定理から
 t^n・det{A} + (1-t)^n・det{B} - det{t・A + (1-t)・B} = t(1-t)g(t;A,B),


(例) n=2 の場合、
 t^2・det{A} + (1-t)^2・det{B} - det{t・A + (1-t)・B} = t(1-t)(tr{AB}-tr{A}tr{B}),
177Fランク受験生:2011/02/05(土) 21:00:57
>実数 0< t <1 と自然数 n に対して、不等式 0< t^n < t は常に成立します。

(1)n>=3のばあいは
(2)実数 0< t <1 と自然数 n に対して、不等式 0< an t^n+an-1t^(n-1)+,,, < t は常に成立しますか?
178132人目の素数さん:2011/02/08(火) 14:47:14
掃除していたら、昔のメモが出てきた、懐かしす…

問1.レベル(目糞)---------------------

a、b、c、d≧0 のとき、次式を証明せよ

1/a + 1/b + 4/c + 16/d ≧ 64/(a+b+c+d)


問2.レベル(鼻糞)---------------------

x、y、z≧0、x+y+z=1 のとき、次式を証明せよ

0 ≦ xy + yz + zx -2xyz ≦ 7/27
179回答ー目糞:2011/02/08(火) 23:58:19
(1)P=1/a + 1/b + 4/c + 16/d - 64/(a + b + c + d)
   q=abcd(a+b+c+d)P の極点をもとめると( 11個)
{(a,b,c,d)=(0,0,c,0),(0,0,0,d),(-d,0,0,d),(a,0,0,0),(a,0,-a,0),....(d/4,d/4.d/2.d)}

どの極点の値もq=0になる。
qの形態から 是が最小値になる。
q>=0
180回答ー鼻糞:2011/02/09(水) 00:33:05
q=xy + yz + zx -2xyz-Lamda(x+y-1)
の局地をもとめて
(L,x,y,z)={(4/9,1/3,1/3,1/3),(1/2,0,1/2,1/2),...}
qの値は{7/27,1/4,1/4,1/4}になる。
境界の(x、y、z)=(1,0,0) でq=0
0≦ xy + yz + zx -2xyz ≦ 7/27
181132人目の素数さん:2011/02/09(水) 02:29:03
>>178

問1(目糞)
 {a, b, c/2, c/2, d/4, d/4, d/4, d/4} の8個で相加・調和平均あるいはコーシー
だな。

問2(鼻糞)
右側
 -1/2 ≦ 1/2 -x, 1/2 -y, 1/2 -z ≦ 1/2, より
 -(1/2)^3 ≦ (1/2 -x)(1/2 -y)(1/2 -z) < (1/2)^3,
上限は 3つの因子が同符号(正)のときで、相乗・相加平均より
  (1/2 -x)(1/2 -y)(1/2 -z) ≦ {(3/2 -x -y -z)/3}^3 = (1/6)^3,
これを展開する。左側は (ry
182132人目の素数さん:2011/02/09(水) 07:48:14
問3.レベル(耳糞)-------------------------------

実数 x、y、z が、x+y+z=0 をみたすとき、次式を証明せよ

(|a|+|b|+|c|)^2 ≧ 2(a^2 + b^2 + c^2)

-----------------------------------------------

※ 差を取るのはミジンコでもできるので、エレガントに証明してください
 ミジンコといえば…
http://www.yomiuri.co.jp/science/news/20110204-OYT1T00057.htm

※ さらに n乗の場合に拡張できるなら、お願いします
183132人目の素数さん:2011/02/09(水) 07:49:18
>>182
ごめん、a、b、cをx、y、zで読み替えてください
184132人目の素数さん:2011/02/11(金) 00:21:14
>>182-183

問3(耳糞)
 ミジンコだが、差を取ってみた。

a+b+c = 0 より 三角不等式が成り立つ。
よって
 (左辺) - (右辺) = 2|ab| + 2|bc| + 2|ca| - a^2 - b^2 - c^2
 = |a|(|b|+|c|-|a|) + |b|(|c|+|a|-|b|) + |c|(|a|+|b|-|c|) ≧ 0,
185184:2011/02/13(日) 02:46:47
>>183
 ごめん、a、b、cはn次元ベクトルだよね・・・・
186132人目の素数さん:2011/02/13(日) 05:53:59
>>153
あまりエレガントぢゃねぇが・・・・

6 > 6 - 2/(9^2) = (22/9)^2,
∴ √6 > 22/9,

(√2 + √3)^2 = 5 + 2√6 > 89/9 = (22/7)^2 + 5/(21^2) > (22/7)^2,
∴ √2 + √3 > 22/7,

ところで、
 (1/6)π^2 = ζ(2) < 5/3 - 1/48 < 5/3 - 1/49 = (1/6)(22/7)^2,
∴ 22/7 > π,

http://kamome.2ch.net/test/read.cgi/math/1287119136/187 , 199
東大入試作問者スレ19
187132人目の素数さん:2011/02/13(日) 06:02:07
〔問題〕
 (14/3)√(5/11) > √2 + √3 を示してくださいです。
188132人目の素数さん:2011/02/13(日) 06:24:28
>>187

 485 > (√484)(√486) = 22(9√6) = 99(2√6),
∴ 485/99 > 2√6,
∴ (14/3)^2・(5/11) = 980/99 > 5 + 2√6 = (√2 + √3)^2,
189132人目の素数さん:2011/02/13(日) 11:06:40
>>136
Xは正定値なので、小行列式 det(A) >0である。
とくに A は正則行列なので、行列 X は以下のように分解できる。
 [A B] = [ I   O][A B]
 [C D]   [CA^{-1} I][O D-CA^{-1}B].
両辺の行列式を取れば、
 det(X) = det(A)・det(D-CA^{-1}B)
ここで X は対称行列なので C=B^t で A^{-1} も正定値行列なので、
det(D-CA^{-1}B) = det (D - B^t A^{-1}B) ≦ det(D).
よって、
 det(X) ≦ det(A)・det(D)
が示された。
190132人目の素数さん:2011/02/13(日) 17:13:26
>>189
上手いなあ・・・
191132人目の素数さん:2011/02/13(日) 21:58:06
>>153
ちっともエレガントぢゃねぇが・・・・

6 > 6 - 2/(9^2) = (22/9)^2,
∴ √6 > 22/9,
∴ (√2 + √3)^2 = 5 + 2√6 > 89/9,

ところで、
 (1/6)π^2 = ζ(2) < 5/3 - 1/48 < 5/3 - 1/54 = (1/6)(89/9) < (1/6)(√2 + √3)^2,
∴ √2 + √3 > π,

http://kamome.2ch.net/test/read.cgi/math/1287119136/187 , 199
東大入試作問者スレ19
192132人目の素数さん:2011/02/14(月) 00:58:05
〔補題〕
3 + 1/8 < 31^(1/3) < π < 355/113 < 22/7 < √2 + √3 < √10,

http://kamome.2ch.net/test/read.cgi/math/1287119136/220-222
東大入試作問者スレ19

・3 + 1/8 < 31^(1/3) は
 (3 + 1/8)^3 = 27 +27/8 +9/64 +(1/8)^3 = 31 -(5 -9/8 -1/64)/8 < 31,

・355/113 = 3 + 16/113 < 3 + 16/112 = 3 + 1/7 = 22/7,

・22/7 < √2 + √3 は >>186

・√2 + √3 < √10 は
 y = √x は上に凸: √a + √b < 2√{(a+b)/2} = √{2(a+b)},
 あるいは √a + √b = √{2(a+b) - (√a -√b)^2} ≦ √{2(a+b)},
193132人目の素数さん:2011/02/14(月) 14:02:05
  π^6 = 945ζ(6) = 945*(1+1/2^6+1/3^6+...)
> 945*(1+1/2^6+1/3^6) = 945 + 945/64 + 35/27 = 945 + (14*64+48+1)/64 + {1+ (27+5)/(27*4)} = 961 + 1/64 + 5/108
> 961 = 31^2 = ([3]√31)^6
194132人目の素数さん:2011/02/15(火) 02:48:13
>>193
お見事でござる。それでは次を出さねば・・・・

〔補題〕
 31^(1/3) < 306^(1/5) < π

左側は
 31^3 = 29791 < 29800,
 31^5 < 29800*31*31 = 298*310*310 < 306^3, (相乗・相加平均)

ハァハァ
195132人目の素数さん:2011/02/16(水) 21:56:14
このレベルの不等式を示すのに、(1/6)π^2 = ζ(2) を使うのはナンセンス。
証明すべきことより、(1/6)π^2 = ζ(2) の証明の方が遥かに難しく、証明になっとらんわw

ていうか、ここいつから大学入試問題スレになったんだよw
糞みたいな問題ばっかりで、詰まらない。

受験生は受験板でやれや

どうせ sin の無限積展開とか知らん奴は (1/6)π^2 = ζ(2) の事実を使うな!
196132人目の素数さん:2011/02/16(水) 21:57:24
>>191-194
巣に戻れやボケ!
197132人目の素数さん:2011/02/16(水) 22:23:34
君こそ消えたまえ!
198132人目の素数さん:2011/02/16(水) 23:09:57
sin(23^23)>1/2
199132人目の素数さん:2011/02/17(木) 06:26:46
>>197
あんたこそスレ違い
ここは受験問題のスレじゃない!

しかも、他スレのコピペだし
200132人目の素数さん:2011/02/17(木) 12:29:35
157 :132人目の素数さん:2011/02/04(金) 13:51:59
>>156
お前、受験板に帰れよ!
カーッ(゚Д゚≡゚д゚)、ペッ
201132人目の素数さん:2011/02/17(木) 14:14:52
どうして荒れるのかなぁ…
初代スレからの住人としては悲しい限り
202132人目の素数さん:2011/02/17(木) 14:33:46
このスレは今まで荒れることがなかったから耐性ないねw
スレの数が減ったために目につきやすくなったからある程度は仕方ないね。
スルーしとけばいいと思うよ。
203132人目の素数さん:2011/02/18(金) 13:53:48
>>201
入試問題ばかりになっちまったからだろ

スレ住人が塾講師の連中ばかりになっちまったというよ。
204132人目の素数さん:2011/02/18(金) 22:38:59
>>198 お願いします
205132人目の素数さん:2011/02/19(土) 21:50:20.35
206132人目の素数さん:2011/02/20(日) 00:42:17.01
>>194
このレベルの不等式を示すのに (1/93555)π^10 = ζ(10) を使うのはナンセンス。
証明すべきことより、(1/93555)π^10 = ζ(10) の事実の方が遥かに難しいから、証明になっとらんわw
とは思いつつ・・・・

 π^10 = 93555ζ(10)
  = 93555・{1 + (1/2)^10 + ・・・・}
  > 93555・(1 + 1/1024)
  > 93555・(1 + 1/1155)
  = 93555 + 91
  = 93646
  = 306^2,
207206:2011/02/20(日) 03:54:59.36
>>194 (訂正)

π^10 = ・・・・
 = ・・・・・
 > 93555・(1 + 1/1155)
 = 93555 + 81
 = 93636
 = 306^2,

∴ 306^(1/5) < π
208132人目の素数さん:2011/02/20(日) 04:19:51.61
>>192

y=1/x^6 は下に凸だから
 1/k^6 < ∫[k-0.5, k+0.5] 1/x^6 dx = (1/5){1/(k-0.5)^5 - 1/(k+0.5)^5},
これを使って
 (1/945)π^6 = ζ(6)
  = 1 + 1/2^6 + 1/3^6 + ・・・・
  < Σ[k=1,n] 1/k^6 + ∫[n+1/2,∞) 1/x^6 dx
  = 1 + 1/2^6 + 1/3^6 + 1/4^6 + 1/5^6 + 1/6^6 + 1/(5*6.5^5) (n=6 とおく)
  = 1 + (3^6 + 2^6 + 1)/(6^6) + 1/4^6 + 1/5^6 + 32/(5*13^5)
  = 1 + 794/(6^6) + 1/4^6 + 1/5^6 + 32/(5*13^5)
  < 1 + (35431020482 + 508289003 + 133244913 + 35886729)/(113^6)
  = 1 + 36108441127/(113^6)
  = 1 + 34122476865015/(945・113^6)
  < 1 + 34122530050120/(945・113^6)
  = (1/945)(355/113)^6,

∴ π < 355/133 ・・・・・・ 密率(「隋書」)

祖沖之(429-500)
209208:2011/02/20(日) 04:33:00.68
>>192
またまた訂正・・・・

 π < 355/113 ・・・・・・ 密率
 π < 22/7   ・・・・・・ 約率
210132人目の素数さん:2011/02/20(日) 07:58:37.53
222/77 < π < 22/7
211132人目の素数さん:2011/02/20(日) 08:02:12.86
3555/1133 < π < 355/113
212132人目の素数さん:2011/02/20(日) 11:38:15.55
〔問題649〕
 a,b,c>0 のとき a+b+c=s とおくと
 a/√(s-b) + b/√(s-c) + c/√(s-a) < (5/4)√s,


casphy - 高校数学 - 不等式スレ 330, 375-376
→ 前スレ.649
213132人目の素数さん:2011/02/20(日) 23:44:56.31
>>195
 正n角形とその内接円・外接円を使うのは、面倒な割に、精度がいまいち・・・・
 ζ函数の方が効率がいいし・・・
 他にいい方法がないかと思う今日この頃・・・
214132人目の素数さん:2011/02/26(土) 05:15:12.79
今年の京大

1/2<a[i]<1のとき
Π(1-a[i])>1-2Σ(a[i]/2^i)
(i=2,3,…n)
215132人目の素数さん:2011/02/27(日) 07:19:26.19
>>214

左辺を P[n] とおくと、
 P[n] = P[n-1]・(1-a[n])
   = P[n-1] - a[n]・P[n-1]
   = ・・・・・
   = 1 - Σ[i=1,n] a[i]・P[i-1]

 P[0] = 1,
 P[i] < 1/(2^i)   (i≧1)
から。

http://nyushi.yomiuri.co.jp/11/sokuho/kyoto/zenki/sugaku_ri/images/mon4_1.gif

http://kamome.2ch.net/test/read.cgi/math/1287119136/253
東大入試作問者スレ19
216132人目の素数さん:2011/02/27(日) 10:20:07.83
〔問題〕
 0 < x < π のとき 次を示せ。
(1) 1/sin(x) - 1/x > x/6,
(2) {1/sin(x)}^2 - 1/x^2 > 1/3,
217132人目の素数さん:2011/02/28(月) 03:57:23.98
今年の阪大で不等式の問題が出たが、難問だったらしい
218132人目の素数さん:2011/02/28(月) 06:32:54.58
>>217
+   +
  ∧_∧  +
 (0゚・∀・)    ワクワク、テカテカ…
 (0゚∪ ∪ +
 と__)__) +
219132人目の素数さん:2011/02/28(月) 10:19:21.16
220Fランク受験生:2011/03/03(木) 00:49:07.67
内容は簡単だと思うけど計算間違いをしているかも。。。

(1)
S(a)=(1/2)(a^2+1)^(n-1)
(2)
a=4 のときx=1.76になる
221132人目の素数さん:2011/03/03(木) 23:48:54.04
>>217
全然難問じゃないじゃん。
大数で言うと C*** クラス。
222132人目の素数さん:2011/03/04(金) 02:52:40.02
↑ 解いてから言え
223132人目の素数さん:2011/03/04(金) 08:37:16.00
新参はまず過去レスを読んでこのスレの空気を知ってほしいね。
224132人目の素数さん:2011/03/04(金) 08:47:21.33
数人の自演スレだからなぁ
225132人目の素数さん:2011/03/04(金) 09:37:23.59

   / ̄ ̄\     
 /   _ノ  \  
 |   ( ●)(●)  <おっとそこまでだ
. |     (__人__)____
  |     ` ⌒/ ─' 'ー\
.  |       /( ○)  (○)\
.  ヽ     /  ⌒(n_人__)⌒ \
   ヽ   |、    (  ヨ    |
   /    `ー─−  厂   / ←>>224
   |   、 _   __,,/     \
226132人目の素数さん:2011/03/04(金) 10:49:02.96
その後224の行方を知る者は誰もいなかった。
227132人目の素数さん:2011/03/05(土) 12:42:07.49
228132人目の素数さん:2011/03/07(月) 21:32:08.20
もう工房の入試問題スレになっちまったな・・・orz
229132人目の素数さん:2011/03/07(月) 21:46:33.46
ageんな
230猫は廃人 ◆MuKUnGPXAY :2011/03/08(火) 13:50:17.70

231猫はボケ老人 ◆MuKUnGPXAY :2011/03/08(火) 19:29:40.79

232132人目の素数さん:2011/03/08(火) 19:36:06.98
猫は小便垂れ流し
233猫はボケ老人 ◆MuKUnGPXAY :2011/03/08(火) 19:40:50.01
目的を達成スル為であれば小便でも何でも垂れ流しますワ。


234132人目の素数さん:2011/03/14(月) 01:35:56.47
「不等式」大関清太
http://www.kyoritsu-pub.co.jp/series/kandokoro.html#zokkan

  ∧_∧
  ( ;´∀`) < こ、こりゃたまらんっ!
  人 Y /
 ( ヽ し
 (_)_)
235132人目の素数さん:2011/03/17(木) 15:37:44.25
a,b,c,dをabcd=1を満たす正の実数とするとき,
(a-1)(3a-7)+(b-1)(3b-7)+(c-1)(3c-7)+(d-1)(3d-7)≧0
を証明せよ。
236132人目の素数さん:2011/03/19(土) 01:36:36.10
>>235

 (左辺) = f(log(a)) + f(log(b)) + f(log(c)) + f(log(d)),
ここで
 f(x) = (e^x - 1)(3e^x - 7) = 3(e^x - 5/3)^2 - 4/3 ≧ -4/3,
とおいた。

 f "(x) = 12{e^x - (5/6)}e^x > 0,   (x≧0)
ゆえ、
 x≧0 では f は下に凸。
 f '(0) = -4,
 k = -0.64298265 = log(0.5257220384) < x < 0 では f(x) > -4x,  
そこで
 F(x) = f(x),    x < k, 0 < x
    = -4x,    k ≦ x ≦ 0
とおく。(函数凸包、function convex hull)
 F(x) は x ≧ k で下に凸(広義)である。

(1) a,b,c,d ≧ e^k のとき、凸不等式より
 (左辺) ≧ F(log(a)) + F(log(b)) + F(log(c)) + F(log(d))
    ≧ 4F(log(abcd)/4) = 4F(0) = 4f(0) = 0,

(2) a,b,c,d の中に e^k より小さいものが1つだけある(a)とき、凸不等式より
 f(log(b)) + f(log(c)) + f(log(d)) ≧ F(log(b)) + F(log(c)) + F(log(d))
  ≧ 3F(log(bcd)/3) = 3F(-log(a)/3) = 3f(-log(a)/3)
 (左辺) ≧ f(log(a)) + 3f(-log(a)/3) ≧ f(k) + 3f(-k/3) = 0.21780074,

(3) a,b,c,d の中に e^k より小さいものが2つ以上ある(a,b)とき
  f(log(a)) ≧ f(log(b)) ≧ 2.5719306, f(log(c))≧-4/3, f(log(d))≧-4/3,
 により成立。
237132人目の素数さん:2011/03/22(火) 06:15:56.83
今月は不等式が一杯載っている

Problem 365.
http://www.math.ust.hk/excalibur/v15_n4.pdf
238132人目の素数さん:2011/03/22(火) 06:18:57.91
239132人目の素数さん:2011/03/25(金) 23:48:58.94
>>237

Problem 365.
負でない実数 a,b,c が ab+bc+ca = 1 を満たすとき、
 1/(a+b) + 1/(b+c) + 1/(c+a) - 1/(a+b+c) ≧ 2,
を示せ。
240132人目の素数さん:2011/03/26(土) 01:40:10.75
>>239
俺もこれが面白いと思った
まだ解けぬ〜
241132人目の素数さん:2011/03/27(日) 23:30:24.83
>>239-240

 附帯条件から考えて、a→cotα, b→cotβ, c→cotγ と置いてみる・・・
 α+β+γ = π より,
 (左辺) = 1/(cotα+cotβ) + 1/(cotβ+cotγ) + 1/(cotγ+cotα) - 1/(cotα+cotβ+cotγ)
  = (sinα・sinβ)/sin(α+β) + (sinβ・sinγ)/sin(β+γ) + (sinγ・sinα)/sin(γ+α) - (sinα・sinβ・sinγ)/(1-cosα・cosβ・cosγ)
  = (sinα・sinβ)/sinγ + (sinβ・sinγ)/sinα + (sinγ・sinα)/sinβ -2(sinα・sinβ・sinγ)/[(sinα)^2 + (sinβ)^2 + (cosγ)^2]
  = (sinα・sinβ・sinγ){(1/sinα)^2 + (1/sinβ)^2 + (1/sinγ)^2 - 2/[(sinα)^2 + (sinβ)^2 + (cosγ)^2]}
  = 2勍{(1/a')^2 + (1/b')^2 + (1/c')^2 -2/[(a')^2 + (b')^2 + (c')^2]}
  = ・・・・
ここに、 a'=2R・sinα, b'=2R・sinβ, c'=2R・sinγ, = 2R^2・sinα・sinβ・sinγ,

まだ解けぬるぽ〜
242132人目の素数さん:2011/03/28(月) 07:26:05.27
1/x+y+1/y+z+1/z+x-1/x+y+z
=1/x+y+z*(z/x+y+x/y+z+y/z+x)+2/x+y+z
>=1/x+y+z*(x+y+z)^2/2(xy+yz+zx)+2/x+y+z
=x+y+z/2+2/x+y+z>=2 Q.E.D.
243132人目の素数さん:2011/03/28(月) 08:37:32.00
>>242
エスパー検定3級の俺には、どれが分母なのか読み取れねぇ・・・
244132人目の素数さん:2011/03/28(月) 09:52:08.74
>242
2行目から3行目に何を使ったのか分からない

1/(x+y) + 1/(y+z) + 1/(z+x) - 1/(x+y+z)
= 1/(x+y) + 1/(y+z) + 1/(z+x) - 3/(x+y+z) + 2/(x+y+z)
1/(x+y+z)で括る
= {1/(x+y+z)}*{(x+y+z)/(x+y) + (x+y+z)/(y+z) + (x+y+z)/(z+x) - 3} + 2/(x+y+z)
= {1/(x+y+z)}*{1 + z/(x+y) + 1 + x/(y+z) + 1 + y/(z+x) - 3} + 2/(x+y+z)
= {1/(x+y+z)}*{z/(x+y) + x/(y+z) + y/(z+x)} + 2/(x+y+z)
後は頼んだ>243
245243:2011/03/28(月) 10:52:27.44
では、引き継いで頑張ってみます

3(x^2 + y^2 + z^2) - (x+y+z)^2 = (x-y)^2 + (y-z)^2 + (z-x)^2 ≧ 0

∴(x^2 + y^2 + z^2) ≧ (1/3)*(x+y+z)^2

z/(x+y) + x/(y+z) + y/(z+x)
= z^2/(zx+yz) + x^2/(xy+zx) + y^2/(yz+xy)
≧ z^2/(xy+yz+zx) + x^2/(xy+yz+zx) + y^2/(xy+yz+zx)
= (x^2 + y^2 + z^2)
≧ (1/3)*(x+y+z)^2

>>242の4行目から

= {1/(x+y+z)}*{z/(x+y) + x/(y+z) + y/(z+x)} + 2/(x+y+z)
≧ (x+y+z)/3 + 2/(x+y+z)
≧ 2√(2/3)

うむ、失敗したようじゃ…
246243:2011/03/28(月) 10:56:30.96
>>245

>>242の4行目から → >>244の下から2行目から

だけど、もはやどうでもいい…
247132人目の素数さん:2011/03/29(火) 02:54:50.22
>>241
 鋭角△に限定しなければならぬ〜

 (a')^2 + (b')^2 ≧ (c')^2, etc.
248132人目の素数さん:2011/04/03(日) 15:07:21.35
>>238

M1852.
 f∈ C^1([0,1]) で f(0) = f(1) = -1/6. のとき次を示せ。

 ∫[0,1] {f '(x)}^2 dx ≧ 2∫[0,1] f(x) dx + 1/4,
249132人目の素数さん:2011/04/03(日) 15:38:26.07
>>248

部分積分により
 (右辺) = 2[ (x - 1/2)f(x) ](x=0,1) -2∫[0,1] (x - 1/2)f '(x)dx + 1/4
   = f(0) + f(1) + 1/4 -2∫[0,1] (x - 1/2)f '(x) dx
   = -1/12 -2∫[0,1] (x - 1/2)f '(x) dx,

  ∫[0,1] (x - 1/2)^2 dx = [ (1/3)(x - 1/2)^3 ](x=0,1) = 1/12,

よって
 (左辺) - (右辺) = ∫[0,1] {f '(x) + (x - 1/2)}^2 dx ≧ 0,
250132人目の素数さん:2011/04/03(日) 15:43:12.09
>>249
x - 1/2 という発想はどこから?
251132人目の素数さん:2011/04/03(日) 18:18:19.82
>>238

C925.
 f ∈ C^2([0,1]) で
 ∫[0,1] f(x)dx = 2∫[1/4,3/4] f(x)dx,
のとき、f "(x0) =0 を満たす点 x0 ∈ (0,1) が存在することを示せ。


C932.
 f : [0,1] → R は連続関数
 ∫[0,1] {f(x)}^3 dx = 0,
のとき、次を示せ。
 ∫[0,1] {f(x)}^4 dx ≧ (27/4){∫[0,1] f(x)dx}^4,


C944.
 f ∈ C^1([0,1])
 ∫[0,1] f(x) dx = 0,
 A ≦ f '(x) ≦ B   x∈[0,1]
のとき、次を示せ。
 A ≦ 12∫[0,1] x・f(x)dx ≦ B,
252132人目の素数さん:2011/04/03(日) 18:49:37.86
>>251

C925.
 (左辺) - (右辺)
  = ∫[0,1/4] f(x)dx - ∫[1/4,1/2] f(x)dx - ∫[1/2,3/4] f(x)dx + ∫[3/4,1] f(x)dx
  = ∫[0,1/4] {f(x) -f(x+1/4) -f(x+1/2) +f(x+3/4)} dx
  = ∫[0,1/4] g(x) dx
平均値の定理より
  = (1/4)g(a)         (0<a<1/4)
  = (1/4){f(a) -f(a+1/4) -f(a+1/2) +f(a+3/4)}
  = (1/16){f '(b) - f '(c)}  (a<b<a+1/4, a+1/2<c<a+3/4)
  = (1/16)(b-c)f "(x0),    (b<x0<c)
253132人目の素数さん:2011/04/04(月) 01:51:35.18
>>239-240
しょうがねぇなぁ・・・・

基本対称式を a+b+c = s, ab+bc+ca = t, abc = u とおくと
 (左辺) = (s^2 +t)/(st-u) -1/s = (s^3 + u)/{s(st-u)},
 (右辺) = 2/√t,

 (s^3 + u)^2 - (4/t){s(st-u)}^2 = (s^3)F_1 + (s^2)(u/t)(st-4u) + u^2 ≧ 0,
ここに
 F_1 = a(a-b)(a-c) + b(b-c)(b-a) + c(c-a)(c-b) = s^3 -4st +9u ≧ 0,  (Schurの不等式)

やっと解けた。しかし、めんどくせぇなぁ・・・
254132人目の素数さん:2011/04/04(月) 03:44:39.82
http://d.hatena.ne.jp/wasabiz/20110403
なんかみつけたけん
255132人目の素数さん:2011/04/04(月) 08:24:10.34
>>254
よくある間違い。
1<2と100≦100で等号が成り立つから1より100の方が小さいとやってしまう。
a=b−>0とすれば下の式は2より小さくなるから駄目。
256132人目の素数さん:2011/04/04(月) 08:37:51.12
>>255
> 1<2と100≦100で等号が成り立つから1より100の方が小さいとやってしまう。

言っている意味が分からないが、>>254が初歩的なミスをしていることは同意。
例えば、次のように説明すると分かりやすいかも?
>>254の主張を、グラフで視覚化してみよう!

y=x^2 と y=2x-1 において、x^2 ≧ 2x-1 が成り立つ。
等号成立条件は x=1のときで、このとき右辺は 2・1-1=1だから、x^2≧1

どう考えてもおかしいよね (・A・)イクナイ!
257132人目の素数さん:2011/04/04(月) 09:04:09.55
a=b=c=1/3^(1/2)
1/(a+b)+1/(a+c)+1/(b+c)-1/(a+b+c)=7*3^(1/2)/6=2.0207259421
7/(3((a+b)(a+c)(b+c))^(1/3))=7*3^(1/2)/6=2.0207259421

a=b=0.1,c=4.95
1/(a+b)+1/(a+c)+1/(b+c)-1/(a+b+c)=5.2018648466
7/(3((a+b)(a+c)(b+c))^(1/3))=1.3555198072

2.0207259421>=2.0207259421>=2
5.2018648466>=1.3555198072>=2
258132人目の素数さん:2011/04/04(月) 10:01:03.59
>>253
等号成立は0,1,1のときしかないんだよね?
259132人目の素数さん:2011/04/05(火) 01:02:06.67
>>244の最後からね
{1/(x+y+z)}*{z/(x+y) + x/(y+z) + y/(z+x)} + 2/(x+y+z)
={1/(x+y+z)}*{z/(x+y) + x/(y+z) + y/(z+x)} (xy+xz+yz)+ 2/(x+y+z)
=1/2*({1/(x+y+z)}*{z/(x+y) + x/(y+z) + y/(z+x)} (z(x+y)+x(y+z)+y(x+z)))+ 2/(x+y+z)
>=1/2*({1/(x+y+z)}*(x+y+z)^2+ 2/(x+y+z)
=(x+y+z)/2+ 2/(x+y+z)>=2
260239:2011/04/05(火) 01:47:07.55
>>259 >>244
 お見事でござる。
 コーシー不等式 z/(x+y) + x/(y+z) + y/(z+x) ≧ (s^2)/(2t) がミソだった。

>>258
 そうでつね。
261132人目の素数さん:2011/04/05(火) 02:01:12.29
>>242
が言いたかったことが やっと分かった。
{[( )]}をたくさん使ってくれると ありがたいです。
262 ◆BhMath2chk :2011/04/06(水) 12:00:00.55
>>248
>>250
 ∫_[0,1]((df/dx)(x)−g(x))^2dx
=∫_[0,1](df/dx)(x)^2dx+∫_[0,1]2(dg/dx)(x)f(x)dx+∫_[0,1]g(x)^2dx+2(f(0)g(0)−f(1)g(1))。

(dg/dx)(x)=−1,(df/dx)(x)=g(x)となるfが存在するようにgをとるとg(x)=−x+1/2。
263 ◆BhMath2chk :2011/04/06(水) 12:59:59.72
>>251
A=∫_[0,1]f(x)dx。
 ∫_[0,1](f(x)^2+3Af(x)−(9/2)A^2)^2dx
=∫_[0,1]f(x)^4dx+6A∫_[0,1]f(x)^3dx−(27/4)A^4。

 12∫_[0,1]xf(x)dx
=∫_[0,1](12x−6)f(x)dx
=[(6x^2−6x)f(x)]_0^1−∫_[0,1](6x^2−6x)(df/dx)(x)dx
=∫_[0,1](6x−6x^2)(df/dx)(x)dx
=∫_[0,1](6x−6x^2)dx(df/dx)(s) (0<s<1)
=(df/dx)(s)。
264132人目の素数さん:2011/04/09(土) 09:02:08.28
a、b、c≧0のとき、(a^3 + b^3 + c^3)^4 ≧ (a^4 + b^4 + c^4)^3 を示せ

前にやったっけ?( ゚∀゚)
265132人目の素数さん:2011/04/09(土) 13:52:52.11
バンチで
266132人目の素数さん:2011/04/10(日) 09:44:59.54
【数学検定】数検・児童数検総合スレッド Part.4
http://kamome.2ch.net/test/read.cgi/math/1284438189/
の4,45

模範解答はa=b≧cの場合が抜けている。


>>107

267132人目の素数さん:2011/04/10(日) 18:56:36.30
>>264 >>107

>>76 の方法でござるな・・・・  >>111

 a^4 = a^3・a ≦ a^3・(a^3 +b^3 +c^3)^(1/3),
巡回的にたすと
 a^4 + b^4 + c^4 ≦ (a^3 +b^3 +c^3)^(4/3),

 a^3 = a^4 /a ≧ a^4 /(a^4 +b^4 +c^4)^(1/4),
巡回的にたすと
 a^3 + b^3 + c^3 ≧ (a^4 +b^4 +c^4)^(3/4),

あるいは Max{a,b,c}=M とおいて
 a^4 + b^4 + c^4 ≦ M(a^3 +b^3 +c^3),
 (a^4 +b^4 +c^4)^3 ≦ (M^3)(a^3 +b^3 +c^3)^3 ≦ (a^3 +b^3 +c^3)^4,

>>266

http://kamome.2ch.net/test/read.cgi/math/1284438189/44-45
 数検総合スレ4

http://www.suken.net/gakushu/sample/index.html → 1級 検定問題(2次)
http://www.suken.net/gakushu/sample/sample_img/1-5/1kyu_mondai_2ji.pdf
268132人目の素数さん:2011/04/12(火) 10:48:14.35
a, b, c を実数とするとき, (a^2+b^2+c^2)^2≧3(a^3b+b^3c+c^3a)を証明せよ。
269じゅー:2011/04/12(火) 21:29:19.35
>>268
キャスフィ高校数学板 - 不等式 - 517,519,523
…………
ではキャスフィからもう一題。
a,b,cを三角形の三辺とするとき、
a^3+b^3+c^3+3abc
≧2(ab^2+bc^2+ca^2)
を示せ。
キャスフィ高校数学板 - チャレンジ問題 - 60
270訂正:2011/04/12(火) 21:34:07.11
523→522
60→59
に訂正です。
271132人目の素数さん:2011/04/13(水) 21:45:47.80
>>269
|a-b|<cの両辺を2乗して変形し
a^2+b^2-c^2<2ab
ca^2+cb^2-c^3<2abc
同様に
ab^2+ac^2-a^3<2abc
bc^2+ba^2-b^3<2abc
足して
ca^2+cb^2+ab^2+ac^2+bc^2+ba^2<a^3+b^3+c^3+6abc
ab^2+ bc^2+ca^2+3abc<a^3+b^3+c^3+6abc
ab^2+ bc^2+ca^2<a^3+b^3+c^3+3abc

失敗した…
272132人目の素数さん:2011/04/14(木) 08:28:54.62
失敗かよ!
273 ◆BhMath2chk :2011/04/14(木) 18:00:00.06
p=(b+c−a)/2。
q=(a+c−b)/2。
r=(a+b−c)/2。
a=q+r。
b=p+r。
c=p+q。

 a^3+b^3+c^3+3abc−2(ab^2+bc^2+a^2c)
=2(pq^2+p^2r+qr^2−3pqr)
≧0。
274じゅー:2011/04/14(木) 21:33:49.09
正解!!
275132人目の素数さん:2011/04/14(木) 21:55:29.13
276271:2011/04/14(木) 22:28:05.71
>>273
すげー!

このスレ見てたら、不等式に魅了されたよ

>>275
相加相乗
277132人目の素数さん:2011/04/14(木) 23:26:13.93
>>273みたいなアクロバティックな変形は思いつかなかったので・・・

a^3-a^2c+abc-ab^2=a(a^2-ac-bc-b^2)=a(a-b)(a+b-c)>a(a-b)
b^3-b^2a+abc-bc^2=b(b^2-ba+ac-c^2)=b(b-c)(b+c-a)>b(b-c)
c^3-c^2b+abc-ca^2=c(c^2-cb+ab-a^2)=c(c-a)(c+a-b)>c(c-a)
全部足して
=a^3+b^3+c^3+3abc-2(ab^2+bc^2+ca^2)>a(a-b)+b(b-c)+c(c-a)
=a^2-ab+b^2-bc+c^2-ca={(a-b)^2+(b-c)^2+(c-a)^2}/2≧0

よって(左辺)-(右辺)≧0


むーん
278277:2011/04/14(木) 23:31:35.12
符号打ち間違えた
×a^3-a^2c+abc-ab^2=a(a^2-ac-bc-b^2)=・・・
○a^3-a^2c+abc-ab^2=a(a^2-ac+bc-b^2)=・・・

273はいったいどういう発想でその置換を思いついたのだろう
レベルの低いおいらにはわかんないや
279132人目の素数さん:2011/04/15(金) 12:42:51.39
p,q,rはヘロンの公式に出てくる量だから、三角形という条件がある場合には、
全く新しい発想というわけではないと思う。
目的関数が、非対称なので、コーシー・シュワルツ形へ持って行くのかと思っていたが、
[3]√((p/q)(q/r)(r/p))形を通して、相加相乗形へ持って行ったのには、感心した。
280132人目の素数さん:2011/04/16(土) 00:58:02.78
p>0,x[i]≧0のとき

min{1,n^(p-1)}Σ[i=1→n](x[i])^p
≦(Σ[i=1→n]x[i])^p
≦max{1,n^(p-1)}Σ[i=1→n](x[i])^p
281132人目の素数さん:2011/04/17(日) 11:42:27.80
流れてしまった春の学会で話そうとしていた内容をUPしておきました。
http://www.math.s.chiba-u.ac.jp/~ando/ineq.pdf
282132人目の素数さん:2011/04/17(日) 14:12:27.19
@273

a=x+y, b=y+z, z=x+yとおけば十分,
Σ_cyc (x^2y-xyz)≧0 Q.E.D.
283132人目の素数さん:2011/04/17(日) 20:03:58.97
>>269
 F_1 = (a+b+c)^3 - 4(a+b+c)(ab+bc+ca) + 9abc,
  = (a-b)(b-c)(c-a),
とおくと、
 F_1 ≧ ,
284132人目の素数さん:2011/04/18(月) 12:55:14.00
(a^2+b^2+c^2)^2-3(a^3b+b^3c+c^3a)

=(1/2){(a^2-b^2-ab++2bc-ca)^2+(b^2-c^2-bc+2ca-ab)^2+(c^2-a^2-ca+2ab-bc)^2}
285じゅー:2011/04/18(月) 15:27:18.36
すげぇ
286132人目の素数さん:2011/04/18(月) 17:17:34.76
どうやって思いついたんだ??
287132人目の素数さん:2011/04/18(月) 21:12:59.72
 二 `丶、`丶、_\__\〉ノノへ!
`‐-、 二. `ヽ、 ミ ̄ /⌒シ′)
二‐/,ィ┐|=ミ=┘ ,r‐'_二ニ....イ
‐ニ| i<  i ,..-=ニ‐''\  /彡}   
二‐ヽ ┘ |     lヾ. } } / /リ
ニ ‐'"/   /    |_{;)} レ' /((   エレガントな証明を見ると・・・・・
'  /   /     '" ` `゙ / ソ
  /    ,      F'′/    なんていうか・・・・・・その・・・
  ヽ.    \、 L`___l       
 _\    ヽ._>┘         下品なんですが・・・・・・フフ・・・・・
 /了\_ノ
 ◆(                 勃起・・・・・・しちゃいましてね・・・・・・・・・
 門|
288132人目の素数さん:2011/04/19(火) 01:17:01.91
>>286

 p = a^2 -ab +bc,
 q = b^2 -bc +ca,
 r = c^2 -ca +ab,
とおくと、
 p + q + r = a^2 + b^2 + c^2,
 pq + qr + rp = a^3・b + b^3・c + c^3・a,
これらを↓に代入する。
 (p+q+r)^2 -3(pq+qr+rp) = (1/2){(p-q)^2 + (q-r)^2 + (r-p)^2} ≧ 0,
289132人目の素数さん:2011/04/19(火) 03:02:58.17
>>288
>  p = a^2 -ab +bc,
>  q = b^2 -bc +ca,
>  r = c^2 -ca +ab,
> とおくと、


どうやって思いついたんだ??
290286:2011/04/19(火) 06:56:22.13
確かに。
p+q+r=a^2+b^2+c^2
pq+qr+rp=a^3b+b^3c+c^3a
からp=a^2-ab+bc……を出すのは難しいと思う。
291132人目の素数さん:2011/04/19(火) 23:03:28.82
〔類題〕
a,b,c ≧ 0、3/4≦r≦8/3 のとき
 (a^2 +b^2 +c^2)^2 ≧ 3{a^r・b^(4-r) + b^r・c^(4-r) + c^r・a^(4-r)},

なら簡単だが・・・・・
292132人目の素数さん:2011/04/19(火) 23:12:30.94
>>291 の訂正 スマソ

 4/3 ≦ r ≦ 8/3 のとき
293132人目の素数さん:2011/04/20(水) 00:16:10.72
>>291-292

相加・相乗平均で
{(3/4)r -1}a^4 +2(ab)^2 +{2 -(3/4)r}b^4 ≧ 3a^r・b^(4-r),
巡回的にたす。
294132人目の素数さん:2011/04/20(水) 00:20:00.49
思いつくんじゃないのならできる。
295132人目の素数さん:2011/04/20(水) 01:31:20.75
>>289,290

それは, 秘密です. DX
296132人目の素数さん:2011/04/20(水) 02:06:55.71
初代スレの頃には、ここで不等式を探してハァハァ…してたんだけど、移転したのかな?

Kalva homepage
http://web.archive.org/web/20080205091131/www.kalva.demon.co.uk/index.html

検索したら、次のサイトが出てきたけど、扱ってる問題が減ってない?

http://www.cs.cornell.edu/~asdas/imo/index.html
297132人目の素数さん:2011/04/23(土) 23:38:27.16
〔問題549〕
任意の実数 x[1], x[2], ……, x[n] に対して 次を示せ。
 納k=1,n] {x[k]/(1+納L=1,k] x[L]^2)} < √n,
                     (じゅー)
キャスフィー 不等式 549, 574
298132人目の素数さん:2011/04/24(日) 02:39:10.57
>>297

〔補題〕
負でない実数 y[1], y[2], ……, y[n] に対して
 Σ[k=1,n] y[n]/(1+y[1]+y[2]+・・・・+y[n])^2 < 1,
299132人目の素数さん:2011/04/25(月) 20:25:11.82
平面上に4つの定点A,B,C,Dと動点Pがある
Dが△ABCの内部にあるとき
PA+PB+PC+PDが最小となるPはP=Dのときであることを示せ
300132人目の素数さん:2011/04/26(火) 00:31:40.08
>>298 の訂正....

〔補題〕
負でない実数 y[1], y[2], ……, y[n] に対して
 Σ[k=1,n] y[k]/(1+y[1]+y[2]+・・・・+y[k])^2 < 1,
301132人目の素数さん:2011/04/26(火) 12:45:59.84
★東大入試作問者になったつもりのスレ★ 第十九問
ttp://kamome.2ch.net/test/read.cgi/math/1287119136/542

AB=3、BC=4、CA=5 の三角形において
36°< C < 37° を示せ。
302132人目の素数さん:2011/04/26(火) 22:58:09.11
>>301 左側

 ∠B = 90゚ だから
 cos(C) = 4/5, sin(C) = 3/5,
 cos(2C) = 7/25,
 sin(4C)/sin(C) = 4cos(C)cos(2C) = 112/125 < 1,
 sin(4C) < sin(C),
 36゚ < 180゚/5 < C

 C ≒ 36.8699゚
303132人目の素数さん:2011/04/27(水) 02:34:23.79
>>299
 PA ≧ DA - PDcos(∠ADP), (PからADの延長線に垂線を下ろす)
 PB ≧ DB - PDcos(∠BDP),
 PC ≧ DC - PDcos(∠CDP),

∴ PA + PB + PC + PD ≧ (DA + DB + DC) + PD{1-cos(ADP) −cos(BDP) -cos(CDP)}

次に 1-cos(ADP) −cos(BDP) -cos(CDP) ≧ 0 を示せばよい。
304132人目の素数さん:2011/04/27(水) 09:45:05.17
>>302
正解です。
右側評価をお願いします。
305132人目の素数さん:2011/04/27(水) 10:02:46.60
306132人目の素数さん:2011/04/28(木) 02:09:28.16
>>301 >>304

 ∠B=90゚ ゆえ直角三角形で
 tan(C) = 3/4,
 tan(5C) = {(tan C)^5 -10(tan C)^3 +5(tan C)}/{1 -10(tan C)^2 +5(tan C)^4}
   = 237/(4・19・41),
 0 < 5C - 180゚ < tan(5C) = 237/(4・19・41) ≒ 4.3578625゚,
 180゚ < 5C < 184.3578625゚
 36゚ < C < 36.8715725゚

東大入試作問者スレ19-578
307132人目の素数さん:2011/04/28(木) 12:55:42.80
a_1,a_2,....a_nを正の数列とし、b_1, b_2....b_nを、その数列の任意の置換とする。
このとき、

a_1/b_1 + a_2/b_2 + ..... a_n/b_n ≧ n を示せ。

って有名だっけ?
308132人目の素数さん:2011/04/28(木) 14:44:03.42
はい, AM-GM or C.S.で秒殺です^_^
309132人目の素数さん:2011/04/28(木) 14:51:01.39
a, b, cを正の実数とするとき, 次の不等式を証明せよ。

\frac{a^2}{a+b}+\frac{b^2}{b+c}+\frac{c^2}{c+a}≧\frac{\sqrt{2}}{4}}(\sqrt{a^2+b^2} +\sqrt{b^2+c^2}+\sqrt{c^2+a^2}\right)
310307:2011/04/28(木) 23:31:17.26
>>308

すまぬ、文系のおれに、その略語の意味をおしえてくれ。
C.S.はコーシーシュワルツ?・・・ってどうやって?
311132人目の素数さん:2011/04/29(金) 06:47:11.86
>>309

 2(a^2 + b^2) = (a+b)^2 + (a-b)^2 ≧ (a+b)^2, より
 (a^2)/(a+b) = (a-b)/2 + (a^2 +b^2)/{2(a+b)} ≧ (a-b)/2 + {(√2)/4}√(a^2 + b^2),
循環的にたす。
312132人目の素数さん:2011/04/29(金) 08:23:28.12
>>311
そんな変形、思いつきませぬ!
313132人目の素数さん:2011/04/29(金) 09:15:15.97
>>311
(a^2 +b^2)/{2(a+b)} ≧ (a+b)/4 と変形できるから、循環的に足して、

a^2/(a+b) + b^2/(b+c) + c^2/(c+a) ≧ (a+b+c)/2

となったけど、

{(√2)/4}*{√(a^2 + b^2) + √(b^2 + c^2) +√(c^2 + a^2)}

と、どっちが大きいん?
314132人目の素数さん:2011/04/29(金) 09:51:49.21
>>309を改造

a^2/(a+b) + b^2/(b+c) + c^2/(c+a)
 ≧ {(√2)/4}*{√(a^2 + b^2) + √(b^2 + c^2) +√(c^2 + a^2)}
 ≧ (a+b+c)/2

これで合ってるよね? ウヒョッ!
315132人目の素数さん:2011/04/29(金) 11:14:28.55
a,b,c,d,e≧0
2a-b+3c-15d-12e=23
2a-6b-c-5d+11e=46

のとき

6a-3b+9c-15d+24e

の最小値を求めよ
316311:2011/04/29(金) 16:09:24.65
>>313
 √(a^2 + b^2) ≧ {(√2)/2}(a+b) より、・・・・・
の方がベターだな。


>>314 は対称式。
a^2/(a+b) + b^2/(b+c) + c^2/(c+a)
 ≧ {(√3)/2}√(a^2 + b^2 + c^2)
 ≧ {(√2)/4}{√(a^2 + b^2) + √(b^2 + c^2) +√(c^2 + a^2)}
 ≧ (a+b+c)/2,
かな?
317132人目の素数さん:2011/04/29(金) 16:45:31.78
>>315
 f = 2a -b +3c -15d -12e,
 g = 2a -6b -c -5d +11e,
 h = 10b +8c +3d,
とおくと
6a -3b +9c -15d +24e
 = (9/23)f + (60/23)g + (30/23)h
 = (9/23)*23 + (60/23)*46 + (30/23)h (← 題意)
 = 129 + (30/23)h
 ≧ 129,               (← 題意)
等号成立は (a,b,c,d,e) = (35/2,0,0,0,1) のとき。
318132人目の素数さん:2011/04/29(金) 18:43:56.56
>>316
> a^2/(a+b) + b^2/(b+c) + c^2/(c+a)
>  ≧ {(√3)/2}√(a^2 + b^2 + c^2)
>  ≧ {(√2)/4}{√(a^2 + b^2) + √(b^2 + c^2) +√(c^2 + a^2)}
>  ≧ (a+b+c)/2,

さらに改良しやがったな、こんちきしょう(笑)
さすが不等式ヲタ! にくいぜっ!
319132人目の素数さん:2011/04/29(金) 20:59:49.54
>>316
> a^2/(a+b) + b^2/(b+c) + c^2/(c+a)
>  ≧ {(√3)/2}√(a^2 + b^2 + c^2)

ここが分かりません…

>  ≧ {(√3)/2}√(a^2 + b^2 + c^2)
>  ≧ {(√2)/4}{√(a^2 + b^2) + √(b^2 + c^2) +√(c^2 + a^2)}

ここはCSでシコシコするんですね
320132人目の素数さん:2011/04/29(金) 21:13:18.80
コーシー・シュワルツの不等式を用いることを、
シコシコする、or シコる、というのか・・・(笑)
321132人目の素数さん:2011/04/29(金) 22:06:13.23
>>317
f と g の係数をうまく変えて
 6a -3b +9c -15d +24e = ●f + ●g + ●h'
となる h と異なる h' > 0 が取れて、最小値が変わったりしないのかな?
322132人目の素数さん:2011/04/30(土) 01:34:37.87
R^3\{(0,0,0)}上の関数

f(x,y,z)=(4x^2+4xz+3y^2+3z^2)/(2x^2+2xz+y^2+z^2)
の最大値を求めよ
323132人目の素数さん:2011/04/30(土) 03:58:12.84
>>322
 f(x,y,z) ={(2x+z)^2 +3y^2 +2z^2}/{(1/2)(2x+z)^2 +y^2 +(1/2)z^2},

 4 - f(x,y,z) = {(2x+z)^2 +y^2}/{(1/2)(2x+z)^2 +y^2 +(1/2)z^2} ≧ 0,
  等号成立は 2x+z=y=0 のとき。

ついでに最小値は
 f(x,y,z) - 2 = (y^2 +z^2)/{(1/2)(2x+z)^2 +y^2 +(1/2)z^2} ≧ 0,
  等号成立は y=z=0 のとき。
324132人目の素数さん:2011/04/30(土) 12:13:00.52
a, b, cをa^2+b^2+c^2+(a+b+c)^2≦4を満たす正の実数とするとき,
frac{ab+1}{(a+b)^2}+frac{bc+1}{(b+c)^2}+frac{ca+1}{(c+a)^2}≧3
を証明せよ。
325132人目の素数さん:2011/04/30(土) 22:33:26.86
>>319
a^2/(a+b) + b^2/(b+c) + c^2/(c+a) ≧ {(√3)/2}√(a^2 + b^2 + c^2)

やっぱ、これが分からんです
326132人目の素数さん:2011/05/01(日) 12:59:13.28
>>324

 2{a^2 + b^2 + (b+c)(c+a)} = a^2 + b^2 + c^2 + (a+b+c)^2 ≦ 4,
 ab + 1 ≧ ab + (1/2){a^2 + b^2 + (b+c)(c+a)}
   = (1/2)(a+b)^2 + (1/2)(b+c)(c+a),

 (左辺) ≧ (3/2) + (1/2){(b+c)(c+a)/(a+b)^2 + cyclic.}
    ≧ (3/2) + (3/2)   (← 相加・相乗平均)
    = 3,
327132人目の素数さん:2011/05/01(日) 14:41:38.31
>>320

 相撲の四股を踏む動作は、大地を踏みしめることで土の下に潜む「醜(シコ)」を鎮めるための動作とされている。

 醜(シコ):醜悪なもの、強く恐ろしいもの。
328132人目の素数さん:2011/05/01(日) 20:33:40.90
このスレの不等式ヲタって只者じゃないな。
暇つぶしにフラリと訪れて、サックリ解いて立ち去るような・・・
何者なんだ?
329132人目の素数さん:2011/05/01(日) 20:59:56.98
ただの通りすがりの不等式ヲタです
330132人目の素数さん:2011/05/02(月) 05:19:12.09
>>303

1 -cos(ADP) - cos(BDP) - cos(CDP) = 1 - eP・(e_A + e_B + e_C),
ここに
 e_A、e_B、e_C、eP は DA、DB、DC、DP方向の単位ベクトルである。
 |e_P| = 1 と下の補題から 上式 ≧ 0 が成り立つ。


〔補題〕
Dが△ABCの内部にあるとき、
 |e_A + e_B + e_C | ≦ 1,

(略証)
 e_A = (cosα, sinα)
 e_B = (cosβ, sinβ)
 e_C = (cosγ, sinγ)
とおく。(0≦α<β<γ<2π)
 題意より、DA,DB,DC が 180゚以内に収まることはない。
∴ 0 <β-α<π,
  0 <γ-β<π,
  π <γ-α<2π,
このとき
 |e_A + e_B + e_C |^2
 = (cosα+cosβ+cosγ)^2 + (sinα+sinβ+sinγ)^2
 = 3 + 2cos(β-α) + 2cos(γ-β) + 2cos(γ-α)
 = -3 + 4cos((β-α)/2)^2 + 4cos((γ-β)/2)^2 + 4cos((γ-α)/2)^2
 = 1 + 8cos((β-α)/2)cos((γ-β)/2)cos((γ-α)/2)
 ≦ 1,     (終)
331132人目の素数さん:2011/05/02(月) 10:21:35.49
a,b,cはabc=8を満たす正の実数とする。
frac{1}{a+2b+3}+frac{1}{b+2c+3}+frac{1}{c+2a+3}≦1/3
を証明せよ。
332132人目の素数さん:2011/05/02(月) 21:54:16.57
>>323
同次型の二変数関数の最大最小の解法って
何か定石みたいなのあるの?
333132人目の素数さん:2011/05/02(月) 22:06:04.19
>>331

(abc)^(1/3) = g とおくと、与式は
 1/{a+2b+3(g/2)} + 1/{b+2c+3(g/2)} + 1/{c+2a+3(g/2)} ≦ 1/{3(g/2)}


 (右辺) - (左辺)

 = {(a+2b)(b+2c)(c+2a) - (27/4)(a+b+c)gg - (27/4)ggg} / D   (←通分)

 = {24(aab+bbc+cca) + 48(abb+bcc+caa) +27abc -81(a+b+c)gg} /(12D)

 = {(19aab +5cca + 35caa +13abb +9abc -81agg) + cyclic} /(12D)

 = {a(19ab +5cc +35ca +13bb +9bc -81gg) + cyclic} /(12D)

 = {5a(ab+ab+cc-3gg) + 13a(ca+ca+bb-3gg) + 9a(ab+bc+ca-3gg) + cyclic} /(12D)

 ≧ 0,   (相加・相乗平均)

ここに D = [3(g/2)] [a+2b+3(g/2)] [b+2c+3(g/2)] [c+2a+3(g/2)],
334132人目の素数さん:2011/05/02(月) 23:09:47.20
>>332

ない。

y/x=u で一変数に還元するのみ。
335132人目の素数さん:2011/05/03(火) 06:01:51.43
>>328
少人数の自演者が、自分で問題出して自分で解いてるんだよ。
336132人目の素数さん:2011/05/03(火) 07:30:14.11
数式の最後に , があるかみたらいい
337132人目の素数さん:2011/05/03(火) 14:02:22.47
||Ax-b||^2の最小値に最も近い数値はどれか

A=
┌+4,+2,+6┐
│+1,+2,+5│
│+0,+1,+1│
└-3,+0,+3┘

b=
┌-3┐
│+1│
│+2│
└+3┘

1.0.102
2.0.103
3.0.104
4.0.105
5.0.106
338132人目の素数さん:2011/05/03(火) 14:15:35.62
x^2+y^2+z^2=1のもとで
f(x,y,z)=x^2+3y^2+z^2+4xy+4yz
の最大値を求めよ

log1.5に最も近い数値はどれか
1.0.38
2.0.4
3.0.42
4.0.44
5.0.46

-2a-b+c+d=2
-3a+b+c-d=1
a,b,c,d≧0
のもとで
-6a+2b+5c+3dの最小値を求めよ
339◇Pandysv26:2011/05/03(火) 14:43:50.27
バカオツ(ーー;)
340132人目の素数さん:2011/05/03(火) 17:35:34.36
>>337

Ax-b =
 [4x+2y+6z+3]
 [x+2y+5z-1]
 [y+z-2]
 [-3x+3z-3]

||Ax-b||^2 = (4x+2y+6z+3)^2 + (x+2y+5z-1)^2 + (y+z-2)^2 + (-3x+3z-3)^2
  = 26x^2 + 9y^2 + 71z^2 + 20xy + 46yz + 40zx +40x +4y +4z + 23
  = 26X^2 + 9Y^2 + 71Z^2 + 20XY + 46YZ + 40ZX + (9/85)
  = (1/9){(10X+9Y+23Z)^2 + 134X^2 + 110Z^2 -100ZX} + (9/85)
  = (1/9){(10X+9Y+23Z)^2 + 50(X-Z)^2 + 84X^2 + 60Z^2} + (9/85)
  ≧ 9/85
  = 0.105882353,

ここに、X=x+(225/170), Y=y-(363/170), Z=z+(59/170) とおいた。
(平行移動した。)

〔別法〕
 F(X,Y,Z) = 26X^2 + 9Y^2 + 71Z^2 + 20XY + 46YZ + 40ZX
とおく。Fの固有多項式は
 f(λ) = (26-λ)(9-λ)(71-λ) + 2(10*20*23) -23*23(26-λ) -20*20(9-λ) -10*10(71-λ)
    = -λ^3 +106λ^2 -1690λ+1360
∴ λ≦0 ならば f(λ) ≧ 1360
ところで、実対称行列の固有値はすべて実数(*)。
∴ λ > 0
∴ Fは正定値、 F(X,Y,Z) ≧ 0 (等号成立は X=Y=Z=0 のみ)

*) エルミート行列の場合も成立つ。
341132人目の素数さん:2011/05/03(火) 18:26:59.88
>>338 (上)

最大値
 5(x^2+y^2+z^2) - f(x,y,z) = 4x^2 +2y^2 +4z^2 -4xy -4yz
              = (2x-y)^2 + (y-2z)^2 ≧ 0,
最小値
 f(x^2+y^2+z^2) + (x^2+y^2+z^2) = 2x^2 +4y^2 +2z^2 +4xy +4yz
              = (x+2y+z)^2 + (x-z)^2 ≧ 0,

〔別解〕
軸を回して
 u = (x-y+z)/√3,
 v = (x-z)/√2,
 w = (x+2y+z)/√6,
とおく。
 u^2 +v^2 +w^2 = x^2 +y^2 +z^2,

 f(x,y,z) = (-1)u^2 +1v^2 +5w^2,
342132人目の素数さん:2011/05/03(火) 22:19:53.71
>>338 (中)

〔解1〕
 (3/2)^2 = 2*(9/8) = 2*(1 + 1/8),
 2log(3/2) = log(2) + log(9/8) ≦ log(2) + 1/8,
   = 0.69314718 + 0.125
   = 0.81814718
 log(3/2) ≦ 0.40907359

〔解2〕
 (3/2)^5 = (2^3)(243/256) = (2^3)(1 - 13/256)
 5log(3/2) = 3*log(2) + log(243/256)
   ≦ 3*log(2) - 13/256
   = 3*0.69314718 - 0.05078125
   = 2.02866029
 log(3/2) ≦ 0.40573206

〔解3〕
 (3/2)^12 = (2^7)(531441/524288) = (2^7){1 + 7153/(2^19)},
 12log(3/2) = 7log(2) + log(531441/524288)
   ≦ 7log(2) + 7153/(2^19)
   = 7*0.69314718 + 0.013643265
   = 4.86567353
 log(3/2) ≦ 0.40547279

なお、log(3/2) = 0.405465108
343132人目の素数さん:2011/05/04(水) 01:24:52.76
>>299 >>303
 PA + PB + PC = f(P) とおく。
〔系〕
P,Qが△ABCの内部にあるとき
 |f(P)-f(Q)|/PQ ≦ 1,
344132人目の素数さん:2011/05/04(水) 03:14:00.34
この場合log2の値出すの反則じゃない?
345 忍法帖【Lv=9,xxxP】 :2011/05/04(水) 10:14:23.79

    |┃三 ./  ≧ \   
    |┃   |::::  \ ./ | 
    |┃ ≡|::::: (● (● |  不等式と聞ゐちゃぁ
____.|ミ\_ヽ::::... .ワ......ノ     黙っちゃゐられねゑ…
    |┃=__    \           ハァハァ
    |┃ ≡ )  人 \ ガラッ
346132人目の素数さん:2011/05/04(水) 11:11:36.85
>>338 (中)
>>344
 e = 2.71828183 は使っていい?
 3^(1/3) ≦ e^(1/e),
の対数をとって
 log(3) ≦ 3/e = 1.10363832  (1.09861229)

〔解1〕
 (3/2)^3 = 3*(9/8) = 3*{1 + 1/(2^3)},
 3log(3/2) = log(3) + log(9/8) ≦ 3/e + 1/(2^3),
 log(3/2) ≦ 1/e + 1/(3*2^3) = 0.4095461

〔解2〕
 (3/2)^8 = (3^3)(243/256) = (3^3){1 - 13/(2^8)},
 8log(3/2) = 3log(3) + log(243/256) ≦ 9/e - 13/(2^8),
 log(3/2) ≦ (9/8e) - 13/(8*2^8) = 0.4075167

〔解3〕
 (3/2)^19 = (3^7)(531441/524288) = (3^7){1 + 7153/(2^19)},
 19log(3/2) = 7log(3) + log(531441/524288) ≦ 21/e + 7153/(2^19)
 log(3/2) ≦ (21/19e) + 7153/(19*2^19) = 0.40732166
347132人目の素数さん:2011/05/04(水) 15:06:56.84
>>346
 log(3) = 1 + log(3/e) ≦ 1 + (3/e -1) = 3/e,
348132人目の素数さん:2011/05/06(金) 16:43:56.28
平面上に4つの定点A,B,C,Dと動点Pがある
A,B,C,Dのどの三点をとっても同一直線上になく
線分ACとBDが一点で交わるとき
PA+PB+PC+PDが最小となる点Pの位置を決定せよ
349132人目の素数さん:2011/05/06(金) 21:59:52.30
>>348
 >>299 の類題でござるな。

線分ACとBDが交わるから、ABCD は凸四角形。
 PA + PC ≧ AC,
 PB + PD ≧ BD より、
 PA + PB + PC + PD ≧ AC + BD,
より 対角線の交点。


一方、Dが△ABCの内部(または辺上)にあるときは
>>299-303 により D.
350132人目の素数さん:2011/05/06(金) 22:07:51.60
〔問題593〕
a,b,c≧ 0 とする。相加・相乗平均を用いて次式を示せ。
 {(a+b)/2・(b+c)/2・(c+a)/2}^(1/3) ≧ {√(ab)+√(bc)+√(ca)}/3,
 等号成立は a=b=c のとき。


http://www.casphy.com/bbs/test/read.cgi/highmath/1169210077/593-595
 キャスフィー - 高校数学 - 不等式スレ
351132人目の素数さん:2011/05/06(金) 23:06:06.34
>>350
(;´д`) ハァハァ…
352132人目の素数さん:2011/05/06(金) 23:55:39.25
A(x,y)は非負整数から非負整数への二変数関数であり

A(0,y)=y+1
A(x+1,0)=A(x,1)
A(x+1,y+1)=A(x,A(x+1,y))

を満たす

A(x+1,y)>A(x,y)を示せ
ゐとかゑってどうやって入力するの?
354132人目の素数さん:2011/05/07(土) 15:18:57.49
〔350の類題〕
a,b,c≧0 のとき
 (a+b+c)/3 ≧ {(a+b)/2・(b+c)/2・(c+a)/2}^(1/3) ≧ √{(ab+bc+ca)/3} ≧ {√(ab)+√(bc)+√(ca)}/3 ≧ (abc)^(1/3),

これで合ってるよね? ウヒョッ!
355132人目の素数さん:2011/05/07(土) 16:19:36.97
>>354

左から2つめ
 ab+bc+ca =t とおく。
 (a+b)/2・(b+c)/2・(c+a)/2 = (1/8){(a+b+c)t-abc}
  = (1/9)(a+b+c)t + (1/72){(a+b+c)t-9abc}
  ≧ (1/9)t(a+b+c)
  = (1/9)t√(a^2 +b^2 +c^2 +2t)
  ≧ (1/9)t√(3t)
  = (t/3)^(3/2),

3つめは
 (1/3)(ab+bc+ca) = (1/9){(ab+bc+ca) +a(b+c) +b(c+a) +c(a+b)}
  ≧ (1/9){(ab+bc+ca) +2a√bc +2b√(ca) +2c√(ab)}
  = (1/9){√(ab) +√(bc) +√(ca)}^2,
ぬるぽ
356132人目の素数さん:2011/05/07(土) 19:16:23.92
>>354
(*゚∀゚)=3 ハァハァ…
357132人目の素数さん:2011/05/08(日) 21:07:45.02
a,b,cをa+b+c=0を満たす実数とするとき, 次の不等式を証明せよ。

frac{a(a+2)}{2a^2+1}+\frac{b(b+2)}{2b^2+1}+frac{c(c+2)}{2c^2+1}≧0

358132人目の素数さん:2011/05/09(月) 02:53:53.07
>>325

対称式なので、いつものように a+b+c=s, ab+bc+ca=t, abc=u とおいて通分する。
 (左辺) = {(a^2+b^2+c^2)(ab+bc+ca) + [(ab)^2+(bc)^2+(ca)^2]}/{(a+b)(b+c)(c+a)}
  = {(s^2 -2t)t + (t^2 -2su)}/(st-u)
  = {2(s^2 -2t)t/3 + (s^2 -2t)t/3 + (t^2 -2su)}/(st-u)
  ≧ {2(s^2 -2t)t/3 + (9/32t)(st-u)^2}/(st-u)   (←補題)
  ≧ {(√3)/2}√(s^2 -2t)    (←相加・相乗平均)
  = (右辺),

〔補題〕
 (s^2 -2t)t/3 + (t^2 -2su) ≧ (9/32t)(st-u)^2,

(略証)
 (左辺) - (右辺) = (1/3)(t^2 -3su) + (7/144)s(st-9u) + (1/288t){(st)^2 -81u^2} ≧0,

しかし、基本対称式を使うやり方は、どうもマンドクセ.....
359132人目の素数さん:2011/05/09(月) 04:48:30.04
>>358
ありがとうございます、(*゚∀゚)=3 ハァハァ…

公式一発ではムリポだったので、基本対称式を使うしかないと思って、
ゴリゴリ計算はしていたのですが、私には辿りつけませんでした…orz
360132人目の素数さん:2011/05/09(月) 23:53:25.24
>>316 (>>314 >>309)と >>354 (>>350) を組合わせたら、どうなりまつか?
361132人目の素数さん:2011/05/09(月) 23:56:19.96
>>360

 {(a^3+b^3+c^3)/3}^(1/3)     (r=3)
 ≧ {(a^2+b^2)/(a+b) + (b^2+c^2)/(b+c) + c^2/(c+a)}/3 (r〜5/2)
 ≧ √{(a^2 + b^2 + c^2)/3}   RMS(r=2)
 ≧ {(√2)/6}{√(a^2 + b^2) + √(b^2 + c^2) +√(c^2 + a^2)} (r〜3/2)
 ≧ (a+b+c)/3          相加平均(r=1)
 ≧ {(a+b)/2・(b+c)/2・(c+a)/2}^(1/3)  (r〜3/4)
 ≧ √{(ab+bc+ca)/3}          (r〜1/2)
 ≧ {√(ab)+√(bc)+√(ca)}/3      (r〜1/4)
 ≧ (abc)^(1/3)         相乗平均(r→0)
 ≧ 3abc/(ab+bc+ca),        調和平均(r=-1)


〔rの意味〕
a,b,c が近いときは
 {(a^r + b^r + c^r)/3}^(1/r) 〜 (abc)^(1/3) + (r/18)*{(a-b)^2 + (b-c)^2 + (c-a)^2},
となる。
362132人目の素数さん:2011/05/10(火) 02:46:58.18
ひどい自演見た
363132人目の素数さん:2011/05/10(火) 06:41:22.70
>>362
2ch初心者は黙ってろ!
364132人目の素数さん:2011/05/10(火) 11:08:11.69
>>354

a,b,c≧0 のとき
 (a+b+c)/3 ≧ {(a+b)/2・(b+c)/2・(c+a)/2}^(1/3) ≧ √{(ab+bc+ca)/3} ≧ {√(ab)+√(bc)+√(ca)}/3 ≧ (abc)^(1/3)

By AM-GM, frac{a+b+c}{3}=\frac 13(frac{a+b}{2}+frac{b+c}{2}+frac{c+a}{2})≧(frac{a+b}{2}*frac{b+c}{2}*frac{c+a}{2})^(1/3)

By AM-GM, (a+b)(b+c)(c+a)≧8abc⇒(frac{a+b}{2}*frac{b+c}{2}*frac{c+a}{2})^(1/3)≧(abc)^(1/3)≧(frac{ab+bc+ca}{3})^(1/2)

by Newton's Inequality.

By QM-AM, (frac{ab+bc+ca}{3})^(1/2)≧frac{√ab+√bc+√ca}{3}≧(abc)^(1/3) Done!
365132人目の素数さん:2011/05/11(水) 16:25:20.69
360:132人目の素数さん[sage]
2011/05/09(月) 23:53:25.24
>>316 (>>314 >>309)と >>354 (>>350) を組合わせたら、どうなりまつか?


361:132人目の素数さん[sage]
2011/05/09(月) 23:56:19.96
>>360



この間約3分
366132人目の素数さん:2011/05/11(水) 23:56:28.85
>>365
別に珍しくなかろう
俺なんか起きている間はずっと2ch見てるから
その気になれば直ぐに返事できるぜ
367132人目の素数さん:2011/05/11(水) 23:57:33.38
>>365
それより不等式の話をしろ
嫌なら消えろ!
368132人目の素数さん:2011/05/11(水) 23:59:32.17
うるせえ
369132人目の素数さん:2011/05/12(木) 00:03:06.74

('A` ) プウ
ノヽノ) =3'A`)ノ ヒャー
  くく へヘノ ←>>368
370132人目の素数さん:2011/05/12(木) 00:05:48.10
くせえ
371132人目の素数さん:2011/05/12(木) 02:19:08.02
3分でsageでついたレスをチェックして計算を書き上げるのかー

さすがに苦しいだろw
372132人目の素数さん:2011/05/12(木) 07:00:03.07
俺も自演しながら荒らしてます!
バカオツ(ーー;)バカオツ(ーー;)バカオツ(ーー;)
バカオツ(ーー;)バカオツ(ーー;)バカオツ(ーー;)
バカオツ(ーー;)バカオツ(ーー;)バカオツ(ーー;)
バカオツ(ーー;)バカオツ(ーー;)バカオツ(ーー;)
バカオツ(ーー;)バカオツ(ーー;)バカオツ(ーー;)
バカオツ(ーー;)バカオツ(ーー;)バカオツ(ーー;)
バカオツ(ーー;)バカオツ(ーー;)バカオツ(ーー;)
バカオツ(ーー;)バカオツ(ーー;)バカオツ(ーー;)
373必ずレスくるよ! ◆jK4/cZFJQ0Q6 :2011/05/12(木) 17:02:29.89
>>372
気持ち悪いぞ!キチガイ!
バカオツ(^∇^)!
キチガイはたくさんだな!
パクリ乙(ーー;)警!
キチガイ丸出し!
偽物オツピーオツピー♪バカオツケー♪
頑張れ!偽物!
374132人目の素数さん:2011/05/14(土) 13:31:11.56
Challenge!

a+b+c=0を満たすすべての実数に対して,

frac{a(a+p)}{pa^2+1}+frac{b(b+2)}{pb^2+1}+frac{c(c+p)}{pc^2+1}≧0

が成り立つとき, pのとりうる値の範囲を求めよ。
375132人目の素数さん:2011/05/14(土) 13:34:04.62
問題, 打ち間違えました。正しくは, こちらです。

a+b+c=0を満たすすべての実数a,b,cに対して,

frac{a(a+p)}{pa^2+1}+frac{b(b+p)}{pb^2+1}+frac{c(c+p)}{pc^2+1}≧0

が成り立つとき, pのとりうる値の範囲を求めよ。
376132人目の素数さん:2011/05/14(土) 13:36:23.44
何で最後のcだけ全角
377132人目の素数さん:2011/05/14(土) 23:15:41.41
えっ, どの部分ですか?
378132人目の素数さん:2011/05/15(日) 01:13:00.72
バカオツ(ーー;)
バカオツ(ーー;)
バカオツ(ーー;)
バカオツ(ーー;)
バカオツ(ーー;)
バカオツ(ーー;)
バカオツ(ーー;)
バカオツ(ーー;)
バカオツ(ーー;)
バカオツ(ーー;)
バカオツ(ーー;)
バカオツ(ーー;)
バカオツ(ーー;)
バカオツ(ーー;)
バカオツ(ーー;)
バカオツ(ーー;)
バカオツ(ーー;)
バカオツ(ーー;)
バカオツ(ーー;)
バカオツ(ーー;)
バカオツ(ーー;)
バカオツ(ーー;)
バカオツ(ーー;)
バカオツ(ーー;)
バカオツ(ーー;)
バカオツ(ーー;)
バカオツ(ーー;)
バカオツ(ーー;)
バカオツ(ーー;)
バカオツ(ーー;)
379132人目の素数さん:2011/05/15(日) 06:13:34.94
ここの不等式のレベルは, タイトルの割には, レベル, 低すぎ。
海外では, 中学生レベルにしか値しない。
さっさと, 店じまいしろ。378は, 精神年齢, 低すぎ!
380132人目の素数さん:2011/05/15(日) 06:32:31.69
バカオツ(ーー;)
バカオツ(ーー;)
バカオツ(ーー;)
バカオツ(ーー;)
バカオツ(ーー;)
バカオツ(ーー;)
バカオツ(ーー;)
バカオツ(ーー;)
バカオツ(ーー;)
381132人目の素数さん:2011/05/15(日) 08:08:29.47

('A` ) プウ
ノヽノ) =3'A`)ノ ヒャー
  くく へヘノ ←>>379
382偽物発生中 ◆jK4/cZFJQ0Q6 :2011/05/15(日) 08:41:56.70
>>378
偽物注意!!!!!

頑張れよ!偽物キチガイ!

パクリ乙(ーー;)バカオツ(ーー;)
ニートは数学勉強だ!

>>380
パクリ乙!!!!!

さすがキチガイ!!!!!

悔しいのか???www
頑張れよ!偽物カスカスニート!
383132人目の素数さん:2011/05/21(土) 21:32:59.16
〔問題〕
a,b,cは実数、ab+bc+ca =t とおくとき、次を示せ。(じゅー)

(1) (a^2 +2)(b^2 +2)(c^2 +2) ≧ 9t + (8/9)(t-3)^2,
  等号成立は a=b=c=±1, t=3 のとき。

(2) (a^2 +2)(b^2 +2)(c^2 +2) ≧ t^2 +(13/4)t +8,
  等号成立は a=b=c=±√(3/2), t=9/2 のとき。
384猫は海賊 ◆4c5pft6zx. :2011/05/21(土) 21:36:53.45
385猫は海賊 ◆MuKUnGPXAY :2011/05/21(土) 21:48:52.94

386猫は海賊 ◆4c5pft6zx. :2011/05/21(土) 21:59:32.00
387132人目の素数さん:2011/05/24(火) 21:43:11.36
>>383

(3) (a^2 +2)(b^2 +2)(c^2 +2)
 を 3つの対称式の平方和で表わせ。
388132人目の素数さん:2011/05/25(水) 01:33:08.93
>>387
(p^2 + q^2)(r^2 + s^2) = (pr + qs)^2 + (ps - qr)^2   … Lagrangeの恒等式

を繰り返し用いると、

(a^2 +2)(b^2 +2)(c^2 +2)
= { (ab + 2)^2 + (a√2 - b√2)^2 }*(c^2 +2)
= { (ab + 2)c + (a√2 - b√2)√2 }^2 + { (ab + 2)√2 + (a√2 - b√2)c }^2
= { (ab + 2)c + 4(a-b) }^2 + 2{ (ab + 2) + (a - b)c }^2
= (abc + 2c + 4a -4b)^2 + 2(ab + 2 + ac - bc)^2

失敗でござるよ、 ドンマイ ( ゚∀゚)ノ
389132人目の素数さん:2011/05/25(水) 02:20:45.94
>>388
(a^2 + p^2)(b^2 + q^2)(c^2 +r^2) = (abc-aqr-pbr-pqc)^2 + (pbc+aqc+abr-pqr)^2,
だと2つになるし・・・・・
390132人目の素数さん:2011/05/28(土) 10:38:16.63
>>389
 p=q=r=√2 を入れて
 {abc-2(a+b+c)}^2 + (bc+ca+ab-2)^2 + (bc+ca+ab-2)^2,
391132人目の素数さん:2011/06/10(金) 17:58:19.78
2(x-1)/(x^2-2x+2) の最小値と最大値は? (-1≦x≦3)
392132人目の素数さん:2011/06/10(金) 20:45:14.10
-1と1
393132人目の素数さん:2011/06/10(金) 20:49:20.27
>>391
分子を2にして、場合分けしてAM-GM
AM-GMを使うときは、正でないと使えないぞ!
∴-1 ≦ 2(x-1)/(x^2-2x+2) ≦ 1

宿題は質問スレに逝け!
394132人目の素数さん:2011/06/11(土) 02:05:20.80
>>391
 x-1 = X とおくと、
 y = 2X/(1+X^2),
 1±y = 1 ± 2X/(1+X^2) = (1±X)^2 /(1+X^2) ≧ 0,
でもいい?
395132人目の素数さん:2011/06/11(土) 04:24:49.92
イイヨイイヨー!
396132人目の素数さん:2011/06/11(土) 05:50:22.01
a、b、c、d、e、f > 0 に対して、
ab/(a+b) + cd/(c+d) + ef/(e+f) ≦ abcdef/(a+b+c+d+e+f)

( ゚∀゚)わけがわからないよ
397132人目の素数さん:2011/06/12(日) 19:09:15.01
398132人目の素数さん:2011/06/12(日) 19:50:00.19
a=b=c=d=e=f.
399132人目の素数さん:2011/06/15(水) 02:11:04.50
>>397

〔補題〕
 a1 + a2 = A,
 b1 + b2 = B,
とおくと
 a1・b1/(a1+b1) + a2・b2/(a2+b2) ≦ A・B/(A+B),
(略証)
 (右辺) - (左辺) = (a1・b2-a2・b1)^2/{(a1+b1)(a2+b2)(A+B)} ≧ 0,
400132人目の素数さん:2011/06/19(日) 05:51:38.00
〔補題〕
 a_ij>0, (i=1,2,・・・・・,n)(j=1,2,・・・・,m)
 Σ[j=1,m] a_ij = A_i, とおくとき

 Σ[j=1,m] 1/{Σ[i=1,n] 1/a_ij} ≦ 1/(1/A1 + 1/A2 + ・・・・・ + 1/An),

 (by kuing, Nov.20, 2009, 5:47 am) m=2
 (by Mavropnevma, Jun.10, 2011, 2:25 pm) m=2
401132人目の素数さん:2011/06/19(日) 05:54:25.60
>>400

(略証)
右辺を S とおく。1/S = 1/A1 + 1/A2 + ・・・・ + 1/An,
コーシーより
 {Σ[i=1,n] a_ij/(Ai)^2}・{Σ[i=1,n] 1/a_ij} ≧ (1/A1 + ・・・・ + 1/An)^2 = 1/S^2,
よって
 1/{Σ[i=1,n] 1/a_ij} ≦ {Σ[i=1,n] a_ij/(Ai)^2}・S^2,
j=1,2,・・・・・,m についてたす。
 (左辺) ≦ (1/A1 + 1/A2 + ・・・・・ + 1/An)・S^2 = S,

 (proof by Vn2009, Nov.20, 2009, 8:21 am) m=2
 (proof by tang zy, Nov.21, 2009, 2:50 am) m=2
 (proof by hendrata01, Nov.23, 2009, 4:23 pm) m=2
 (note by spanferkel, Nov.21, 2009, 3:30 am) m=3, etc.

http://www.artofproblemsolving.com/Forum/viewtopic.php?t=313265
402132人目の素数さん:2011/06/19(日) 06:11:46.69
>>400 と同じだが・・・・

〔補題〕
x_ij > 0 のとき
 A_i = (Σ[j=1,m] x_ij)/m,   (i=1,2,・・・・,n)
 H_j = n/(Σ[i=1,n] 1/x_ij),  (j=1,2,・・・・,m)
とおくと、
 (H1 + H2 + ・・・・ + Hm)/m ≦ n/(1/A1 + 1/A2 + ・・・・・ + 1/An),
403132人目の素数さん:2011/06/19(日) 07:48:50.29
>>402 念のため...

(略証)
右辺を S とおく。n/S = 1/A1 + 1/A2 + ・・・・ + 1/An,
コーシーより
 {Σ[i=1,n] x_ij/(Ai)^2}・(Σ[i=1,n] 1/x_ij) ≧ (1/A1 + ・・・・ + 1/An)^2 = (n/S)^2,
よって
 H_j ≦ n・{Σ[i=1,n] x_ij/(Ai)^2}・(S/n)^2,
j=1,2,・・・・・,m について相加平均する。
 (左辺) ≦ n(1/A1 + 1/A2 + ・・・・・ + 1/An)・(S/n)^2 = S,
404132人目の素数さん:2011/06/29(水) 17:33:48.88
405132人目の素数さん:2011/07/01(金) 12:35:17.26
>>350
abc = u とおく。
 (上式)^3 = (a+b)/2・(b+c)/2・(c+a)/2
 = {ab(a+b) +bc(b+c) +ca(c+a) +2u}/8
 ≧ (1/27){2ab(a+b) +2bc(b+c) +2ca(c+a) +15u} ← ※
 = (1/27){ab(a+b)/2 +bc(b+c)/2 +ca(c+a)/2
     +3(aab+u)/2 +3(abb+u)/2 +3(bbc+u)/2 +3(bcc+u)/2 +3(cca+u)/2 +3(caa+u)/2 +6u}
 ≧ (1/27){ab√(ab) +bc√(bc) +ca√(ca)
     +3ab√(ca) +3ab√(bc) +3bc√(ab) +3bc√(ac) +3ca√(bc) + 3ca√(ab) +6u}
 = (1/27){√(ab) +√(bc) +√(ca)}^3
 = (右辺)^3


※のところが、どうやって見つけたのか分かりませぬ…

ところで、√a、√b、√c の基本対称式 s、t、u を使って、
力任せに (左辺)^3-(右辺)^3 を計算しても出来ますか?

差をとって分母払った式は 27s^2t^2 - 54s^3u -62t^3 +108stu -27u^2 で、
これが0以上になるかが示せない…
406132人目の素数さん:2011/07/01(金) 22:16:20.04
>>404

[問題A-2]
 n個の実数値函数 u_1(x),u_2(x) 〜 u_n(x) (a≦x≦b) を考える。このとき、次の不等式を示せ。
 √{Σ[i=1,n] (∫[a,b] u_i(x)dx)^2 } ≦ ∫[a,b] √([i=1,n] u_i(x)^2) dx,

(略証)
 √{Σ[i=1,n] u_i(x)^2} = U(x) ≧ 0 とおく。
コーシーより
 Σ[i=1,n] u_i(x)・u_i(y) ≦ U(x)・U(y),
よって
 (左辺)^2 = ∫[a,b] ∫[a,b] Σ[i=1,n] u_i(x)・u_i(y) dxdy
     ≦ ∫[a,b] U(x)dx・∫[a,b] U(y)dy
     = (右辺)^2,
407132人目の素数さん:2011/07/01(金) 23:13:10.37
>>405
 s^2 → (s^2 -3t) + 3t,
 t^2 → (t^2 -3su) + 3su,
のように分解すると
 27(s^2 -3t)(t^2 -2su) + 19t(t^2 -3su) + 3(st-9u)u ≧ 0,
408132人目の素数さん:2011/07/04(月) 20:26:12.66
>>402-403 の続き

〔補題〕
x_ij > 0 のとき
 A_i = (Σ[j=1,n] x_ij)/n,   (i=1,2,・・・・,m)
 G_j = (Π[i=1,m] x_ij)^(1/m),  (j=1,2,・・・・,n)
 H_i = n/(Σ[j=1,n] 1/x_ij),  (i=1,2,・・・・,m)
とおくと、

(1) (A1・A2・・・・Am)^(1/m) ≧ (G1 + G2 + ・・・・ + Gn)/n,
(2) (H1・H2・・・・Hm)^(1/m) ≦ n/(1/G1 + 1/G2 + ・・・・・ + 1/Gn),
409132人目の素数さん:2011/07/04(月) 20:33:17.08
>>408
 ヘルダーの不等式。
 たとえば、まとめWiki を参照 >>1

(1) p_i → m, |a_ij|^m → x_ij, b_i = n・A_i, とおく。
(2) p_i → m, |a_ij|^m → 1/xij, b_i = n/H_i, とおいて、逆数をとる。
410132人目の素数さん:2011/07/14(木) 04:04:48.39
外出だったらスマソ.

〔問題〕
abc=1, a,b,c>0 のとき
 (a^2 +b^2)/(c^2 +a +b) + (b^2 +c^2)/(a^2 +b +c) + (c^2 +a^2)/(b^2 +c +a) ≧ 2,
411132人目の素数さん:2011/07/14(木) 11:52:16.90
>>410
分母の次数を2次の項だけに変えたいけど、うまくいかん…
412132人目の素数さん:2011/07/15(金) 21:09:37.12
これ前にもやったっけ?

〔問題〕
正の数 a、b、c、d に対して、
{(ab+ac+ad+bc+bd+cd)/6}^(1/2) ≧ {(abc+abd+acd+bcd)/4}^(1/3)
413132人目の素数さん:2011/07/16(土) 03:40:25.36
>>412

[初代スレ.455-456]

(略解)
f(x) = (x-a)(x-b)(x-c)(x-d), とおく。
f(x)=0 は重根を含めて4個の正の根をもつ。
f '(x)=0 も重根を含めて3個の正の根 α,β,γ をもつ。
 f '(x) = 4(x-α)(x-β)(x-γ),
xの係数より 2(ab+ac+ad+bc+bd+cd) = 4(αβ + βγ + γα),
定数項より -(abc+abd+acd+bcd) = -4αβγ,
これを用いて 示すべき不等式を α,β,γ で表わすと
 √{(αβ+βγ+γα)/3} ≧ (αβγ)^(1/3),
となる。これは相加・相乗平均の関係だから不等式は示された。
等号成立条件は α=β=γ で、このとき a=b=c=d.
414132人目の素数さん:2011/07/16(土) 03:42:23.82
>>298

[初代スレ.465]
415132人目の素数さん:2011/07/16(土) 06:55:35.88
>>2
まとめサイトの参考文献[9]の後に

[10] 三角法の精選103問(シリーズ:数学オリンピックへの道 2),T.アンドレースク・Z.フェン著,朝倉書店,2010年
   http://www.asakura.co.jp/books/isbn/978-4-254-1180...

を追加し、[10]〜[13]を[11]〜[14]にずらしました ( ゚∀゚)

証明する際に、三角関数に置き換えるものも含めて、
三角関数がらみの不等式の問題がたくさん載っています
416132人目の素数さん:2011/07/16(土) 07:00:21.68
●刊行予定●
不等式(数学のかんどころシリーズ)、大関清太、共立出版、未定
http://www.kyoritsu-pub.co.jp/series/kandokoro.html

「不等式への招待」が絶版となったので、超期待! (*゚∀゚)=3 ハァハァ…
417132人目の素数さん:2011/07/16(土) 07:10:48.41
〔問題〕
実数 a、b、c に対して、
(a^2 + 1)(b^2 + 1)(c^2 + 1) ≧ (ab +bc +ca -1)^2

左辺は良く見かけるけど、これは初めてのような希ガス…
418132人目の素数さん:2011/07/16(土) 15:00:01.26
(1+ai)(1+bi)(1+ci)=(1−ab−ac−bc)+(a+b+c−abc)i。
419132人目の素数さん:2011/07/16(土) 15:03:45.05
>>418
なん…だと!
420132人目の素数さん:2011/07/16(土) 15:22:13.62
>>417
a=tanα, b=tanβ, c=tanγとおく。明らかにcosαcosβcosγ≠0
1≧|cos(α+β+γ)|=|cosαcosβcosγ-sinαsinβcosγ-cosαsinβsinγ-sinαcosβsinγ|
|1/(cosαcosβcosγ)|≧|1-tanαtanβ-tanβtanγ-tantγtanα|
(cosα)^(-2)*(cosβ)^(-2)*(cosγ)^(-2)≧(1-tanαtanβ-tanβtanγ-tantγtanα)^2
{(tanα)^2+1}{(tanβ)^2+1}{(tanγ)^2+1}≧(tanαtanβ+tanβtanγ+tantγtanα-1)^2
より示される
等号成立は
Arctan(a)+Arctan(b)+Arctan(c)=0, ±π
(a,b,c)=(1,-1/2,-1/3)とか(2+√3,√3,1)とか
421132人目の素数さん:2011/07/17(日) 02:08:41.04
>>420
 cos(α+β+γ) = ・・・・
 sin(α+β+γ) = cosα・cosβ・sinγ + cosα・sinβ・cosγ + sinα・cosβ・cosγ - sinα・sinβ・sinγ,
を使えば
 (左辺) = 1/(cosα・cosβ・cosγ)^2
     = (1-tanα・tanβ-tanβ・tanγ-tanγ・tanα)^2 + (tanα + tanβ + tanγ - tanα・tanβ・tanγ)^2
     = (1-ab-bc-ca)^2 + (a+b+c-abc)^2,
422132人目の素数さん:2011/07/17(日) 08:05:05.56
a,b,cは正の実数とするとき,
a^3/(a+b)^2+b^3/(b+c)^2+c^3/(c+a)^2≧(a+b+c)/4
423132人目の素数さん:2011/07/17(日) 09:15:12.50
>>422
4a^3-(a+b)^2*(2a-b)=(a-b)^2*(2a+b)≧0より
4a^3≧(a+b)^2(2a-b)
a^3/(a+b)^2≧(2a-b)/4
同様に繰り返して辺々足して与不等式
424132人目の素数さん:2011/07/17(日) 15:20:11.27
4a^3-(a+b)^2*(2a-b)=(a-b)^2*(2a+b)のideaはどこから?
425132人目の素数さん:2011/07/17(日) 17:50:11.85
定石ですよ、定石!
426132人目の素数さん:2011/07/17(日) 18:07:58.81
ならば、不等式の証明に使える定石とやらを列挙してもらおうか?
427132人目の素数さん:2011/07/17(日) 21:16:34.07
>>412 (別法)

 P1 = (a+b+c+d)/4,
 P2 = (ab+ac+ad+bc+bd+cd)/6,
 P3 = (abc+abd+acd+bcd)/4,
 P4 = abcd,
とおくと
 P1^4 ≧ P2^2 ≧ P1・P3 ≧ P4,

(略証)
 P1^2 - P2 = (1/48){(a-b)^2 +(a-c)^2 +(a-d)^2 +(b-c)^2 +(b-d)^2 +(c-d)^2} ≧ 0,
 P2^2 - P1・P3 = (1/288){(ab-ac)^2 + (ab-ad)^2 + (ab-bc)^2 + (ab-bd)^2 + ・・・・
            + 4(ab-cd)^2 + 4(ac-bd)^2 + 4(ad-bc)^2} ≧ 0,
 P1・P3 - P4 ≧ 0, (相加・相乗平均)

〔系〕 P1 ≧ √P2 ≧ (P3)^(1/3),
428132人目の素数さん:2011/07/17(日) 22:06:41.54
>>427

n変数のときも同様に、
 P1 = (a1 + a2 + ・・・・・ + an)/n,
 P2 = {a1・a2 + ・・・・・ + a(n-1)・an}/C[n,2],
 P3 = {a1・a2・a3 + ・・・・・ + a(n-2)・a(n-1)・an}/C[n,3],
とおくと
 P1^4 ≧ P2^2 ≧ P1・P3,

(略証)
 P1^2 - P2 = {1/[n^2 (n-1)]}{(a-b)^2 +(a-c)^2 +(a-d)^2 +(a-e)^2 + ・・・・・} ≧ 0,
 P2^2 - P1・P3 = {1/[n^2 (n-1)^2 (n-2)]}{(5-n)(ab-ac)^2 + (5-n)(ab-ad)^2 + (5-n)(ab-ae)^2 +・・・・
            + 4(ab-cd)^2 + 4(ab-de)^2 + ・・・・・} ≧ 0,

〔系〕 P1 ≧ √P2 ≧ (P3)^(1/3),
429132人目の素数さん:2011/07/18(月) 00:06:54.77
>>425

その変形は,自然に気づかないでしょう?
だれか, もう少し詳しく教えていただきませんか?
430132人目の素数さん:2011/07/18(月) 01:21:39.49
>>424 >>429
生姜ねぇ....

 a^3 /(a+b)^2 ≧ γ/4,
とおく。
 a^3 ≧ {(a+b)/2}{(a+b)/2}γ,

右辺は (a+b)/2, (a+b)/2, γの相乗平均の3乗。
これらの相加平均が a なら、相加・相乗平均で成立。
 (a+b)/2 + (a+b)/2 + γ = 3a,
 γ = 2a-b,
431132人目の素数さん:2011/07/18(月) 01:26:59.25
うーん、ぬぬぬ…
432132人目の素数さん:2011/07/18(月) 02:19:45.59
>>430 やっぱ、AM-GMかあ。これが自然だよな。
あとは, CS, Jensen
これって,今月号の大数にのってたやつじゃねえ?
(1) に4a^3-(a+b)^2*(2a-b)=(a-b)^2*(2a+b)があったような。

433132人目の素数さん:2011/07/18(月) 04:27:35.04
>>431
 う〜ん、ぬぬぬるぽ
と言いたかったのだな。

等号成立条件 (a+b)/2 = γ ⇔ a=b にも注意。
434132人目の素数さん:2011/07/18(月) 11:59:43.81
1. Holder Σa^3/(a+b)^2≧(a+b+c)^3/(Σ(a+b))^2

2.AM-GM Σ(4a^3/(a+b)^2+(a+b)/2+(a+b)/2)≧3Σa

3. C.S. (a+b+c)(Σa^3/(a+b)^2)≧(Σa^2/(a+b))^2≧((a+b+c)/2)^2

4. Jensen
435132人目の素数さん:2011/07/18(月) 14:17:03.31
ちぇびちぇび、へるだあ、みんこ、しゅうあ、まじょらい、ぐろんを、並べ替え不等式、…
彼らのことも、たまには思い出してやってください
436132人目の素数さん:2011/07/18(月) 15:11:08.55
AM-GMは中学の時に出会うほど基本的なのに最強だな
437132人目の素数さん:2011/07/18(月) 15:17:17.77
Soient a,b,c,d dans R^{+} tels que a+b++d=6, a^2+b^2+c^2+d^2=12.
Prouver que :
36≦4(a^3+b^3+c^3+d^3)-(a^4+b^4+c^4+d^4)≦48.
438132人目の素数さん:2011/07/18(月) 15:18:42.48
Soient a,b,c,d dans R^{+} tels que a+b+c+d=6, a^2+b^2+c^2+d^2=12.
Prouver que :
36≦4(a^3+b^3+c^3+d^3)-(a^4+b^4+c^4+d^4)≦48.
439132人目の素数さん:2011/07/18(月) 19:05:38.49
>>410

 1/(1-x) ≧ 1+x より
 1/(c^2 +a +b) = 1/{a^2 +b^2 +c^2 -(a^2 +b^2 -a-b)}
   = 1/{S - (a^2 +b^2 -a-b)]}
   ≧ 1/S + (a^2 +b^2 -a-b)]/S^2
   ≧ 1/S + [a^2 +b^2 -(a+b)(a+b+c)/3]/S^2  (←題意)
   = 1/S + [2(a-b)^2 +2ab-bc-ca]/(3S^2)
   ≧ 1/S + (2ab-bc-ca)/(3S^2),
 ここに S = a^2 +b^2 +c^2 とおいた。

 (左辺) ≧ 2 + {(a^2 +b^2)(2ab-bc-ca) + cyclic}/(3S^2)
    = 2 + {a(b-c)(b^2 -c^2) + cyclic}/(3S^2)
    ≧ 2,
440132人目の素数さん:2011/07/18(月) 22:26:06.82
〔問題〕
正の数 a、b、c が abc=1 をみたすとき、
(a^2+b^2)/(c^2+a+b) + (b^2+c^2)/(a^2+b+c) + (c^2+a^2)/(b^2+c+a) ≧ 2
441132人目の素数さん:2011/07/19(火) 00:21:55.69
   ∩___∩三 ー_        ∩___∩
   |ノ      三-二     ー二三 ノ      ヽ
  /  (゚)   (゚)三二-  ̄   - 三   (゚)   (゚) |
  |    ( _●_)  ミ三二 - ー二三    ( _●_)  ミ  テンション上がってきた!!
 彡、   |∪|  、` ̄ ̄三- 三  彡、   |∪|  ミ     テンション上がってきた!!
/ __  ヽノ   Y ̄) 三 三   (/'    ヽノ_  |
(___) ∩___∩_ノ    ヽ/     (___)
442132人目の素数さん:2011/07/19(火) 06:33:56.04
>>438

まづ 0≦a,b,c,d≦3 を示す。 コーシーより
 3(12-a^2) = (1+1+1)(b^2 + c^2 + d^2) ≧ (b+c+d)^2 = (6-a)^2,
 0 ≧ 3(a^2 -12) + (6-a)^2 = 4a(a-3),
 0≦a≦3,
 b,c,d についても同様。

次に 0≦x≦3 で 2x^2 +4x -3 ≦ 4x^3 -x^4 ≦ 4x^2, を示す。
 (4x^3 -x^4) - (2x^2 +4x-3) = (x+1)(3-x)(x-1)^2 ≧ 0,
 4x^2 - (4x^3 -x^4) = x^2・(2-x)^2 ≧ 0,

x=a,b,c,d について和をとると
 2*12 +4*6 -3*4 ≦ 与式 ≦ 4*12,
 36 ≦ 与式 ≦ 48,
左等号成立は {3,1,1,1}
右等号成立は {2,2,2,0}

くそ〜、テンション上がっちまった...
443132人目の素数さん:2011/07/19(火) 09:27:04.02
Nice Solution!
444132人目の素数さん:2011/07/19(火) 12:25:03.59
>>442
> 次に 0≦x≦3 で 2x^2 +4x -3 ≦ 4x^3 -x^4 ≦ 4x^2, を示す。

神! この不等式をどうやって思いつくのか謎!
445132人目の素数さん:2011/07/19(火) 13:20:00.38
(0<=x<=3)=>(f(x)=x^4-4x^3+ax^2+bx+c<=0).
f(1)=0.
f(3)=0.

f(x)=(x-1)^2(x-3)(x-d)=x^4-4x^3+ax^2+bx+c.
d=-1.

f(x)=x^4-4x^3+2x^2+4x-3.
446132人目の素数さん:2011/07/20(水) 10:14:37.63
このスレ恐ろしすぎる
447132人目の素数さん:2011/07/20(水) 17:21:02.92
不等式ヲタ ≒ 数ヲタ ⇒ ロリコン だからですか?
448132人目の素数さん:2011/07/20(水) 18:25:13.74
〔問題〕
正の数 x、y が x+y=1 をみたすとき、(x^x)(y^y) + (x^y)(y^x) ≦ 1
449132人目の素数さん:2011/07/20(水) 21:26:23.94
>>447
正解!
450132人目の素数さん:2011/07/20(水) 23:35:36.28
>>448
x^x-y^xとx^y-y^yは正負が一致するかともに0かなので
(x^x-y^x)(x^y-y^y)≧0
x^(x+y)+y^(x+y)≧(x^x)(y^y)+(x^y)(y^x)
左辺=x+y=1より
1≧(x^x)(y^y)+(x^y)(y^x)
451132人目の素数さん:2011/07/21(木) 00:29:09.54
>>450
神すぎる…
452132人目の素数さん:2011/07/21(木) 09:53:30.81
Soient a,b tels que 0<a≦1, 0<b≦1.
Prouver que : a^{b-a}+b^{a-b}≦2.
453132人目の素数さん:2011/07/21(木) 10:21:14.26
>>448

By the Weighted AM-GM, x^xy^y≦x^2+y^2, x^yy^x≦2xy

∴x^xy^y+x^yy^x≦(x+y)^2=1 Done!
454132人目の素数さん:2011/07/21(木) 10:23:42.09
>>452
難しい (;´д`) ハァハァ…

ところで a^{b-a}+b^{a-b} の下限はいくらになるのですか? 0にいくらでも近づく?
455132人目の素数さん:2011/07/21(木) 10:27:42.91
>>454
下限というか最小値は1かな?
456132人目の素数さん:2011/07/21(木) 10:43:07.23
>>453

Sorry, my proof was wrong. I thought that x, y are positive integers.

457132人目の素数さん:2011/07/21(木) 10:59:00.10
No, your proof is CORRECT!
458132人目の素数さん:2011/07/21(木) 20:29:14.21
>>453 の weighted AM-GM というのは
 p,q,x,y>0, p+q=1 ⇒ x^p・y^q ≦ px + qy,
459132人目の素数さん:2011/07/22(金) 04:35:26.03
重み付き相加相乗って懐かしいな
すっかり忘れていた…
460132人目の素数さん:2011/07/22(金) 04:35:52.11
>>453, >>458

 凸不等式から出る。別名 ベルヌーイの式。
 数セミ、2010/08月号 NOTE (大塚氏) も参照。
461132人目の素数さん:2011/07/30(土) 15:16:53.93
x≧0, y≧0, x+y=1 のとき, 自然数m,nに対して

( 1-x^m )^n + ( 1-y^n )^m ≧1
462132人目の素数さん:2011/07/30(土) 17:25:09.46
>>859
いつから名前がバカオツなんだかwww
クソキチガイアホ晒しできてるぞ?
頑張れよクソキチガイ
クソキチガイアホ晒しできてるぞ?
クソキチガイアホ晒しできてるぞ?
頑張れ!クソキチガイ!
顔真っ赤にしてクソキチガイ反応




さっきから必死に頑張ってます!

by>>859
463コピペキチガイ必死w ◆osMsTqWzXY :2011/07/30(土) 17:25:21.59
>>462
いつから名前がバカオツなんだかwww
クソキチガイアホ晒しできてるぞ?
頑張れよクソキチガイ
クソキチガイアホ晒しできてるぞ?
クソキチガイアホ晒しできてるぞ?
頑張れ!クソキチガイ!
顔真っ赤にしてクソキチガイ反応




さっきから必死に頑張ってます!

by>>462
464132人目の素数さん:2011/07/30(土) 18:00:36.44
>>461
むむむ…、分からん
465132人目の素数さん:2011/07/30(土) 21:48:35.22
どうみても二項定理だろアホw
466132人目の素数さん:2011/07/30(土) 22:10:33.54
>>465
証明してみろ!
467132人目の素数さん:2011/07/30(土) 22:49:10.99
>>461

(略証)
 g(x) = 1 - (1-x)^n とおくと (左辺) = 1 -g(x^m) + {g(x)}^m.
 g(x) の逆函数を f(z) と書くと、 f(0)=0, f(1)=1 かつ
 f(z) = 1 - (1-z)^(1/n) = (1/n)z + (1/2n)(1-1/n)z^2 + (1/3n)(1-1/n)(1-1/2n)z^3 + ……
 a_k = {(k-1)/k}・{1 -1/(k-1)n}・a_{k-1} > 0.
∴ f(z) は下記の【命題268】の条件をみたす。
∴ f(z^m) ≧ {f(z)}^m,
∴ z^m ≧ g({f(z)}^m),
∴ {g(x)}^m ≧ g(x^m),

[初代スレ.563(7), 973]
[第2章.21, 346-347, 353]
468132人目の素数さん:2011/07/30(土) 22:51:17.31
>>467 の続き

【命題268】
f(x) は |x|≦1 で正則な解析函数で、f(0)=0, f(1)=1 かつ
マクローリン展開の係数がすべて非負実数とする。

このとき, 0≦x≦1 において
 r>1  ⇒ f(x^r) ≧ {f(x)}^r.
 0<r<1 ⇒ f(x^r) ≦ {f(x)}^r.
 (math_board_watcherによる)


(略証)
題意より、f(x) = Σ[k=1,∞) a_k・(x^k), a_k ≧ 0.
 Σ[k=1,∞) a_k = f(1) = 1.
Jensenの定理より(収束について適当な条件のもとで)
r>1 ⇒ x^r は下に凸 ⇒
 f(x^r) = Σ[k=1,∞) a_k・(x^r)^k = Σ[k=1,∞) a_k・(x^k)^r > {Σ[k=1,∞) a_k・x^k}^r = {f(x)}^r.
0<r<1 ⇒ x^r は上に凸 ⇒
 f(x^r) = Σ[k=1,∞) a_k・(x^r)^k = Σ[k=1,∞) a_k・(x^k)^r < {Σ[k=1,∞) a_k・x^k}^r = {f(x)}^r.

Yahoo! - 科学板 - 数学カテ - 出題(不等式)トピ - 268,272
469132人目の素数さん:2011/07/31(日) 05:50:42.62

きたか…!!

  ( ゚д゚ ) ガタッ
  .r   ヾ
__|_| / ̄ ̄ ̄/_
  \/    /
470132人目の素数さん:2011/07/31(日) 12:49:57.13
[前スレ.608] の小改良....

以上の評価から
(1/2){(1+t)^(1-t) +(1-t)^(1+t)} ≦ 1 -t^2 +(3/4)t^4,
(1/2){(1+t)^(1-t) -(1-t)^(1+t)} ≦ t -(1/2)t^3,

log(2) = a とおくと
 cosh(a/2) = 3/(2√2) = 1.06066017,
 sinh(a/2) = 1/(2√2) = 0.35355339,

McLaurin展開係数がすべて正だから、t^2 について下に凸
 cosh(at) ≦ 1 +(3√2 -4)t^2,  (0<t<1/2)
 sinh(at) ≦ at +(2√2 -4a)t^3, (0<t<1/2)

以上から
x^(2y) + y^(2x)
 = {(1-t)/2}^(1+t) + {(1+t)/2}^(1-t)
 ≦ {1 -t^2 +(3/4)t^4}・{1 +(3√2 -4)t^2}
 + {t -(1/2)t^3}・{at +(2√2 -4a)t^3}
 = 1 -(5-a-3√2)t^2 +{19/4 -(9/2)a -√2)t^4 +{-3 +2a +(5/4)√2}t^6
 ≦ 1 -(5-a-3√2)t^2 +{4-4a-(11/16)√2}t^4
 ≦ 1 -{4-(181/64)√2}t^2
 = 1 -0.000427268・t^2,  (0<t<1/2)
471132人目の素数さん:2011/07/31(日) 13:28:12.30
>>461
mn個の放射性核種を、m行n列の長方形状に並べる。どの核種も1分以内に確率xで崩壊するとする。
二つの事象を考える:
 [a] 1分後、第1列〜第n列のうち、m個すべてが崩壊している列が少なくとも1列ある。
 [b] 1分後、第1行〜第m行のすべての行で、少なくとも1個が崩壊している。

[a]の確率は 1 - (1-x^m)^n ・・・(1)
[b]の確率は (1-y^n)^m ・・・(2)

事象の包含関係から (2)≧(1) 。
472132人目の素数さん:2011/08/01(月) 23:36:30.55
>>470

 4 > (181/64)√2 の証明
128√2 > 181
 2(128^2) - 181^2 = 7 > 0,

 2X^2 - Y^2 = 7,

 (X_0, Y_0) = (2, -1)
漸化式
 X_{n+1} = 3X_n + 2Y_n,
 Y_{n+1} = 4X_n + 3Y_n,
より
 X_n = {1 - 1/(2√2)}(1+√2)^(2n) + {1 + 1/(2√2)}(1-√2)^(2n),
 Y_n = {√2 -(1/2)}(1+√2)^(2n) + {-√2 -(1/2)}(1-√2)^(2n),
473132人目の素数さん:2011/08/03(水) 10:28:00.18
x+y+z=1を満たす実数x,y,zに対して、次の不等式が成立することを示せ
(x^2+y^2+z^2)^2*(1/x+1/y+1/z)≧1
474132人目の素数さん:2011/08/03(水) 12:00:02.32
x=3。
y=−1。
z=−1。
475132人目の素数さん:2011/08/03(水) 14:18:15.05
>>473
胡散臭い不等式やと思うたら案の定か!
476132人目の素数さん:2011/08/05(金) 01:50:12.86
>>461 の類題

 (1-x^m)^n + n・(1-x^m)^(n-1)・x^m + {1-y^n -nx・y^(n-1)}^m ≧ 1,

 (1-x^m)^n + n・x^m・(1-x^m)^(n-1) + {n(n-1)/2!}x^(2m)・(1-x^m)^(n-2)

   + {1 -y^n -nx・y^(n-1) -[n(n-1)/2!]x^2・y^(n-2)}^m ≧ 1,

つまらねぇ....
477132人目の素数さん:2011/08/05(金) 02:03:18.01
しょうがないなあ

A536, B4364, B4370
http://www.komal.hu/verseny/feladat.cgi?a=honap&h=201105&t=mat&l=en

478132人目の素数さん:2011/08/05(金) 10:10:52.07
a+b+c+d≧4(abcd)^(1/4)=p
abc+abd+acd+bcd≧4(abcd)^(3/4)=q
a+b+c+d=abc+abd+acd+bcdよりp=q
∴abcd=1

(左辺)
=2(ac+bd)+ab+bc+cd+da
≧2(ac+bd)+4(acbd)^(1/2)
=2{(1+ac)+(1+bd)}
≧2*2{(1+ac)(1+bd)}^(1/2)
=(右辺)
479132人目の素数さん:2011/08/05(金) 10:12:41.91

>>477
A536
480132人目の素数さん:2011/08/06(土) 00:04:44.11
>>477

[B4370.]
 頂点A,B,C,の対辺の長さを a,b,c とする。BC=a, CA=b, AB=c,
内心をIとおき、AI=u, BI=v, CI=w とおく。このとき次を示せ。
 (a+b+c)(1/u+1/v+1/w) ≦ 3(a/u + b/v + c/w),

(略解)
 a>b ⇔ BC > CA ⇔ ∠BAC > ∠ABC ⇔ ∠BAI > ∠ABI ⇔ BI > AI ⇔ v > u,
∴ {a,b,c} と {1/u,1/v,1/w} とは同順
あとはチェビシェフに任した…
481132人目の素数さん:2011/08/06(土) 00:36:05.93
質問スレに張られてた奴

a,b,c>0, abc=1のとき
1/(a^3(b+c))+1/(b^3(c+a))+1/(c^3(a+b))≧3/2
を示せ
482132人目の素数さん:2011/08/06(土) 02:51:33.91
>>481
 コーシーより、
 (左辺) ≧ (1/a + 1/b + 1/c)^2 / {a(b+c) + b(c+a) + c(a+b)}
     = (1/a + 1/b + 1/c)^2 / {2abc(1/a + 1/b + 1/c)}
     = (1/a + 1/b + 1/c) / (2abc)
     ≧ 3/{2(abc)^(4/3)}    (相加・相乗平均)
     = 3/2,

※ a=1/x, b=1/y, c=1/z, xyz=1 とおく方法もある。
483132人目の素数さん:2011/08/06(土) 07:12:31.74
>>482
成程な〜
484132人目の素数さん:2011/08/06(土) 11:26:48.95
>>482
          ___  
    |┃三 ./  ≧ \   ちょ〜っと待ったあ!!
    |┃   |::::  \ ./ | 
    |┃ ≡|::::: (● (● |  不等式ヲタ参上!
____.|ミ\_ヽ::::... .ワ......ノ     黙っちゃゐられねゑ…
    |┃=__    \           ハァハァ
    |┃ ≡ )  人 \ ガラッ

>>483の証明で、CS と AM-GM を用いて

  1/(a^3(b+c))+1/(b^3(c+a))+1/(c^3(a+b)) ≧ 3/{2(abc)^(4/3)} …@

が示された。等号成立条件は a=b=c=1/3。 ここまでは見事ですが

だが、ここで abc=1より、@≧3/2 としていいのか?
@が成り立つのは a=b=c=1/3 のときであって、このとき abc = 1/27 なのだから、
@の右辺に abc=1 を代入してはダメじゃないの?
485132人目の素数さん:2011/08/06(土) 12:53:39.49
>>484
出直してこい
486132人目の素数さん:2011/08/06(土) 13:39:40.36
>>477
B4364

a+b≧2c
(a^2-b^2)/c≧2(a-b)…(1)

2a≧b+c
2(b-c)≧(b^2-c^2)/a
(c^2-b^2)/a≧2(c-b)…(2)

a+c>b
(a^2-c^2)/b≧a-c…(3)

(1)(2)(3)を足して
(a^2-b^2)/c+(c^2-b^2)/a+(a^2-c^2)/b≧3a-4b+c
487132人目の素数さん:2011/08/06(土) 13:42:38.09
ダメじゃないの。
488482:2011/08/06(土) 14:17:02.81
>>484

等号成立条件は a=b=c=1。

が抜けてたな.....

ぬるぽ
489132人目の素数さん:2011/08/06(土) 14:20:07.02
すまん、積でしたな
490132人目の素数さん:2011/08/06(土) 14:40:31.10
>>477

[B4364.]
a ≧ b ≧ c > 0 のとき 次を示せ。
 (a^2 - b^2)/c - (b^2 - c^2)/a + (a^2 - c^2)/b ≧ 3a-4b+c,

(略解)
 (左辺) ≧ (a^2 - b^2)/b - (b^2 - c^2)/b + (a^2 - c^2)/b
  = 2(a^2 - b^2)/b
  = {2(a+b)/b}(a-b)
  ≧ 4(a-b),
以下簡単。
491486:2011/08/06(土) 18:26:09.37
>>490
うまい…
492132人目の素数さん:2011/08/06(土) 22:05:29.24
>>477

[B4371.]
 1/{sin(π/14)}^2 + 1/{sin(3π/14)}^2 + 1/{cos(5π/7)}^2 = 24,
を示せ。


(略解)
 (左辺) = 1/{cos(3π/7)}^2 + 1/{cos(2π/7)}^2 + 1/{cos(π/7)}^2
  = 1/{cos(4π/7)}^2 + 1/{cos(2π/7)}^2 + 1/{cos(6π/7)}^2
  = Σ[k=1,3] 1/{cos(2kπ/7)}^2,

 {1 - T_7(x)}/(1-x) = 1 +7x -56x^3 +112x^5 -64x^7
       = (1-x)(1 +4x -4x^2 -8x^3)^2,

 cos(2kπ/7)  (k=1,2,3) は 1 +4x -4x^2 -8x^3 = 0 の根。
 1/cos(2kπ/7) (k=1,2,3) は y^3 +4y^2 -4y -8 = 0 の根。

   Σ[k=1,2,3] 1/cos(2kπ/7) = -4,
   Σ[k<L] 1/{cos(2kπ/7)cos(2Lπ/7)} = -4,
よって
   Σ[k=1,3] 1/{cos(2kπ/7)}^2 = 4^2 -(-4)*2 = 24,
493492:2011/08/06(土) 22:11:32.65
>>492 訂正

[B4371.]
 1/{sin(π/14)}^2 + 1/{sin(3π/14)}^2 + 1/{sin(5π/14)}^2 = 24,
を示せ。
494132人目の素数さん:2011/08/07(日) 09:00:49.56
For real numbers $p,\ q,\ r$, prove that

p(p+q)^3+q(q+r)^3+r(r+p)^3≧(8/27)(p+q+r)^4
495132人目の素数さん:2011/08/07(日) 15:33:09.16
p^2+q^2+r^2=x^2
G=p(p+q)^3+q(q+r)^3+r(r+p)^3-s(p^2+q^2+r^2-x^2)
Gp=(p+q)^3+3p(p+q)^2+3r(r+p)^2-2sp=0
p^3+3p^2q+3pq^2+q^3+3p^3+6p^2q+3pq^2+3r^3+6r^2p+3rp^2-2sp=0
Gq=(q+r)^3+3q(q+r)^2+3p(p+q)^2-2sq=0
Gr=(p+r)^3+3r(p+r)^2+3q(r+q)^2-2sr=0
...
p=q=r=x(1/3)^.5
f=3x^4(8/3^2)=x^4(8/3)
RH=(8/3^3)(3^4x^4/3^2)=x^4(8/3)
496132人目の素数さん:2011/08/08(月) 00:04:38.81
>>494

f(x) = x^m は単調増加で下に凸。(m≧1)
 {p(p+q) + q(q+r) + r(r+p)}/(p+q+r)
  = {4(p+q+r)^2 + (p-q)^2 + (q-r)^2 + (r-p)^2}/{6(p+q+r)}
  ≧ (2/3)(p+q+r),

Jensen より
(左辺) = p・f(p+q) + q・f(q+r) + r・f(r+p)
  ≧ (p+q+r)・f({p(p+q) + q(q+r) + r(r+p)}/(p+q+r)) [下に凸]
  ≧ (p+q+r)・f((2/3)(p+q+r))  [単調増加]
  = (2/3)^m・(p+q+r)^(m+1),
ぢゃね?
497132人目の素数さん:2011/08/08(月) 09:27:01.04
正じゃない。
498132人目の素数さん:2011/08/08(月) 15:02:45.71
For positive real numbers a, b, c, d with abcd=1,

Prove that

1/a + 1/b +1/c +1/d + 9/(a + b + c + d) ≧ 25/4
499132人目の素数さん:2011/08/11(木) 00:34:49.08
a≧b≧c≧dとする。
abcd=1よりa≧1である。

(左辺)
≧1/a+1/a+1/a+1/a+9/(a+a+a+a)
=25/(4a)
≧25/4
500499:2011/08/11(木) 00:42:47.14
間違えたorz
501132人目の素数さん:2011/08/14(日) 14:11:15.95
あほ
502132人目の素数さん:2011/08/14(日) 14:44:52.56
>>501
口が悪いな、直したほうがいい
503132人目の素数さん:2011/08/14(日) 17:09:15.69
>>498 難しくない?
504132人目の素数さん:2011/08/15(月) 01:09:44.14
>>494 >>497 難しくない。

 19 = 3^2 + 3^2 + 1^2

 (左辺) - (右辺) = (1/27){3p^2 -3q^2 +r^2 +(7/4)pq -(22/4)qr +(11/4)rp}^2 + (3/16){p(q-r)}^2 + cyclic.
とか
 (左辺) - (右辺) = (1/27){3p^2 -3q^2 -r^2 +(113/28)pq -(131/28)qr +(46/28)rp}^2 + (3/16){p(q-r)}^2 + cyclic.
とか
505132人目の素数さん:2011/08/15(月) 01:18:09.31
なんだ、ただの神か…
506132人目の素数さん:2011/08/15(月) 10:35:43.41
>>504 の補足

まづ p^4 + q^4 + r^4 の係数を見る。
 左辺は1、右辺は 8/27 だから 1 - (8/27) = 19/27,
そこで 19 を3平方の和で表わした。

難しくない。
507132人目の素数さん:2011/08/15(月) 20:13:45.05
>>477
 
[A536.]
 a,b,c,d は正の実数で a+b+c+d = abc+bcd+cda+dab のとき 次を示せ。
 (a+b)(c+d) + (a+d)(b+c) ≧ 4√{(1+ac)(1+bd)},

(略解)
abcd≧1 のとき
 (左辺) = (a+c)(b+d) + 2(ac+bd) ≧ 4√(abcd) + 2(ac+bd) ≧ 2(1+ac) + 2(1+bd) ≧ (右辺),

abcd≦1 のとき、補題により
 t = (ab+cd) + (ac+bd) + (ad+bc) ≧ 6,
 (左辺) ≧ 6 + (ac+bd) ≧ 4√{2 + (ac+bd)} ≧ 4√(1+ac+bd+abcd) = (右辺),

〔補題〕
 a,b,c,d>0 で a+b+c+d = abc+bcd+cda+dab のとき、
 (ab+cd) + (ac+bd) + (ad+bc) ≧6,
(略証)
 左辺をtとおいて
 2{(a+b+c+d)t - 6(abc+bcd+cda+dab)}
 = (a+b)(c-d)^2 + (a+c)(b-d)^2 + (a+d)(b-c)^2 + (b+c)(d-a)^2 + (b+d)(c-a)^2 + (c+d)(a-b)^2 ≧ 0,
 ∴ t ≧ 6,
508132人目の素数さん:2011/08/16(火) 05:09:19.68
>>498

 左辺を f(a,b,c,d) とおく。
ab<2 のとき
 f(a,b,c,d) - f(√(ab), √(ab),c,d)
 = (√a - √b)^2・{1/ab - 9/[(a+b+c+d)(2√ab +c +d)]}
 ≧ (√a - √b)^2・{1/ab - 9/(2√ab +c +d)^2}
 ≧ (√a - √b)^2・{1/ab - 9/(2√ab +2/√ab)^2}
 = (√a - √b)^2・{1/ab - 9ab/(4(ab+1)^2)}
 = (√a - √b)^2・(2-ab)(2+5ab)/{4ab(ab+1)^2}
 ≧ 0,
ここで c+d ≧ 2√cd = 2/√ab を使った。
a≧b≧c≧d とすると cd≦1
(a,b,c,d) が最小値ならば c=d に限る。
∴ bc = bd ≦1, 
∴ b=c=d≦1,
∴ (a,b,c,d) = (A^3, 1/A, 1/A, 1/A) ただし A≧1.
となって
 f(A^3,1/A,1/A,1/A) ≧ 25/6,  (A≧1)
に帰着する。
509132人目の素数さん:2011/08/16(火) 05:26:39.15
>>498

次に
 f(A^3,1/A,1/A,1/A) ≧ 25/4,  (A≧1)
を示そう。

 f(A^3,1/A,1/A,1/A) - 25/4
 = 1/A^3 + 3A + 9A/(A^4 +3) - 25/4
 = 3(A-1)^2・{A^6 -(1/12)A^5 -(7/6)A^4 -(9/4)A^3 +3A^2 +2A +1}/{A^3(A^4 +3)}
 = 3(A-1)^2・g(A)/{A^3(A^4 +3)}
 ≧ 0,

∵ g(A) = A^6 -(1/12)A^5 -(7/6)A^4 -(9/4)A^3 +3A^2 +2A +1
  = {A^3 -(1/24)A^2 -(673/1152)A -(31777/27648)}^2 + 2.56293026A^2 +0.657105936A -0.3209864
  = {A^3 -(1/24)A^2 -(673/1152)A -(31777/27648)}^2 + 2.56293026(A-1)^2 +5.782966457A +2.899049797
  > 0.

難しくない。>>503
510132人目の素数さん:2011/08/16(火) 05:33:13.51
>>508-509
の最後の式の右辺は間違い。

 25/4
 +5.782966457(A-1)
に訂正。
511132人目の素数さん:2011/08/16(火) 19:20:06.30
>>509

最小を探すなら、微分使った方が簡単....だな

 F(A) = 1/A^3 +3A +9A/(A^4 +3),

 F '(A) = -3/A^4 + 3 + 27(1-A^4)/(A^4 +3)^2
     = 3(A^4 -1)(A^8 -3A^4 +9)/{(A^4)(A^4 +3)^2},

  A^8 -3A^4 + 9 = (A^4 -3)^2 + 3A^4 > 0,
512132人目の素数さん:2011/08/19(金) 01:38:45.69
>>509

F(A)≧ 25/4 だけなら、代数使った方が簡単....だな

A^4 + 3 = (4/√3)A^3 + (A-√3)^2 {A^2 +(2/√3)A +1}
    ≧ (4/√3)A^3
    > (9/4) A^3,
より
F(A) - 25/4 = {(1/A^3) +3A -4} + (9/4){4A/(A^4 +3) -1}
  = (A-1)^2・(3A^2 +2A+1)/A^3 - (9/4)(A-1)^2・(A^2 +2A+3)/(A^4 +3)
  > (A-1)^2・{(3A^2 +2A+1) - (A^2 +2A+3)}/(A^3)
  = (A-1)^2・2(A^2 -1)/(A^3)
  ≧ 0,   (A≧1)
513132人目の素数さん:2011/08/20(土) 15:12:34.36
>>512

相加・相乗平均を使わないなら
 3^5 = 243 < 256 = 16^2,
より
 A^4 + 3 > A^4 + 3(3^5/16^2)^2
  = (9/4)A^3 + (A - 27/16)^2{A^2 + (9/8)A + (3^5)/(16^2)}
  = (9/4)A^3 + (A - 27/16)^2{(A + 9/16)^2 + 162/(16^2)}
  ≧ (9/4)A^3,

どうでもいいけど.....
514132人目の素数さん:2011/08/20(土) 15:44:57.93
【問題1】
正の数 x、y、z が z≧x+y をみたすとき、
x^2 + y^2 + z^2 ≧ (6/5)*(xy + yz + zx) を示せ

【問題2】
0.160 < ∫[0,1] x^2 e^(-x^2) dx <0.215 を示せ

【問題3】
正の数 a、b、c が a+b+c=1 をみたすとき、
(a + 1/a)^2 + (b + 1/b)^2 + (c + 1/c)^2 ≧ 100/3

http://www.asahi-net.or.jp/~nj7h-ktr/kadai10-11.pdf

上の上の数ヲタである不等式ヲタの皆さんには、【問題3】など瞬殺でしょうから、

(a + 1/a)^4 + (b + 1/b)^4 + (c + 1/c)^4 ≧ ?
(a + 1/b)^3 + (b + 1/c)^3 + (c + 1/a)^3 ≧ ?

と変えたところで、やはり秒殺でしょう (by スマートブレイン社社長)
515132人目の素数さん:2011/08/21(日) 05:37:23.73
>>514
【問題1】
 z = x + y + Z' (Z'≧0) を代入して整理する。
 (左辺) - (右辺) = (4/5)(x-y)^2 + (4/5)(x+y)Z' + (Z')^2 ≧ 0,
 等号成立は (x,y,z) = (1,1,2) のとき。

【問題2】
(左) e^(-x^2) = (1/e)e^(1-x^2) > (1/e)(2-x^2), より
 I > (1/e)∫[0,1] (x^2)(2-x^2)dx
  = (1/e) [(2/3)x^3 -(1/5)x^5 ](x=0,1)
  = 7/(15e)
  = 0.171677
(右) x^2 > x^3 より
 I < ∫[0,1] (x^2)e^(-x^3) dx
  = (1/3)[ -e^(-x^3) ](x=0,1)
  = (1/3)(1 - 1/e)
  = 0.210706852
または 相加・相乗平均より
 x^2 < (1/3)x + (3/4)x^3,
 I < ∫[0,1] {(1/3)x + (3/4)x^3}・e^(-x^2) dx
  = [ -(1/24)(13 + 9x^2)e^(-x^2) ](x=0,1)
  = (1/24)(13 - 22/e)
  = 0.204443845

【問題3】
 f(x) = (x + 1/x)^2 は下に凸だから、Jensen で一発だが、
x=1/3 で接線を曳いて
 f(x) = 100/9 - (160/3)(x -1/3) + (x^2 +54x +9)(x -1/3)^2
    ≧ 100/9 - (160/3)(x -1/3),
 f(a) + f(b) + f(c) ≧ 100/3 - (160/3)(a+b+c-1) = 100/3,
でもよい。
516132人目の素数さん:2011/08/21(日) 05:44:22.87
【問題4】
正の数 a、b、c が abc=1 をみたすとき、
(a - 1 + 1/b)(b - 1 + 1/c)(c - 1 + 1/a) ≦ 1 を示せ

   ∧,,∧    
   (`・ω・´)    詳しく聞こうか?
   (    )
 ̄ ̄Φ口U ̄ ̄\
   _ _.        \
_(    )____.\
 ̄┏┳┓)
517132人目の素数さん:2011/08/21(日) 06:18:55.54
>>514

【追加問題1】
 f(x) = (x + 1/x)^n は下に凸だから Jensen で一発だが、
x=1/3 で接線を曳いて
 f(x) ≧ (10/3)^n - 8n(10/3)^(n-1)・(x -1/3),
 f(a) + f(b) + f(c) ≧ 3(10/3)^n - 8n(10/3)^(n-1)(a+b+c-1)
  = 3(10/3)^n,
としてもよい。

【追加問題2】
 (abc)^(1/3) = G とおく。(相乗平均)
相加・相乗平均で
 aa/b + bb/c + cc/a ≧ 3G,
 a/bb + b/cc + c/aa ≧ 3/G,
 3G + 3/G ≧ 6,
より、【1】に帰着する。
 3(10/3)^3 = 1000/9
518132人目の素数さん:2011/08/21(日) 06:57:11.66
>>516

【問題4】
 abc=1 とあるから当然、 a=y/z, b=z/x, c=x/y と置くんだろうな。
 (左辺) = {(y-z+x)/z}{(z-x+y)/x}{(x-y+z)/y},

定義により、-x+y+z, x-y+z, x+y-z の任意の2つの和は正だから、
 正でないのは高々1つだけ。
・1つが正でない場合、明らかに成り立つ。
・3つとも正の場合、 相乗・相加平均より
 √{(x-y+z)(y-z+x)} = √{x^2 - (y-z)^2} ≦ x,
 √{(y-z+x)(z-x+y)} = √{y^2 - (z-x)^2} ≦ y,
 √{(z-x+y)(x-y+z)} = √{z^2 - (x-y)^2} ≦ z,
辺々掛けて
 (x-y+z)(y-z+x)(z-x+y) ≦ xyz,

[第3章.481] 
519132人目の素数さん:2011/08/21(日) 13:58:30.54
>>515 【問題2】(右)

詳しく聞かれちゃ〜生姜ねぇ・・・

∫[0,1] x e^(-x^2) dx = ∫[0,1] (1/2)e^(-t) dt = [ -(1/2)e^(-t) ](t=0,1) = 1/2 - 1/(2e) = 0.3166028,

∫[0,1] (x^3)e^(-x^2) dx = ∫[0,1] (1/2)t e^(-t)dt = [ -(1/2)(t+1)e^(-t) ](t=0,1) = 1/2 - 1/e = 0.13212056

ここでシュワルツを使えば I < 0.2045232 だな。
520132人目の素数さん:2011/08/21(日) 14:13:02.30
>>515 【問題2】

exp( ) をマクローリン展開して計算すると I = 0.189472345820492
521132人目の素数さん:2011/08/21(日) 23:44:35.95
【もんじあ】
実数 x、y、z が (x+y-z)^2 + (y+z-x)^2 + (z+x-y)^2 = 1
をみたすとき、|x+y+z| の 最大値を求めよ
522132人目の素数さん:2011/08/22(月) 00:31:52.68
>>521
 g(x,y,z) = (x+y-z)^2 + (y+z-x)^2 + (z+x-y)^2
     = (1/3)(x+y+z)^2 + (8/3)(x^2 +y^2 +z^2 -xy -yz -zx)
     = (1/3)(x+y+z)^2 + (4/3){(x-y)^2 + (y-z)^2 + (z-x)^2}
     ≧ (1/3)(x+y+z)^2,
∴ |x+y+z| ≦ √{3g(x,y,z)},
 等号成立は x=y=z=±1/√3 のとき。
523132人目の素数さん:2011/08/22(月) 01:28:08.98
g(x,y,z)≡1だから、最大値は√3か
524132人目の素数さん:2011/08/22(月) 04:12:25.39
[B.4355.]
 x,y,z は正数で、xyz=1 とする。このとき次を示せ。
 (x^3+y^3)/(x^2+xy+y^2) + (y^3+z^3)/(y^2+yz+z^2) + (z^3+x^3)/(z^2+zx+x^2) ≧ 2.

http://www.komal.hu/verseny/feladat.cgi?a=honap&h=201104&t=mat&l=en
525132人目の素数さん:2011/08/22(月) 04:26:02.52
 三角形ABCの内部の点Pに対して PA+PB < CA+CB が成り立つ。

[B.4339.]
 四面体ABCDの内部の点Pに対して PA+PB+PC < DA+DB+DC が成り立つか?

http://www.komal.hu/verseny/feladat.cgi?a=honap&h=201102&t=mat&l=en
526132人目の素数さん:2011/08/23(火) 00:30:57.36
[B.4355.] (訂正)
 x,y,z は正数で、xyz=1 とする。このとき次を示せ。
 (z^3 + y^3)/(x^2+xy+y^2) + (x^3 + z^3)/(y^2+yz+z^2) + (y^3 + x^3)/(z^2+zx+x^2) ≧ 2.


>>524 なら瞬殺だろうな.... >>514
527132人目の素数さん:2011/08/23(火) 04:59:16.48
(゚д゚;) ト、トウゼン デ ゴザルヨ…
528132人目の素数さん:2011/08/23(火) 12:50:44.75
>>526-527

相加・相乗平均を使えば >>524 と同じ....
 (左辺) ≧ 3{f(x,y)f(y,z)f(z,x)}^(1/3),

 x^2 +xy +y^2 = 3(x^2 -xy+y^2) - 2(x-y)^2 ≦ 3(x^2 -xy +y^2), より
 f(x,y) = (x^3 + y^3)/(x^2 +xy +y^2)
  ≧ (1/3)(x^3 + y^3)/(x^2-xy+y^2)
  = (1/3)(x+y),
再度、相加・相乗平均より
 (左辺) ≧ {(x+y)(y+z)(z+x)}^(1/3)
  = {8xyz + x(y-z)^2 + y(z-x)^2 + z(x-y)^2}^(1/3)
  ≧ 2(xyz)^(1/3),
529132人目の素数さん:2011/08/23(火) 16:00:00.77
AB<ACでBの近くにDをとり,Cの近くにPをとる。
530132人目の素数さん:2011/08/23(火) 19:51:01.60
>>524
 x,y,z は正数で、x+y+z=3 とする。このとき……
531132人目の素数さん:2011/08/24(水) 21:08:58.13
>>498
第10回(2011年)中国女子数学オリンピック(CGMO)の問題3

http://www.imojp.org/
http://www.imojp.org/challenge/index.html 過去問
532132人目の素数さん:2011/08/24(水) 22:01:55.57
(*゚∀゚)=3 ハァハァ…
533132人目の素数さん:2011/08/24(水) 22:07:45.91
>>531
中華の問3、どっかで見たような希ガス…
534132人目の素数さん:2011/08/25(木) 05:16:58.00
535132人目の素数さん:2011/08/25(木) 06:57:10.69
536132人目の素数さん:2011/08/25(木) 16:54:26.26
そういや3年位前に、高校の先生が相加相乗平均の新証明の記事があったけど、
いまさらながら、その論文のリンクを貼っておく
http://www.emis.de/journals/JIPAM/images/080_08_JIPAM/080_08.pdf

並べ替え不等式を使うのか…
537132人目の素数さん:2011/08/25(木) 23:04:28.49
>>526
 x,y,z は正数で xy+yz+zx = 3 とする。このとき……
538132人目の素数さん:2011/08/26(金) 07:19:11.26
0
539132人目の素数さん:2011/08/26(金) 08:57:16.76
x, y, zは正の実数で x+y+z=11 , x≦2, y≦3 のとき √(xyz) ≦6 .
540132人目の素数さん:2011/08/26(金) 09:07:33.95
>>539
どうやるん?
541132人目の素数さん:2011/08/26(金) 13:08:56.28
>>536
その方法と 全 く 同 じ 方 法 で、
色々な不等式(もちろん相加相乗平均も)を証明した記事が、
数学セミナーに掲載されている。

数学セミナー 2004.2
ttp://www.nippyo.co.jp/magazine/4352.html
>対称性を有する不等式の統一的証明について 仁平政一 52

↑この記事。2004年だから、例の高校の先生より早い。
542132人目の素数さん:2011/08/26(金) 13:19:33.62
>>541
なんと! すごいな
543132人目の素数さん:2011/08/26(金) 13:21:54.25
さて、不等式ヲタ ≒ 三角関数 ≒ nCrヲタであるから、次の問題

【問題】 Σ[r = 0、n] (-1)^k * nCr / (x+r) = ?
544訂正:2011/08/26(金) 13:22:23.41
543 名前:132人目の素数さん[sage] 投稿日:2011/08/26(金) 13:21:54.25
さて、不等式ヲタ ≒ 三角関数 ≒ nCrヲタであるから、次の問題

【問題】 Σ[r = 0、n] (-1)^r * nCr / (x+r) = ?
545541:2011/08/26(金) 13:32:26.78
記事名をキーワードにググってみたら、
数研通信とかいうサイトに まるごと載ってるじゃねーか(^o^)

数研通信 47号2003年8月
不等式の証明の統一的方法(仁平政一)
ttp://www.chart.co.jp/subject/sugaku/suken_tsushin/47/47-5.pdf


>541と若干タイトルが違うが、著者は同じ。で、こっちの方が
さらに年月が古く、2003年8月となっている。

>541のやつは、この記事の加筆修正なのかもしれん(俺の手元に
数セミが無いので、確認できない^o^)。
546132人目の素数さん:2011/08/26(金) 13:40:03.23
>>545
情報サンクス!
数蝉の年2回のNOTEは、コピーしてファイルしてるので見たけど、
数検通信の記事から抜粋したものですな

で、この方法は >>2 参考文献[1] P.25の方法と同じな希ガス…
547541:2011/08/26(金) 13:57:35.26
>で、この方法は >>2 参考文献[1] P.25の方法と同じな希ガス…
ということは、並べ替え不等式を使う方法は
ずっと昔から知られていたと。
548132人目の素数さん:2011/08/26(金) 21:17:47.93
541>所謂, Rearrangememt Inequalityですな。

>>544 int_0^1 x^2 dxは?
549132人目の素数さん:2011/08/27(土) 02:57:18.25
>>539-540

  xy -(x+y+1) +2 = (x-1)(y-1) ≦ 2,
∴ xyz = xy(11-x-y) ≦ (1+x+y)(11-x-y) = 36 - (5-x-y)^2 ≦ 36,
∴ √(xyz) ≦ 6,
550132人目の素数さん:2011/08/27(土) 03:29:51.55
>>549
1行目が思いつかない
どういう発想で、こういう解法に辿りついたのか知りたいです
数字が変わっても、このやり方は使えるのですか?
551132人目の素数さん:2011/08/27(土) 04:35:47.24
>>544
f_n(x) = Σ[r = 0 to n] (-1)^r * nCr / (x+r) とおくと
  f_{n+1}(x) = f_n(x) - f_n(x+1)
552132人目の素数さん:2011/08/27(土) 04:47:31.94
B[x,n+1]だろ
553132人目の素数さん:2011/08/27(土) 05:25:31.22
>>552
なにそれ?
554132人目の素数さん:2011/08/27(土) 13:27:34.85
a,b,cが三角形の三辺の長さのとき

1/4<(a+b)(b+c)(c+a)/(a+2b)(b+2c)(c+2a)


を示せ


高1の宿題です
さっぱりわかりません


555132人目の素数さん:2011/08/27(土) 15:42:42.51
4(a+b)(b+c)(c+a)-(a+2b)(b+2c)(c+2a)
={4(b+c)*a^2 + 4(b+c)^2*a + 4bc(b+c)} - {2(b+2c)*a^2 + (b+2c)(4b+c)*a + 2bc(b+2c)}
=2b*a^2 + (4b^2+8bc+4c^2-4b^2-9bc-2c^2)*a + 2b^2c
=2b*a^2 + (-bc+2c^2)*a + 2b^2c
=2(ba^2+cb^2+ac^2)-abc
=6*(1/3)*(ba^2+cb^2+ac^2)-abc
≧6*abc-abc  (相加相乗平均 等号成立はba^2=cb^2=ac^2⇔a=b=c)
=5abc>0
556132人目の素数さん:2011/08/27(土) 15:51:37.29
>>555じゃないが、「三角形の三辺の長さ」って条件必要か?
557132人目の素数さん:2011/08/27(土) 16:00:47.53
高1で3次の相加相乗平均を勝手に使っていいか不明だし
三角形であることをうまく使って証明できるのかもしれない。
やり方がわからんのだけど。
558132人目の素数さん:2011/08/27(土) 16:09:52.09
>高1で3次の相加相乗平均を勝手に使っていいか不明
うろ覚えだが、2次の相加相乗平均でさえ習うのは高2だったような。
まあa^3+b^3+c^3-3abc=(a+b+c){(a-b)^2+(b-c)^2+(c-a)^2}/2から直接導けるが。
何か数T,A範囲で証明できる方法があるのかね。
559132人目の素数さん:2011/08/27(土) 16:46:12.93
>>555
三角形の条件は?
560132人目の素数さん:2011/08/27(土) 20:16:44.76
つまり、三角形の3辺をなす正の数 a、b、c でなくても成立する不等式だったと…
出題者は、三角形の成立条件を考慮した上で、もっと厳しい評価式を出題しろってこった!
561132人目の素数さん:2011/08/27(土) 20:40:02.91
>>560

a,b,cが三角形の三辺の長さのとき

8/27 ≦ (a+b)(b+c)(c+a)/{(a+2b)(b+2c)(c+2a)},

を示せ。

等号成立は a=b=c のとき。
562132人目の素数さん:2011/08/27(土) 20:42:15.42
てめえが示せこの野郎!
563132人目の素数さん:2011/08/27(土) 21:41:01.83
>>562
君は口が悪いな、このスレにふさわしくない
さっさと、夜光灯を振る仕事に戻るんだ!
564132人目の素数さん:2011/08/27(土) 22:02:32.00
三角形の辺の長さに関する不等式について検索したら…
不等式プロがヒットした!
http://www.researchgate.net/publication/41797900__()
565132人目の素数さん:2011/08/28(日) 01:06:08.23
>>561

27(a+b)(b+c)(c+a) - 8(a+2b)(b+2c)(c+2a)
 = 11(aab +bbc +cca) -5(abb +bcc +caa) -18abc
 = (17/3){2(aab +bbc +cca) -(abb +bcc +caa) -3abc}
  +(1/3){-(aab +bbc +cca) +2(abb +bcc +caa) -3abc}
 = (17/3)P + (1/3)Q,

三角不等式より
 2P = 4(aab +bbc +cca) -2(abb +bcc +caa) -6abc
  = (b+c-a)(a-b)^2 + (c+a-b)(b-c)^2 + (a+b-c)(c-a)^2 ≧ 0,
 2Q = -2(aab +bbc +cca) +4(abb +bcc +caa) -6abc
  = (c+a-b)(a-b)^2 + (a+b-c)(b-c)^2 + (b+c-a)(c-a)^2 ≧ 0,
566132人目の素数さん:2011/08/28(日) 04:33:06.72
>>544
n!/{x(x+1)(x+2)…(x+n)}
567132人目の素数さん:2011/08/28(日) 05:36:37.46
>>561
三角形の3辺だから
 a=q+r, b=r+p, c=p+q,      >>273
とおく。p,q,r≧0

27(a+b)(b+c)(c+a) - 8(a+2b)(b+2c)(c+2a)
 = 27(q+2r+p)(r+2p+q)(p+2q+r) - 8(q+3r+2p)(r+3p+2q)(p+3q+2r)
 = 6(p^3 +q^3 +r^3) -11(ppq+qqr+rrp) +5(pqq+qrr+rpp)
 = (17/3){p^3 + q^3 + r^3 -2(ppq+qqr+rrp) +(pqq+qrr+rpp)}
  +(1/3){p^3 + q^3 + r^3 +(ppq+qqr+rrp) -2(pqq+qrr+rpp)}
 = (17/3)P + (1/3)Q,

 P = p^3 + q^3 + r^3 -2(ppq+qqr+rrp) +(pqq+qrr+rpp)
  = p(p-q)^2 + q(q-r)^2 + r(r-p)^2 ≧ 0,
 Q = p^3 + q^3 + r^3 +(ppq+qqr+rrp) -2(pqq+qrr+rpp)
  = q(p-q)^2 + r(q-r)^2 + p(r-p)^2 ≧ 0,

変わり映えしねぇ.....
568132人目の素数さん:2011/08/28(日) 05:59:42.21
>>29-31 >>44
 a,b,c ≧0,
 m = min{a,b,c}
 {a,b,c} = {m,m+x,m+x+y}
とおく。
 a+b+c=s, ab+bc+ca=t, abc=u として
 s^2 -3t = x^2 +xy +y^2,
 st-9u = 2m(x^2+xy+y^2) + x(x+y)(2x+y),
 |處 = |(a-b)(b-c)(c-a)| = xy(x+y),

 6s^3 -21st + 27u = 12m(x^2+xy+y^2) + 3(2x^3 +3xxy +5xyy +2y^3)
  > 3(2x^3 +3xxy +5xyy +2y^3)
  ≧ 3xy(5x+7y)
  > 15xy(x+y)
  = 15|處,
569132人目の素数さん:2011/08/28(日) 06:06:32.37
>>494 の類題

a,b,cを実数、 = (a-b)(b-c)(c-a)、とするとき
 a^4 + b^4 + c^4 + (a+b+c) ≧ (1/27)(a+b+c)^4,
を示せ。 (こってうし)

http://www.casphy.com/bbs/test/read.cgi/highmath/1169210077/672
570132人目の素数さん:2011/08/28(日) 21:55:29.60
 I+J+K+L+N = 0 のとき

f(x,y,z) = N(x^4 + y^4 + z^4) + I(yx^3 + zy^3 + xz^3) + J(xy^3 + yz^3 + zx^3) + K(xxyy+yyzz+zzxx) + Lxyz(z+y+z),

を平方和で表わせ。ただし、N = A^2 + B^2 + C^2 とする。
571132人目の素数さん:2011/08/28(日) 22:08:28.25
>>570

f(x,y,z) = (Ax^2 + By^2 + Cz^2 + Pyz + Qzx + Rxy)^2 + cyclic. + K '{xxyy+yyzz+zzxx-xyz(x+y+z)}

とおいて、係数 P,Q,R を求めよう。 ここに
 K ' = K - P^2 - Q^2 - R^2 - 2(AB+BC+CA),

まづ
 P + Q + R = - (A+B+C),
 CP + AQ + BR = I/2,
 BP + CQ + AR = J/2,
より
 AP + BQ + CR = -(I+J)/2 -(A+B+C)^2,
クラメルの公式より
 P = {I(B-A) + J(C-A) + 2(A+B+C)(BC-AA)}/D,
 Q = {I(C-B) + J(A-B) + 2(A+B+C)(CA-BB)}/D,
 R = {I(A-C) + J(B-C) + 2(A+B+C)(AB-CC)}/D,
ここに
 D = 2(A^2 + B^2 + C^2 -AB -BC -CA) = (A-B)^2 + (B-C)^2 + (C-A)^2 ≧ 0,

 P^2 + Q^2 + R^2 = (A+B+C)^2 + {(II+IJ+JJ) + 2(I+J)(A+B+C)^2 + 4(AB+BC+CA)(A+B+C)^2}/D,

 PQ + QR + RP = −(1/2){(II+IJ+JJ) + 2(I+J)(A+B+C)^2 + 4(AB+BC+CA)(A+B+C)^2}/D,

これを使えば K ' を計算できる。

 K '≧0 なら平方和になる。そのためには、|A+B+C| がなるべく小さくなるように符号をとるとよい。
572132人目の素数さん:2011/08/28(日) 22:28:51.81
>>571 補足

 xxyy + yyzz + zzxx - xyz(x+y+z) = (1/2){x(y-z)}^2 + (1/2){y(z-x)}^2 + (1/2){z(x-y)}^2 ≧ 0,

573132人目の素数さん:2011/08/29(月) 01:00:36.43
1991 IMO 1/4<(a+b)(b+c)(c+a)/(a+b+c)^3≦8/27
574132人目の素数さん:2011/08/29(月) 01:19:05.81
1/x^4+1/y^4+1/z^4+1/w^4+9/(x^4+y^4+z^4+w^4)
≧8/9(1/x^2y^2+1/x^2z^2+1/x^2w^2+1/y^2z^2+1/y^2w^2+1/z^2w^2)

≧11/3(x^4+y^4+z^4+w^4)

≧25/4xyzw
575132人目の素数さん:2011/08/29(月) 01:40:00.54
x=y=z=w=1.

25/4>=4/27>=11/12>=25/4.
576132人目の素数さん:2011/08/29(月) 02:17:54.40
1/x^4+1/y^4+1/z^4+1/w^4+9/(x^4+y^4+z^4+w^4)
≧(8/9)(1/x^2y^2+1/x^2z^2+1/x^2w^2+1/y^2z^2+1/y^2w^2+1/z^2w^2)

+11/3(x^4+y^4+z^4+w^4)

≧25/4xyzw

だね。
577132人目の素数さん:2011/08/29(月) 02:38:43.69
>>573

右: 4(a+b)(b+c)(c+a) - (a+b+c)^3 = aa(b+c-a) + bb(c+a-b) + cc(a+b-c) + 2abc > 0,
左: 相加相乗平均
  8(a+b+c)^3 -27(a+b)(b+c)(c+a)
  = 3(a+b)(a-b)^2 + 3(b+c)(b-c)^2 + 3(c+a)(c-a)^2 + 2(a^3+b^3+c^3-3abc) ≧ 0,
578132人目の素数さん:2011/08/29(月) 20:25:51.28
a^n+b^n<c^n
となる整数a,bをcで表しなさい。
579132人目の素数さん:2011/08/29(月) 22:27:55.11
 n≧4 で 2^(n-1) < n^(n-2)

を、帰納法以外で示したいのですが
どうすればいいでしょうか。
580132人目の素数さん:2011/08/29(月) 23:14:01.40
>>579

 n ≦ 2(n-2),
 2^n ≦ 2^{2(n-2)} = 4^(n-2) ≦ n^(n-2),
581132人目の素数さん:2011/08/29(月) 23:49:13.57
おおっ
不等式のプロにかかるとさすがにアッサリですね。
ありがとうございます。>>580
582132人目の素数さん:2011/08/30(火) 00:40:02.56
2^2<=n.
2^(n-3)<n^(n-3).
2^(n-1)<n^(n-2).
583132人目の素数さん:2011/08/30(火) 07:06:53.80
>>546-547

で、この方法は >>2 参考文献[3] P.71の方法4.と同じな希ガス…

 x_(n-1) ≦ G ≦ x_n,
を仮定して
 x_(n-1) + x_n - {x_(n-1)・x_n /G + G} = (x_n - G){G - x_(n-1)}/G ≧ 0,
 x_(n-1) + x_n ≧ {x_(n-1)・x_n /G} + G,
を導いています。
584132人目の素数さん:2011/08/30(火) 07:12:31.51
つまり既出の証明でも専門誌に発表できるということですね
585132人目の素数さん:2011/08/30(火) 07:59:46.12
対称性に注目って不等式考える上では突飛なアイデアじゃないよね
ってか定跡やん。これを「新証明」と主張することに不安は感じなかったのだろうか。
586132人目の素数さん:2011/08/30(火) 10:13:00.43
>>573がその後発展してなくて涙目の住民ワロス
587132人目の素数さん:2011/08/30(火) 12:50:39.36
>>573に書き込んだのに誰からもレスされなくて、
あまりのくやしさに>>586で書き込んだのだった

涙拭けよ
588132人目の素数さん:2011/08/30(火) 12:53:45.10
何言ってだこいつら
589132人目の素数さん:2011/08/30(火) 12:59:42.97
いつもの荒らしでしょう
590132人目の素数さん:2011/08/30(火) 17:51:18.51
そもそも573は問題の解釈が間違っている
真ん中はこの式にはならない
これ書いたやつ馬鹿すぎ
591132人目の素数さん:2011/08/30(火) 17:56:41.46
1991問題は
三角形の内心をI、二等分線をAA'BB'CC'とするとき
AIBICI/AA'BB'CCがこの範囲を示せだが
592132人目の素数さん:2011/08/30(火) 18:38:17.84
AI/AA'=(b+c)/(a+b+c) etc.
593132人目の素数さん:2011/08/30(火) 23:19:26.11
>>573 △の辺だから
 >>577


>>592

 AI/AA' = (△ABC-△BCI)/(△ABC)

 △BCI = (1/2)ar, △ABC = (1/2)(a+b+c)r, を入れる。
(rは△ABCの内接円の半径)
594132人目の素数さん:2011/08/31(水) 03:30:40.14
>>570-572

〔例1〕  >>268
 2倍すると
 N=2, I=-6, J=0, K=4, L=0,
 (A,B,C)=(1,-1,0) すると (P,Q,R)=(2,-1,-1)  K'=0,
 >>284-290

〔例2〕  >>494 >>504
 27倍すると
 N=19, I=-5, J=49, K=33, L=-96,
 (A,B,C)=(3,-3,1) (P,Q,R)=(-22/4,11/4,7/4) K ' = 3/8,
 (A,B,C)=(3,-3,-1) (P,Q,R)=(-131/28,46/28,113/28) K ' = 3/8,

〔例3〕  >>569
 27倍すると、
 N=26, I=-31, J=23, K=-6, L=-12,
 (A,B,C)=(4,-1,-3) (P,Q,R)=(-2/26,59/26,-57/26) K '= 29/78
 (A,B,C)=(4, 1,-3) (P,Q,R)=(-144/74,141/74,-145/74) K ' = 13/74,
595132人目の素数さん:2011/08/31(水) 07:09:17.96
>>592
間違ってるんだが

596132人目の素数さん:2011/08/31(水) 07:12:33.43
BA'が(a+b)/(2a+b+c)
AI/AA'=(a+b)/(a+b+BA')

を考えると明白な間違い
597132人目の素数さん:2011/08/31(水) 07:23:05.21
>>596
>BA'が(a+b)/(2a+b+c)

BA'、a、b、cって長さ?
次元が違うんだが
598132人目の素数さん:2011/08/31(水) 07:26:24.06
二等分線だからこういうふうな比になるだろ
なんで分からないの?馬鹿は死ねよ
599132人目の素数さん:2011/08/31(水) 07:27:50.44
          / ̄ ̄ ̄\
         /   ⌒  ⌒ ヽ
         /  ィ●ァ  ィ●ァ |
         |           |
         |     c{ っ  |
         |    __   }   うーっす
        /、.    ー    ヽ
       /            |
       |           | /
       ヽ_|  ┌──┐ |丿
         |  ├──┤ |
         |  ├──┤ |
600132人目の素数さん:2011/08/31(水) 07:28:24.02
間違いに気付いたか
馬鹿め
601132人目の素数さん:2011/08/31(水) 07:37:45.00
訂正

BA'が(a+b)(b+c)/(2a+b+c)
AI/AA'=(a+b)/(a+b+BA')

602132人目の素数さん:2011/08/31(水) 07:39:19.07
上を下に入れると
a+bがきれいにきえて


(2a+b+c)/2(a+b+c)


になるんで、上のほうの解答は大間違い
603132人目の素数さん:2011/08/31(水) 07:41:40.55
>>601
a=b=cのときBA'=(a+b)(b+c)/(2a+b+c)=a=BCになるんだが
604132人目の素数さん:2011/08/31(水) 07:53:35.71
解答が自動化してるイカサマ師が何を言っても恥ずかしいだけ
605132人目の素数さん:2011/08/31(水) 07:59:55.86
ここまで飛ばし読みした俺様に、修正バージョンを書いてくれ
606132人目の素数さん:2011/08/31(水) 08:36:42.89
適当にでっちあげた式にでっちあげた式を入れる遊びは楽しいかね
607132人目の素数さん:2011/08/31(水) 17:25:11.12
     凵@    ○   ∇ 、___,、´`゙;~、  ';冫 ☆
           ┏  ━ゝヽ''/  ≧ \━〆A!゚━━┓。
 ╋┓"〓┃  < ゝ\',冫。' |::::  \ ./ |゛△│´'´,.ゝ'┃.      ●┃ ┃┃
 ┃┃_.━┛ヤ━━━━━━|::::: (● (● |━━━━━━━━━  ━┛ ・ ・
        ∇  ┠─Σ-  ヽ::::... .ワ.....ノ  冫 そ',´; ┨'゚,。
           .。冫▽ <   ⊂     ./⊃     乙  ≧   ▽
         。 ┃   Σ   (⌒ゞ ,l, 、''  │   て く
           ┠─ム┼   ゝ,,ノ ノゝ. 、,, .┼ ァ Ζ┨ ミo''`
         。、゚`。、   i/   レ' o。了 、'' ×  个o
        ○  ┃   `、,~´+√ ▽   ',!ヽ.◇    o┃
            ┗〆━┷ Z,.' /┷━''o ヾo┷+\━┛,゛;
       ヾ   凵@              '、´    ∇

荒れたスレに不等式ヲタが光臨! 整理すると以下の如しだ!

【1991 IMO 問1】
△ABCの内心をI、二等分線をAA'BB'CC'とするとき、
 1/4 < (AI・BI・CI)/(AA'・BB'・CC) ≦ 8/27

【証明】
>>592
角の二等分線の定理から、容易に
 AI/AA' = (b+c)/(a+b+c)、BI/BB' = (c+a)/(a+b+c)、CI/CC' = (a+b)/(a+b+c)
>>573
示すべき不等式は
  1/4 < (a+b)(b+c)(c+a)/(a+b+c)^3 ≦ 8/27
>>577
右: 4(a+b)(b+c)(c+a) - (a+b+c)^3 = aa(b+c-a) + bb(c+a-b) + cc(a+b-c) + 2abc > 0,
左: 相加相乗平均
  8(a+b+c)^3 -27(a+b)(b+c)(c+a)
  = 3(a+b)(a-b)^2 + 3(b+c)(b-c)^2 + 3(c+a)(c-a)^2 + 2(a^3+b^3+c^3-3abc) ≧ 0,
608132人目の素数さん:2011/08/31(水) 17:50:46.29
>>590>>595-602 をあぼーんすればよろし
609132人目の素数さん:2011/08/31(水) 18:59:22.02
なんで590、598、600は偉そうなの? 馬鹿なのに
610132人目の素数さん:2011/08/31(水) 19:52:41.70
a>0 のとき (a-x)^n + (a+x)^n > 2a^n

って明らかですか?どう示せばいいでしょうか。
611610:2011/08/31(水) 19:53:43.74
間違えました。>じゃなくて≧でした。
(a-x)^n + (a+x)^n ≧ 2a^n です。
612132人目の素数さん:2011/08/31(水) 20:13:46.88
n≧1かな?
凸不等式でおk
613132人目の素数さん:2011/08/31(水) 20:35:31.95
変な質問ですが、「不等式評価」って言葉はありますか?
クラスの数学得意なやつが使ってたんですが、先生も初めて聞いたと言っていました。
614132人目の素数さん:2011/08/31(水) 20:46:46.40
不等式で評価する


って普通に使うね。
615132人目の素数さん:2011/08/31(水) 21:48:26.70
進学校じゃないかぎり学校の先生は大抵教育学部出身だから、評価estimateとか言っても基本的には通じない。

数学に限ったことじゃないけど本当に「小中高の勉強」ができてた奴は教師になってない。
616132人目の素数さん:2011/08/31(水) 21:51:24.37
>>615
> 数学に限ったことじゃないけど本当に「小中高の勉強」ができてた奴は教師になってない。

なんで?
617132人目の素数さん:2011/09/01(木) 11:19:10.51
>>607
なんでa+b+cがでてくるんだよ。AB,ACは足したら2a+b+cだろうが
618132人目の素数さん:2011/09/01(木) 11:42:35.32
何言ってだこいつ
619132人目の素数さん:2011/09/01(木) 12:14:01.46
しーっ、目を合わせちゃいけません
620132人目の素数さん:2011/09/01(木) 16:33:02.11
a+b+cってどこにあるの
621132人目の素数さん:2011/09/01(木) 22:15:08.17
上から評価、下から評価

とか言った使い方をよくする
622132人目の素数さん:2011/09/01(木) 22:41:35.38
>>525

〔補題〕 
AB ≦ CA, CB のとき、
三角形ABCの内部の点Pに対して PA + PB + PC < CA + CB.
623132人目の素数さん:2011/09/01(木) 22:45:57.36
>>622

(略証)
Pを直線上で動かすとき、AP,BP,CP は下に凸(*)だから
 f(P) = AP+BP+CP も下に凸。
直線CPと辺ABの交点をQ とすると、凸性から
 f(P) < max{f(C), f(Q)}
ところで 題意より
 f(Q) = (AQ+QB) + CQ = AB + CQ ≦ AB + max{CA,CB} ≦ CA + CB = f(C),
∴ f(P) < f(C),

* この直線をt軸とすると g(t) = √(a^2 + t^2) は
  a≠0 のとき双曲線。
  a=0 のとき g(t)=|t| でV字形の折れ線。
624132人目の素数さん:2011/09/02(金) 22:57:20.36
0<a、a≠1
((a^(2n+1)/(a-1))+(a(1-a^2n)/2n(1-a)^2)^2n)/a^n(2n+1)≧(2n)!
625132人目の素数さん:2011/09/03(土) 08:01:27.09
1 ≦ a、b、c ≦ 2 に対して、(a+b)/(b+c) + (b+c)/(c+a) + (c+a)/(a+b) の最大値(上限)は?
626132人目の素数さん:2011/09/04(日) 20:20:23.84
>>625

通分して
{(19/6) - (与式)}*(a+b)(b+c)(c+a)
 = (19/6)(a+b)(b+c)(c+a) - (c+a)(a+b)^2 - (a+b)(b+c)^2 - (b+c)(c+a)^2
 = (1/6){(aab+bbc+cca) + 7(abb+bcc+caa)} + (1/3)abc -(a^3 +b^3 +c^3)
 = (4/3)[k(aab+bbc+cca)+(1-k)(abb+bcc+caa)] + (1/3)abc -(a^3 +b^3 +c^3)  (k=1/8)
 = (1/4){10[k(aab+bbc+cca)+(1-k)(abb+bcc+caa)] -4(a^3+b^3+c^3) -15abc}
 + (7/12){-2[k(aab+bbc+cca) + (1-k)(abb+bcc+caa)] +7abc}
 = (1/4)F(k) + (7/12)G(k)
 ≧ 0,
627626:2011/09/04(日) 20:25:44.92
>>625 (続き)

〔補題1〕
-1/5≦k≦6/5 のとき
 F(k) = 10[k(aab+bbc+cca)+(1-k)(abb+bcc+caa)] -4(a^3+b^3+c^3) -15abc ≧ 0,
(略証)
 (2a-b)(2b-a)(2a-c) + c.c. = 12(aab+bbc+cca) - 2(abb+bcc+caa) -4(a^3 +b^3 +c^3) -15abc ≧ 0,
 (2a-b)(2b-a)(2b-c) + c.c. = -2(aab+bbc+cca) +12(abb+bcc+caa) -4(a^3 +b^3 +c^3) -15abc ≧ 0,
から。

〔補題2〕
-1≦k≦2 のとき
 G(k) = -2[k(aab+bbc+cca) + (1-k)(abb+bcc+caa)] +7abc ≧ 0,
(略証)
 (2a-b)(2b-c)(2c-a) = -4(aab+bbc+cca) +2(abb+bcc+caa) +7abc ≧ 0,
 (2b-a)(2c-b)(2a-c) = 2(aab+bbc+cca) -4(abb+bcc+caa) +7abc ≧ 0,
から。
628132人目の素数さん:2011/09/04(日) 23:58:47.65
〔類題〕
1 ≦ a,b,c,d ≦ 2 に対して
 4 ≦ (a+b)/(b+c) + (b+c)/(c+d) + (c+d)/(d+a) + (d+a)/(a+b) < 11/2.

[第2章.325-326 , 514-519]
 
上限(〜17/4)を出すのは大変でござるよ、ニンニン。 ( ゚∀゚)
629132人目の素数さん:2011/09/05(月) 01:06:11.79
ついでに....

>>102
 [第2章.643-645]

>>350-356
 [第2章.780 , 786-818]
630132人目の素数さん:2011/09/05(月) 01:54:00.74
>>628

題意より (a-c)/(b+c) ≦ 1/2, 4-b-c≧0.
加比の理 より、
 (a+b)/(b+c) = 1 + (a-c)/(b+c) ≦ 1 + [a-c +(4-b-c)/2]/[(b+c) +(4-b-c)] = 1 + [2 +a -(1/2)b -(3/2)c]/4.
循環的に加える。
 (左辺) ≦ 4 + [8-(a+b+c+d)]/4 ≦ 5.

[第2章.522,526]
631132人目の素数さん:2011/09/05(月) 03:01:55.82
>622

(略証)
点Pを通りCPに垂直な直線Lと 辺CA, 辺CB の交点を A', B' とする。
 CP < CA', CP < CB'

直線L上でPを動かしたとき、AP+BP は単一の極小をもつ。
∴ AP+BP < AA' + A'B または AP+BP < AB' + B'B のいずれかが成立。
 〔 LがBCと交わらない場合は △AA'B ⊃ △APB ∴ AP+BP < AA' + A'B〕

∴ AP+BP+CP < CA + A'B < CA + max{AB,CB} = CA + CB, または
  AP+BP+CP < AB' + CB < max{AB,CA} + CB = CA + CB,

[参考文献3] p.18-19, 例題10.(Visschersの問題)
632132人目の素数さん:2011/09/05(月) 21:11:27.93
>>625
 (a+b)/(b+c) + (b+c)/(c+a) + (c+a)/(a+b) < 3 + 3/10,
なら簡単だが.....
題意より (a-c)/(b+c) < 1/2,
加比の理より
 (a+b)/(b+c) = 1 + (a-c)/(b+c) < 1 + {(a-c)+(a-1)/2}/(a+b+c-1),
循環的にたす。
 (与式) < 3 + {(a+b+c-3)/2}/(a+b+c-1) < 3 + 3/10,


>>628 >>630
 (a+b)/(b+c) + (b+c)/(c+d) + (c+d)/(d+a) + (d+a)/(a+b) < 4 + 2/3
なら簡単だが.....
題意より (a-c)/(b+c) < 1/2,
加比の理より
 (a+b)/(b+c) = 1 + (a-c)/(b+c) < 1 + {(a-c)+(d+a-2)/2}/(a+b+c+d-2),
循環的にたす。
 (与式) < 4 + (a+b+c+d-4)/(a+b+c+d-2) < 4 + 2/3,
633132人目の素数さん:2011/09/05(月) 21:16:15.35
難し杉…
634132人目の素数さん:2011/09/06(火) 19:50:20.37
分かり松…
635132人目の素数さん:2011/09/07(水) 23:02:48.81
それっ桐…
636132人目の素数さん:2011/09/08(木) 10:20:53.57
ネタ切れ梅
637132人目の素数さん:2011/09/08(木) 10:24:34.78
次のネタ投函を待つ竹さ…
638132人目の素数さん:2011/09/09(金) 02:47:11.86
粟てず、ゆっ栗…
639132人目の素数さん:2011/09/09(金) 14:58:55.66
For a>1,b>1,c>1,Prove that for positive integer n

(a-1)(b-1)(c-1)n^3+[(a-1)(b-1)+(b-1)(c-1)+(c-1)(a-1)]n^2
+(a+b+c-3)n+1≦(abc)^n.
640132人目の素数さん:2011/09/10(土) 07:21:21.34
>>639

 LHS = {(a-1)n+1}{(b-1)n+1}{(c-1)n+1},

(i) For n=1, equality holds.

(ii) For n>1 and t≧-1, by AM-GM,
 f(t) = t^n -n(t-1) -1 = t^n -nt +(n-1)
  = (t^n + 1 + …… + 1) - nt
  = (t-1){t^(n-1) + t^(n-2) + …… + t -(n-1)}
  = (t-1)g(t)
  ≧ 0.            (*)
 Equality holds only if t=1.

*)
 For -1≦t<1, g(t) < 0.
 For t>1, g(t) > 0.
641132人目の素数さん:2011/09/10(土) 20:15:07.41
>>611
 nが偶数 または a>0 のとき
 (左辺) - (右辺) = 2Σ_(k=1,[n/2]) C(n,2k) a^(n-2k) x^(2k) ≧ 0,
 等号成立は x=0 のとき。

>>612
 nが奇数(>1)かつ |x| >a のとき ……
642132人目の素数さん:2011/09/10(土) 21:00:30.45
>>611
nについての帰納法による。

・n=1 のとき 等号成立。

・n>1 のとき
f_n(a,x) = (a-x)^n + (a+x)^n - 2a^n
    = a・f_(n-1)(a,x) + x{(a+x)^(n-1) - (a-x)^(n-1)}
  ≧ a・f_(n-1)(a,x),

 x>0 のとき a+x > |a-x|,
 x<0 のとき a-x > |a+x|,
 x{(a+x)^(n-1) - (a-x)^(n-1)} ≧ 0,
よって
 f_n(a,x) ≧ a・f_(n-1)(a,x) ≧ …… ≧ a^(n-1)・f_1(a,x) = 0,
643132人目の素数さん:2011/09/11(日) 00:59:16.87
>>625 >>628

文字の数をn個に拡張すると……

(a,b,c,……) の並びが (1,1,2) と (1,1,2,2) の組み合わせのとき、

n=4m  : n + n/16,
n=4m+1 : n + (n-9)/16 + 1/2,
n=4m+2 : n + (n-6)/16 + 1/3,
n=4m+3 : n + (n-3)/16 + 1/6,

∴ 最大値はこれ以上だが....
644132人目の素数さん:2011/09/11(日) 11:36:09.72
〔問題〕
正の実数 x,y,z が三角形(最大角θ)の3辺の長さとなるとき
 S = (x^2+y^2+z^2)/(xy+yz+zx),
のとりうる値の範囲を求めよ。(じゅー)

http://www.casphy.com/bbs/test/read.cgi/highmath/1169210077/679-
645132人目の素数さん:2011/09/11(日) 14:40:04.06
>>644 のあらすじ

正弦定理より
 S = {sin(A)^2 + sin(B)^2 + (sinθ)^2}/{sin(A)sin(B) + [sin(A)+sin(B)]sinθ},

A+B+θ = 180゚(θ≧60゚) より

 S = 2{2+cosθcos(A-B) -(cosθ)^2}/{4(1+cosθ)sin(θ/2) +cos(A-B) +cosθ}
これは |A-B| について単調増加。

∴ 2(2-cosθ)/{1+4sin(θ/2)} ≦ S < 2,
 左側の等号は A=B=(180゚-θ)/2,
 右側の不等号は {A,B}→{0゚, 180゚-θ}
646132人目の素数さん:2011/09/14(水) 00:12:23.53
For a,b,c>0 with a+b+c=3, Prove that
a/1+(b+c)^2+b/1+(a+c)^2+c/1+(a+b)^2≦3(a^2+b^2+c^2)/(a^2+b^2+c^2+12abc)
647132人目の素数さん:2011/09/14(水) 01:02:37.37
>>646
a/{1+(b+c)^2} のつもりだよな?(残り2つも)
648132人目の素数さん:2011/09/16(金) 22:27:40.55
【うんち問題】
a > b > 0 のとき、a + 1/{(a-b)b} ≧ 3

【本題】
正の数 x、y、z と正の有理数 a、b、c に対して、
  (x^a・y^b・z^c)/{(x+y+z)^(a+b+c)} ≦ (a^a・b^b・c^c)/{(a+b+c)^(a+b+c)}

       ___ 
彡     /  ≧ \    彡 ビュゥ……
  彡   |:::  \ ./ |  彡
      |:::: (● (●|    もう秋ですなぁ…
      ヽ::::......ワ...ノ    過去スレに a+b+c=1の場合があったような希ガス
        人つゝ 人,,         テヘッ!
      Yノ人 ノ ノノゞ⌒〜ゞ    
    .  ノ /ミ|\、    ノノ ( 彡
     `⌒  .U~U`ヾ    丿
             ⌒〜⌒
649132人目の素数さん:2011/09/17(土) 00:45:14.80
>>646
 a+b+c=s, ab+bc+ca=t, abc=u とおくと
 LHS = a/{(s/3)^2 +(s-a)^2} + b/{(s/3)^2 +(s-b)^2} + c/{(s/3)^2 +(s-c)^2}
   = 9(100s^5 -270s^3・t +378s^2・t +81tu)/(100s^6 -180s^4・t +324s^3・u +810s^2・t^2 -1458stu +729u^2),
 RHS = 9(a^2+b^2+c^2)/{(a+b+c)(a^2+b^2+c^2)+36abc}
   = 9(s^2 -2t)/(s^3 -2st+36u),
は使いたくないし...

>>648 【うんち】
 (a-b)b = (a/2)^2 - (a/2 - b)^2 ≦ (a/2)^2,

相加・相乗平均より
 (左辺) ≧ a + (2/a)^2
  = a/2 + a/2 + (2/a)^2
  = 3 + (1+a)(1 - 2/a)^2
  ≧ 3,
650132人目の素数さん:2011/09/17(土) 14:08:46.56
>>【本題】
正の数 x、y、z と正の有理数 a、b、c に対して、
  (x^a・y^b・z^c)/{(x+y+z)^(a+b+c)} ≦ (a^a・b^b・c^c)/{(a+b+c)^(a+b+c)}

Just use weighted AM-GM inequality. Done!
651132人目の素数さん:2011/09/17(土) 15:06:06.78
>>648 >>650

 L(X)=log(X) は上に凸なので、Jensen により

 a・L(x/a) + b・L(y/b) + c・L(z/c) ≦ (a+b+c)・L((x+y+z)/(a+b+c)),
652132人目の素数さん:2011/09/17(土) 18:48:40.09
ということで、>>648を修正すると…

---------------------------------------------------
非負実数 x、y、z と正の数 a、b、c に対して、
  (x+y+z)/(a+b+c) ≧ {(x/a)^a・(y/b)^b・(z/c)^c}^{1/(a+b+c)}
等号成立条件は、x/a = y/b = z/c のとき
---------------------------------------------------

これを使えば、次式も出てくるよね? 間違ってないかな?

正の数 x、y、z が x+y+z=1 をみたすとき、
  x^x・y^y・z^z ≧ (x^y・y^z・z^x + x^z・y^x・z^y)/2
  x^x・y^y・z^z ≧ x^{(y+z)/2}・y^{(z+x)/2}・z^{(x+y)/2}

もっと面白いのできないかな?

     |
 \  __  /
 _ (m) _ピコーン
    |ミ|
 /___\   
 ./  ≧ \  
 |::::  \ ./ |  
 |::::: (● (● | < 相加ッ! 相乗だったのか! ハァハァ…
 ヽ::::... .ワ.....ノ
653132人目の素数さん:2011/09/18(日) 21:34:16.82
For x, y, z>0 with xyz=1.

Prove that

(x+3)/[(x+1)^2]+(y+3)/[(y+1)^2]+(z+3)/[(z+1)^2]
654132人目の素数さん:2011/09/18(日) 21:35:58.39
For x, y, z>0 with xyz=1.

Find the maximum value of

(x+3)/[(x+1)^2]+(y+3)/[(y+1)^2]+(z+3)/[(z+1)^2]
655132人目の素数さん:2011/09/18(日) 21:37:38.42
Sorry for multi posts,

For x, y, z>0 with xyz=1.

Find the minimum value of

(x+3)/[(x+1)^2]+(y+3)/[(y+1)^2]+(z+3)/[(z+1)^2]
656132人目の素数さん:2011/09/19(月) 13:41:18.19
不等式か!

ハーディーと誰かがコレクション集だしてたよね

おまえ等、買った?
657132人目の素数さん:2011/09/19(月) 14:37:40.12
>>656
コレクションですと!
kwsk!
658132人目の素数さん:2011/09/19(月) 16:46:53.20
今日も自演操業乙であります!
659132人目の素数さん:2011/09/19(月) 21:32:54.72
660132人目の素数さん:2011/09/19(月) 22:09:41.76
>>659

つまり >637 によれば
 f(x) = (x-a)(x-b)(x-c) = x^3 -(a+b+c)x^2 +(ab+bc+ca) -abc,
とおくと
 f(x)f(-x) = (-x^2 +a^2)(-x^2 +b^2)(-x^2 +c^2)
  = -x^6 +(a^2+b^2+c^2)x^4 -{(ab)^2+(bc)^2+(ca)^2}x^2 +(abc)^2
  = -(x^2){(ab+bc+ca) + x^2}^2 + {abc + (a+b+c)x^2}^2, (恒等式)

x^2 = -1 を代入して
 (1+a^2)(1+b^2)(1+c^2) = (ab+bc+ca-1)^2 + (abc -a-b-c)^2,
661132人目の素数さん:2011/09/20(火) 12:41:18.89
Just use,1+a^2=(1+ai)(1-ai) Done!
662MaxValu:2011/09/21(水) 12:37:20.35
>>654

 (x+3)/(x+1)^2 = 1/(x+1) + 2/(1+x)^2 ≦ 3/(1+x),

 1/(x+1) + 1/(y+1) + 1/(z+1) = (3+2s+t)/(1+s+t+u)
 = 2 - (-1+t+u)/(1+s+t+u)
 < 2,
ここに、s=x+y+z≧3, t=xy+yz+zx≧3, u=xyz=1,
よって
 (与式) < 6,
上限に近づくのは、(例) x→0, y→0 のとき。
663Aeon:2011/09/21(水) 13:44:34.02
>>662 の訂正
 1/(x+1) + 1/(y+1) + 1/(z+1)
 = (3+2s+t)/(1+s+t+u)
 = 2 - (-1+t+2u)/(1+s+t+u)
 < 2,

>>655
  1/(x+1) + 1/(y+1) + 1/(z+1)
 = (3+2s+t)/(1+s+t+u),

  1/(x+1)^2 + 1/(y+1)^2 + 1/(z+1)^2
 = {(3-6u) + (4-2u)s + 2s^2 + 2st + t^2}/(1+s+t+u)^2,

 (与式) ={9(1-u) + (13-2u)s +(4+u)t +6s^2 +7st +3t^2}/(1+s+t+u)^2
   = 3 + {3(2-5u-u^2) +(7-8u)s -(2+5u)t +3s^2 +st}/(1+s+t+u)^2
   = 3 + (-12 -s -7t +3s^2 +st)/(1+s+t+u)^2   (← u=1)
   = 3 + {(5s/3 +4 +t)(s-3) +(4/3)(s^2 -3t)}/(1+s+t+u)^2
   ≧ 3,
等号成立は s=t=3 すなわち x=y=z=1 のとき。
664132人目の素数さん:2011/09/21(水) 19:33:44.04
For a,b,c>0, prove that
4(a^3+b^3+c^3-3abc)^3≧27(a^2b+b^2c+c^2a-3abc)^3
665132人目の素数さん:2011/09/22(木) 21:55:07.55
>>664

 a^3 +b^3 +c^3 -3abc ≧ 3k・(aab +bbc +cca -3abc),
 k = 1/{4^(1/3)} 〜 0.630
----------------------------------------------------
Let's put
 m = min{a, b, c}
 (a, b, c) = (m, m+x, m+x') or its rotation,
where x≧0, x'≧0.
Then,
 a^3 + b^3 + c^3 -3abc = (a+b+c){(a-c)^2 -(a-c)(b-c) +(b-c)^2}
    = (3m+x+x'){x^2 -xx' +(x')^2},
 aab +bbc +cca -3abc = m{x^2 -xx' +(x')^2} + (x^2)x',
and
 LHS - RHS ≧ (x+x'){x^2 -xx' +(x')^2} -3k(x^2)x'
   = (x + kx')(x - x'/√k)^2
   ≧ 0.
Equality holds for (m, m+x, m+x') = (0, 1, √k)
666132人目の素数さん:2011/09/22(木) 22:37:30.61
>>656
>>2の[1]のことか?
667132人目の素数さん:2011/09/22(木) 23:58:15.43
正の数a、b、c、dに対して
 2(ab + ac + ad + bc + bd + cd)^3 ≧ 27(abc + abd + acd + bcd)^2

( ゚∀゚)ウヒョッ!
668132人目の素数さん:2011/09/23(金) 00:28:08.23
a[k]>0 (1≦k≦n)
(x-a[1])(x-a[2])(x-a[3])…(x-a[n])=Σ[k=0,n] (-1)^(n-k)p[k]x^kとしたとき、
i<j⇒(p[i]/binomial(n,i))^(1/i)≧(p[j]/binomial(n,j))^(1/j) (等号成立はa[1]=a[2]=a[3]=…=a[n]のとき)
らしいのですが、どうやって証明するのが一番きれいですか?
コーシーシュワルツのようなきれいな証明を知りたいです。
669132人目の素数さん:2011/09/23(金) 00:54:07.39
>>668
S_k = p[k]/binomial(n,k) とおいて、(S_k)^2 ≧ S_(k-1)・S_(k+1) を示し、これを用いるのぢゃ
670132人目の素数さん:2011/09/23(金) 08:23:21.67
>>667
[初代スレ.455、473-474]


>>668
 Π[k=1,j-1] (S_k)^(2k) ≧ Π[k=1,j-1] {S_(k-1)・S_(k+1)}^k,
より
 (S_{j-1})^j ≧ S_0・(S_j)^(j-1),
 S_0 = 1,

[初代スレ.257, 263-271]
参考文献[1] Cambridge版 (1934) の 2.22節、公式51-55
E.F.Beckenbach - R.Bellman, "Inequalities", Ergebnisse叢書、Springer (1961) p.11


>>669
 Q_k = (S_k)^2 - S_(k-1)・S_(k+1) = {1/(n・k・C[n,k]・C[n-1,k])}Σ{j=0,k-1} [k;j]/(j+1) ≧ 0,
 ここに [k;j] は {a1・a2・・・・・a(k-j-1)}^2 a(k-j)・・・・a(k+j-1){a(k+j)-a(k+j-1)}^2 という型の積すべての和
ですね。
[初代スレ.480-481]
数セミ増刊「数学の問題 第1集」No.21 (1977.2)
671132人目の素数さん:2011/09/23(金) 09:52:01.92
>>670
   ___
 ./  ≧ \ グッジョブ!
 |::::  \ ./ | 初代スレ懐かしい…
 |::::: (● (● |  あれから7年も経ったのか…
 ヽ::::... .ワ....ノ    n
 ̄ ̄   \    ( E)
フ     /ヽ ヽ_//
672仙石60:2011/09/23(金) 09:56:22.55
じゃかあしい、黙ってろ!
673132人目の素数さん:2011/09/23(金) 10:15:54.71
>>672
誰だね君は?
674仙石60:2011/09/23(金) 10:18:00.02
俺はいまや 毎日が日曜日。
職業に関係する知識とノウハウは誰にも負けん。
675猫vs運営 ◆MuKUnGPXAY :2011/09/23(金) 13:50:55.86
ワシかていまや 毎日が日曜日。
馬鹿潰しに関係する知識とノウハウは誰にも負けん。


676132人目の素数さん:2011/09/23(金) 13:56:54.76
毎日が充実してて何よりですワ
677猫vs運営 ◆MuKUnGPXAY :2011/09/23(金) 15:53:56.74
そうですねん。飯も美味いし酒も美味いワ。


678132人目の素数さん:2011/09/23(金) 19:22:28.01
>>668-669
 (S_k)^2 ≧ S_(k-1)・S_(k+1),
より
 1/S_0 ≧ ・・・・・・ ≧ (S_k)^(k-1)/(S_{k-1})^k ≧ (S_{k+1})^k/(S_k)^(k+1) ≧ ・・・・,
 S_0 = 1,
とすべきか....
679132人目の素数さん:2011/09/24(土) 00:27:51.12
>>669>>670>>678
遅くなりましたがありがとうございます。
これは相加相乗平均の関係の拡張版と見なしていいですよね。
不等式の奥深さを改めて感じました。
n=6の場合についての問題が本に載っていたのですが、
皆目見当が付かず、答えが載っていなかったため数週間迷った挙句本屋に行っても
これについて解説している本が見つからなくて途方に暮れていました。
紹介していただいた参考文献[1]を是非読んでみたいと思います。
680132人目の素数さん:2011/09/24(土) 01:53:55.13
>>679
> n=6の場合についての問題が本に載っていたのですが、

その本の紹介きぼんぬ!ですぢゃ
681132人目の素数さん:2011/09/25(日) 09:38:07.22
使えんやっちゃな
682132人目の素数さん:2011/09/25(日) 18:40:04.63
>>680
シュプリンガーの『数学発想ゼミナール』3巻(第7章の題は「不等式」です)p.357

「x^6-6x^5+ax^4+bx^3+cx^2+dx+1=0の解は全て正であるという。
このときa,b,c,dを決定せよ。」

という問題です。
2次、3次方程式の解が全て実数かつ正であるための条件は
増減表などによって調べることが出来ました。
ここで上の命題を予想し、解が全て正である4次以上の方程式についても確かめたところ、正しそうだと分かりました。
相加-相乗平均の関係についての節の問題だったうえ、2項係数が出てきたため、
予想を導くこと自体はそれほど難しくありませんでした。
もし正しければa=15,b=-20,c=15,d=-6と定まり、x=1を6重解として持ちます。
しかし、証明がなかなか思いつかなかったので今回質問させていただきました。
683132人目の素数さん:2011/09/25(日) 19:55:12.30
>>682

根をα,β,γ,δ,ε,ζ とする。根と係数の関係より
 (α+β+γ+δ+ε+ζ)/6 = 1 = (αβγδεζ)^(1/6),
 相加平均 = 相乗平均,
また、題意より 根>0 だから 等号条件より
 α = β = γ = δ = ε = ζ = 1,
以下ry)
684132人目の素数さん:2011/09/26(月) 07:09:34.82
定理に辿りつけたのはご明察だが…
685132人目の素数さん:2011/09/27(火) 00:12:09.52
>>682

根を A,B,C,D,E,F >0 とするとき
 (A+B+C+D+E+F)^6 - (6^6)ABCDEF = Σ' {・・正の式・・・}(A-B)^2,
の形になることを示そう。
686685:2011/09/27(火) 00:18:25.59
(略証)
まづ、
 (A+B+C+D+E+F)^6 = (1/60)g + (1/2)h + (5/4)i + 5j1 + (5/6)j2 + 20k + 20L1 + 5L2 + 45m + 30n + 2o,
ここに
 g = 60(A^6 + B^6 + … + F^6) = 60[6],
 h = 12Σ (A^5)B = 12[5,1],
 i = 12Σ (A^4)(B^2) = 12[4,2],
 j1 = 6Σ(A^4)BC = 6[4,1,1],
 j2 = 24Σ (AB)^3 = 24[3,3],
 k = 3Σ (A^3)(B^2)C = 3[3,2,1],
 L1 = 6Σ (A^3)BCD = 6[3,1,1,1],
 L2 = 18Σ (ABC)^2 = 18[2,2,2],
 m = 4Σ (AB)^2・CD = 4[2,2,1,1],
 n = 12Σ (A^2)BCDE = 12[2,1,1,1,1,1],
 o = 360・ABCDEF,
である。

ここで、Muirhead により
 g - h = 12Σ (A^5 - B^5)(A-B) = 12Σ {A^4 +A^3・B + (AB)^2 +AB^3 +B^4}(A-B)^2,
 h - i = 12Σ AB(A^3 - B^3)(A-B) = 12Σ AB(A^2 +AB +B^2)(A-B)^2,
 i - j1 = 12Σ C^4・(A-B)^2,
 i - j2 = 12Σ (AB)^2・(A-B)^2,
 j1 - k = 3Σ (A^2)BC・(A-B)^2,
 j2 - k = 3Σ (B^3)C・(A-B)^2,
 k - L1 = 3Σ (C^3)D・(A-B)^2,
 k - L2 = 3Σ AB(C^2)・(A-B)^2,
 L1 - m = 2Σ ABCD・(A-B)^2,
 L2 - m = 2Σ (CD)^2・(A-B)^2,
 m - n = 4Σ (C^2)DE・(A-B)^2,
 n - o = 12Σ CDEF・(A-B)^2,
を使う。(終)
687685:2011/09/27(火) 00:58:03.82
補足
 Σ' はあらゆる文字の入替えに亘る和。(ただし同じものは1回ずつ)
 g = 60Σ' A^6,
688132人目の素数さん:2011/09/27(火) 01:51:31.96
>>687
顔文字に見えた
689132人目の素数さん:2011/09/28(水) 21:32:43.48
>>683
恥ずかしながら、上述の定理を予想していざ計算!
…という段階になってはじめてその解法に気付きました。
もっとも、遠回りの結果美しい不等式に出会えたので良かったのですが。

>>685-687
調べてみたところ、Muirhead's inequalityという名称があるのですね。
かなり複雑に見えますが、じっくり読ませていただきます。
690132人目の素数さん:2011/10/03(月) 22:24:05.40
x>0
⇒e*x^(ex)≧1
691132人目の素数さん:2011/10/07(金) 13:50:28.39
xln x≧-1/e (x>0)
692132人目の素数さん:2011/10/08(土) 02:12:36.05
x>0
⇒ 1/x = e/(ex) ≦ e^(1/(ex)),
⇒ -log(x) ≦ 1/(ex),
⇒ x・log(x) ≧ -1/e,
693132人目の素数さん:2011/10/08(土) 12:12:24.28
f : R→R
∀x、 y∈R に対して f(x+y) ≦ yf(x) + f(f(x)) が成立するとき、
∀x<0 に対して f(x)=0 を示せ
694132人目の素数さん:2011/10/08(土) 12:13:03.15
いちおう不等式がらみということで… ( ゚∀゚)
695132人目の素数さん:2011/10/10(月) 00:08:46.16
>>689

http://aozoragakuen.sakura.ne.jp/taiwa/taiwasen/egyptian/egyptian.html
http://messages.yahoo.co.jp/bbs?.mm=GN&action=m&board=1835554&tid=bdpbja1jiteybc0a1k&sid=1835554&start=46
 Yahoo!掲示板 - 科学 - 数学 - 出題(不等式)
http://planetmath.org/encyclopedia/MuirheadsInequality.html (英語)

示野信一:「対称式と不等式」数セミ、48巻、2号、通巻569 (2009/Feb) の p.26-29
G.H.Hardy、J.E.Littlewood & G.Polya: 「不等式」、シュプリンガー・フェアラーク東京 (2003/9) \5040 の 2.19節
696132人目の素数さん:2011/10/10(月) 00:22:46.59
697132人目の素数さん:2011/10/11(火) 08:37:52.47
>>693

IMO 2011 Problem 3
698132人目の素数さん:2011/10/19(水) 01:01:58.86
a,b,c>0→a^{b+c}+b^{c+a}+c^{a+b}≧1
699132人目の素数さん:2011/10/19(水) 11:52:19.94
x, y, z >0 (xyz=1)⇒ x^4+y^4+z^4+33≧12(xy+yz+zx)
700132人目の素数さん:2011/10/20(木) 11:41:10.61
>>699
わからん!
701132人目の素数さん:2011/10/21(金) 22:08:38.80
受験板より

f :R → Rは三回微分可能な関数で,全てのxについて次の条件@,Aが成り立っている
@f(x)>0,f'(x)>0,f''(x)>0,f'''(x)>0
Af'''(x)≦f(x)
このとき全てのxについて2f(x)>f'(x)が成立することを示せ
702132人目の素数さん:2011/10/22(土) 01:18:30.97
d/dx e^{2x}f(x)
703132人目の素数さん:2011/10/22(土) 08:08:04.99
そんなのは誰でも思いつくが、そこから先は?
704132人目の素数さん:2011/10/22(土) 08:33:46.75
>>703
俺は気づかなんだが、あとは推して知るべしだぞ!
705132人目の素数さん:2011/10/22(土) 16:57:35.04
d/dx e^{-2x}f(x) じゃないのけ?
706132人目の素数さん:2011/10/24(月) 16:01:25.22
>>701
これどうするん?
和歌んねーよ!
707132人目の素数さん:2011/10/24(月) 18:16:31.51
>>682
これスツルムの定理で解けない?
708132人目の素数さん:2011/10/26(水) 22:05:09.91
>>698

(1) a,b,c の中に1以上のものがあるときは明らか。

次に M = Max{b+c,c+a,a+b} とおく。

(2) a,b,c ≦ 1 かつ M ≦ 1 のとき
 b+c≦1, …, …
 y=x^(b+c) は xについて上に凸だから(x=1での)接線の下側にある。
 x^(b+c) ≦ 1 +(b+c)(x-1) ≦ 1 + (b+c)x,
 (1/x)^(b+c) ≧ 1/{1 + (b+c)x},  (ベルヌーイの式)
x=1/a とおいて
 a^(b+c) ≧ a/(a+b+c),
 循環的にたす。

(3) a,b,c ≦ 1 かつ M ≧ 1 のとき
 0 < a ≦ b,c ≦ 1 としても一般性を失わない。
 a+b, a+c ≦ b+c = M,
 (与式) ≧ b^(c+a) + c^(a+b)
   ≧ b^M + c^M
   ≧ 2・(M/2)^M   (← 下に凸)
   ≧ 2(1/2)    (← *)
   = 1,

*) {M・log(M/2)} ' = 1 + log(M/2),
∴ (M/2)^M は M>2/e  で単調増加。
∴ (M/2)^M ≧ 1/2,   (M≧1)

 casphy - 高校数学 - 不等式 - 710〜713
709132人目の素数さん:2011/10/31(月) 21:43:49.36
3辺の長さがa、b、cの三角形の外接円、内接円の半径をR、rとおくとき、a+b+c < 4(R+r)
( ゚∀゚) プケラッチョ!
710132人目の素数さん:2011/11/01(火) 11:57:24.96
a, b, c>0, √a+√b+√c=3⇒

a/√(a+b)+b/√(b+c)+c/√(c+a)≧3/√2
711132人目の素数さん:2011/11/01(火) 19:18:16.06
ウィキペの相加相乗平均の説明しょぼい・・・
712132人目の素数さん:2011/11/01(火) 23:21:22.64
>>709
例によって
 (a+b+c)/2 = s,
 (s-a)(s-b)+(s-b)(s-c)+(s-c)(s-a) = t,
 (s-a)(s-b)(s-c) = u,
とおく。
△不等式より、s-a>0, s-b>0, s-c>0,
ヘロンの公式: S = √{s(s-a)(s-b)(s-c)} = √(su),
 4R = abc/S = (st-u)/√(su),
 r = S/s = u/√(su)
よって
 {4R + (5/2)r}^2 = {st + (3/2)u}^2 /su
   = (2s)^2 + (s/tu)(t^3 -4stu +9u^2) + (3/t)(t^2 -3st) + (9u/4s)
   = (2s)^2 + (su/t)F_{-2}(s-a,s-b,s-c) + (3u/t)F_{-1}(s-a,s-b,s-c) + (9u/4s)
   > (2s)^2
   = (a+b+c)^2,

〔Schur不等式〕
 F_n(x,y,z) = x^n・(x-y)(x-z) + y^n・(y-z)(y-x) + z^n・(z-x)(z-y)
  = x^n・(x-y)^2 + (x^n -y^n +z^n)(x-y)(y-z) + z^n・(y-z)^2 ≧ 0.
 ここで y は x と z の中間にあるとした。(終)
713132人目の素数さん:2011/11/03(木) 20:20:36.93
>>712
> 例によって
>  (a+b+c)/2 = s,
>  (s-a)(s-b)+(s-b)(s-c)+(s-c)(s-a) = t,
>  (s-a)(s-b)(s-c) = u,

いや、この置き換え、初見なんだけど… (゚∀゚;)ブルブル
714β:2011/11/03(木) 20:24:14.57
常識やろw
715132人目の素数さん:2011/11/03(木) 23:12:58.69
>>713
 (a+b+c)/2 = s とおくと
 (s-a) + (s-b) + (s-c) = s,
 ……
なのですが、簡単なので省略しました。
716132人目の素数さん:2011/11/04(金) 00:11:08.55
nが自然数で、0 < x < π のとき、以下を証明せよ
(1) 1 + sin x + (sin 2x)/2 + … + (sin nx)/n > 0
(2) 1 + cos x + (cos 2x)/2 + … + (cos nx)/n > 0

( ゚A゚) ぐぬぬ…
717132人目の素数さん:2011/11/04(金) 04:03:13.97
>>716 (1)

nについての帰納法による。
与式の左辺を 1 + S_n(x) とおく。
 S_1(x) = sin(x) >0,
 S_2(x) = sin(x){1+cos(x)} > 0,
n>2 のとき
 (d/dx)S_n(x) = cos(x) + cos(2x) + …… + cos(n・x)
    = {sin((n+1/2)x) - sin(x/2)}/{2sin(x/2)} (積和公式)
    = sin(n・x/2)cos((n+1)x/2)/sin(x/2),   (和積公式)
S_n が極値をとる点 x=x0 に注目する。
 (1) sin(n・x0/2) = 0 のとき
   S_(x0) - S_{n-1}(x0) = sin(n・x0) = 0,
 (2) cos((n+1)x0 /2) = 0 のとき
   倍角公式より sin((n+1)x0) = 0, cos((n+1)x0) = -1,
   S_n(x) - S_{n-1}(x) = sin(n・x)
    = sin((n+1)x-x)
    = sin((n+1)x)cos(x) - cos((n+1)x)sin(x), (加法公式)
   S_n(x0) - S_{n-1}(x0) = sin(x0) > 0,

 参考文献[3] p.134-135, 例題8


〔類題〕
 S_n(x) = Σ[k=1,n] sin(kx)/k は sin(x) と同符号で、
  | S_n(x) | < G' = Si(π) = 1.8519370519824…,

 [第2章.50、53、55]
718132人目の素数さん:2011/11/04(金) 04:53:07.18
>>716 (2)

与式の左辺を C_n(x) とおく。
 C_1(x) = 1 + cos(x) > 0,
 C_2(x) = 1 + cos(x) + cos(2x)/2
   = 1/2 + cos(x) + cos(x)^2  (倍角公式)
   = 1/4 + {(1/2) + cos(x)}^2 > 1/4,
さて、
 (d/dx)C_n(x) = -sin(x) - sin(2x) + …… - sin(n・x)
   = {cos((n+1/2)x) - cos(x/2)}/{2sin(x/2)} (積和公式)
   = -(1/2)sin(n・x) - {1-cos(n・x)}cos(x/2)/{2sin(x/2)} (加法公式)
   < -(1/2)sin(n・x),
 C_n(π) - C_n(x) < -(1/2)∫[x,π] sin(n・t)dt
   = {(-1)^n - cos(n・x)}/(2n)
   < {(-1)^n + 1}/(2n)  (n:偶数のとき 1/n、n:奇数のとき 0)
   < (1-1) + (1/2 - 1/3) + …… + (-1)^n /n
   = C_n(π),
 ∴ C_n(x) > 0,  (森氏、ζ氏による.)

数セミ、34巻7号(1995/7)出題、34巻10号(1995/10)解説
719132人目の素数さん:2011/11/04(金) 09:35:22.58
( ゚∀゚) アナタ ガ 神 カ?
720132人目の素数さん:2011/11/04(金) 11:24:52.78
任意の z、w∈C に対して、| (1+|z|^2)w - (1+|w|^2)z | ≧ | z \bar{w} - \bar{z} w |

( ゚∀゚) プケラッチョ!
721132人目の素数さん:2011/11/04(金) 15:27:43.34
>>720

 (1+|z|^2)w - (1+|w|^2)z = (w-z) - zw(w-z)~,

 zw~ - z~w = (1/2){(w+z)(w-z)~ - (w+z)~・(w-z)},

∴ |左辺|^2 - |右辺|^2 = {(w-z) -zw(w-z)~}{(w-z)~ -z~w~(w-z)}
   - (1/4){(w+z)(w-z)~ - (w+z)~(w-z)}{(w+z)~(w-z) - (w+z)(w-z)~}
   = (w-z)(w-z)~(1 + |zw|^2 -zw~ -z~w)
   = (w-z)(w-z)~(1-zw~)(1-z~w)
   = |w-z|^2 |1-zw~|^2
   ≧ 0,
722132人目の素数さん:2011/11/04(金) 19:58:51.76
>>701を誰か...
723132人目の素数さん:2011/11/06(日) 22:11:14.99
>>699を誰か...
724132人目の素数さん:2011/11/06(日) 22:33:32.63
>>723
君が解き給へ(・∀・)!
725132人目の素数さん:2011/11/06(日) 23:47:08.21
生姜ねぇ...

>>699
いつものように
 f(x,y,z) = x^4 + y^4 + z^4 -12(xy+yz+zx) + 33
とくおく。

 f(x,y,z) - f(x, √(yz), √(yz)) = (y^2 -z^2)^2 -12x(√y -√z)^2
 = (√y -√z)^2 {(y+z)^2・(√y +√z)^2 - 12x}
 ≧ (√y -√z)^2 {(4yz)(4√yz) - 12x}
 = (√y -√z)^2 {16/(x^1.5) - 12x}   (← yz=1/x)

ところで f(x,y,z) は対称式だから x≦y,z としてもよい。
∴ x ≦ 1,
∴ 16/(x^1.5) - 12x > 0,
∴ f(x,y,z) ≧ f(x, 1/√x, √x),  (x≦1)

また
 x^2・f(x, 1/√x, 1/√x) = x^6 -24x^2.5 +33x^2 -12x +2
 = (√x - 1)^2 (x^5 +2x^4.5 +3x^4 +4x^3.5 +5x^3 +6x^2.5 +7x^2 -16x^1.5 -6x +4√x +2)
 = (√x - 1)^2 g(x)
 ≧ 0,

∵ g(x) ≧ g(0.4811730855931836) = 0.258670936041927

なお、x = 0.0394556281276082 に極大がある。(2.44552474861856)
726132人目の素数さん:2011/11/07(月) 00:08:48.59
>>725 訂正

真ん中より少し下
∴ f(x,y,z) ≧ f(x, 1/√x, 1/√x),  (x≦1)
727132人目の素数さん:2011/11/07(月) 07:42:17.98
a, b, c, d≧0, a+b+c+d=4 のとき,

a/(a^3+8)+b/(b^3+8)+c/(c^3+8)+d/(d^3+8)≦4/9
728132人目の素数さん:2011/11/08(火) 00:41:43.07
>>727

 (x^3 +8)(2x+1) - 27x = (x-1)^2・(2x^2 +5x+8) ≧ 0,
 x/(x^3 +8) ≦ (2x+1)/27,  … x=1 での接線
x = a,b,c,d についてたす。
729132人目の素数さん:2011/11/08(火) 02:04:44.31
>>727

相加・相乗平均より
 x^3 + 1 + 1 + 1 + 1 + 1 + 1 + 1 + 1 ≧ 9 x^(1/3),

∴ (左辺) ≦ (1/9){a^(2/3) + b^(2/3) + c^(2/3) + d^(2/3)}
   ≦ (4/9){(a+b+c+d)/4}^(2/3)   (上に凸)
   = 4/9,
730132人目の素数さん:2011/11/08(火) 16:04:19.53
a, b, cが実数のとき,
a^4+b^4+c^4+2abc(a+b+c)≧a^3b+b^3c+c^3a
731132人目の素数さん:2011/11/08(火) 16:57:38.99
微分→Jensen→AM-GMと解法が易しくなってきている。

Step 1 a^3≧3a-2

Step 2 AM-GM-HM
Done!

732132人目の素数さん:2011/11/09(水) 22:36:50.76
>>730

 (左辺) - (右辺) = a^4 +b^4 +c^4 + 2abc(a+b+c) -a^3・b -b^3・c -c^3・a
   = (1/2)(a^2 -b^2 -ab -ca)^2 + cyclic
   ≧ 0,

〔類題268〕
 (a^2 + b^2 + c^2)^2 ≧ 3(a^3・b + b^3・c + c^3・a),
>>268
>>284-290
733132人目の素数さん:2011/11/12(土) 12:11:39.84
a, b, c>0, a+b+c=1.

ab(c+2)/(c+1)+bc(a+2)/(a+1)+ca(b+2)/(b+1)≦7/12
734132人目の素数さん:2011/11/13(日) 02:00:45.58
>>733

 (a+b+c)^2 -3(ab+bc+ca) = (1/2){(a-b)^2 + (b-c)^2 + (c-a)^2} ≧ 0 より
 ab + bc + ca ≦ (1/3)(a+b+c)^2 = 1/3,
これを与式から差引くと、つまり次式を示せばよい。

 ab/(c+1) + bc/(a+1) + ca/(b+1) ≦ 1/4,

 (左辺) = ab{1 - c/(c+1)} + bc{1 - a/(a+1)} + ca{1 - b/(b+1)}
  = (ab+bc+ca) -abc{1/(c+1) + 1/(a+1) + 1/(b+1)}
  ≦ (ab+bc+ca) - 9abc/(a+b+c+3) (← 相加・調和平均 または y=1/x:下に凸)
  = (ab+bc+ca) - 9abc/{4(a+b+c)} (← a+b+c=1)
  = (1/4)(a+b+c)^2 - F_1(a,b,c)/{4(a+b+c)}
  ≦ 1/4,  (← a+b+c=1)

ここに
 F_1(a,b,c) = a(a-b)(a-c) + b(b-c)(b-a) + c(c-a)(c-b)
   = (a+b+c)^3 -4(a+b+c)(ab+bc+ca) + 9abc ≧ 0, (Schur)
735132人目の素数さん:2011/11/13(日) 04:20:40.33
キタコレ(・∀・)!
最初の3行に気づかなんだ
難しく見せているゴミを消すんだな
736132人目の素数さん:2011/11/17(木) 09:34:49.23
n次以下の整式 f(x) において、-1≦x≦1 における |f(x)| の最大値を M、
|f’(x)| の最大値 M’とおくとき、M’≦ n^2M が成り立つことを示せ
( ゚∀゚)プケラッチョ!
737132人目の素数さん:2011/11/17(木) 12:10:36.92
有名じゃね?
738132人目の素数さん:2011/11/17(木) 23:24:58.13
>>737
kwsk!
739132人目の素数さん:2011/11/18(金) 15:34:07.09
電波テロ装置の戦争(始)
エンジニアと参加願います公安はサリンオウム信者の子供を40歳まで社会から隔離している
オウム信者が地方で現在も潜伏している
それは新興宗教を配下としている公安の仕事だ
発案で盗聴器を開発したら霊魂が寄って呼ぶ来た
<電波憑依>
スピリチャル全否定なら江原三輪氏、高橋佳子大川隆法氏は、幻聴で強制入院矛盾する日本宗教と精神科
<コードレス盗聴>
2004既に国民20%被害250〜700台数中国工作員3〜7000万円2005ソウルコピー2010ソウルイン医者アカギ絡む<盗聴証拠>
今年5月に日本の警視庁防課は被害者SDカード15分を保持した有る国民に出せ!!<創価幹部>
キタオカ1962年東北生は二十代で2人の女性をレイプ殺害して入信した創価本尊はこれだけで潰せる<<<韓国工作員鸛<<<創価公明党 <テロ装置>>東芝部品)>>ヤクザ<宗教<同和<<公安<<魂複<<官憲>日本終Googl検索
740132人目の素数さん:2011/11/18(金) 15:34:50.73
魂は幾何学


誰か(アメリカ)気づいた
ソウルコピー機器
741132人目の素数さん:2011/11/18(金) 15:55:38.02
a, b, c, d>0, a^2+b^2+c^2+d^2=4.

(a+b+c+d-2)(1/a+1/b+1/c+1/d+1/2)≧9.
742132人目の素数さん:2011/11/19(土) 00:17:55.69
有名じゃね?
743β:2011/11/19(土) 00:20:06.15
おいおい、暗算で解けたしw
744132人目の素数さん:2011/11/19(土) 01:03:24.34
藤原先生は虚偽申請をやりました。藤原先生は虚偽申請をやりました。
藤原先生は虚偽申請をやりました。藤原先生は虚偽申請をやりました。
藤原先生は虚偽申請をやりました。藤原先生は虚偽申請をやりました。
藤原先生は虚偽申請をやりました。藤原先生は虚偽申請をやりました。
藤原先生は虚偽申請をやりました。藤原先生は虚偽申請をやりました。
藤原先生は虚偽申請をやりました。藤原先生は虚偽申請をやりました。
藤原先生は虚偽申請をやりました。藤原先生は虚偽申請をやりました。
藤原先生は虚偽申請をやりました。藤原先生は虚偽申請をやりました。
藤原先生は虚偽申請をやりました。藤原先生は虚偽申請をやりました。
藤原先生は虚偽申請をやりました。藤原先生は虚偽申請をやりました。
藤原先生は虚偽申請をやりました。藤原先生は虚偽申請をやりました。
藤原先生は虚偽申請をやりました。藤原先生は虚偽申請をやりました。
藤原先生は虚偽申請をやりました。藤原先生は虚偽申請をやりました。
藤原先生は虚偽申請をやりました。藤原先生は虚偽申請をやりました。
藤原先生は虚偽申請をやりました。藤原先生は虚偽申請をやりました。
藤原先生は虚偽申請をやりました。藤原先生は虚偽申請をやりました。
745猫 ◆MuKUnGPXAY :2011/11/19(土) 01:21:19.31

746132人目の素数さん:2011/11/19(土) 02:01:00.07
猫は学生の頃物理と化学もみっちりやりましたか?
747猫は痴漢野郎 ◆MuKUnGPXAY :2011/11/19(土) 02:04:41.16
馬鹿院生は虚偽院生となりました。馬鹿院生は虚偽院生となりました。
馬鹿院生は虚偽院生となりました。馬鹿院生は虚偽院生となりました。
馬鹿院生は虚偽院生となりました。馬鹿院生は虚偽院生となりました。
馬鹿院生は虚偽院生となりました。馬鹿院生は虚偽院生となりました。
馬鹿院生は虚偽院生となりました。馬鹿院生は虚偽院生となりました。
馬鹿院生は虚偽院生となりました。馬鹿院生は虚偽院生となりました。
馬鹿院生は虚偽院生となりました。馬鹿院生は虚偽院生となりました。
馬鹿院生は虚偽院生となりました。馬鹿院生は虚偽院生となりました。
馬鹿院生は虚偽院生となりました。馬鹿院生は虚偽院生となりました。
馬鹿院生は虚偽院生となりました。馬鹿院生は虚偽院生となりました。
馬鹿院生は虚偽院生となりました。馬鹿院生は虚偽院生となりました。
馬鹿院生は虚偽院生となりました。馬鹿院生は虚偽院生となりました。
馬鹿院生は虚偽院生となりました。馬鹿院生は虚偽院生となりました。
馬鹿院生は虚偽院生となりました。馬鹿院生は虚偽院生となりました。
馬鹿院生は虚偽院生となりました。馬鹿院生は虚偽院生となりました。
馬鹿院生は虚偽院生となりました。馬鹿院生は虚偽院生となりました。
馬鹿院生は虚偽院生となりました。馬鹿院生は虚偽院生となりました。
馬鹿院生は虚偽院生となりました。馬鹿院生は虚偽院生となりました。
馬鹿院生は虚偽院生となりました。馬鹿院生は虚偽院生となりました。


748猫は痴漢野郎 ◆MuKUnGPXAY :2011/11/19(土) 02:08:03.06
>>746
私は数学科ではなくて応用物理みたいな学科の学卒なので、従って物理
は仕方無く勉強しました。そんで化学は必修科目として結構含まれてい
ましたが、全然好きにはなれませんでした。とにかく実験の演習は苦痛
でしかアリマセンでしたね。

だから物理も化学もみっちりでも何でもありません。最低限ですね。


749132人目の素数さん:2011/11/19(土) 11:01:43.28
741はむずいぞ、これ。
750132人目の素数さん:2011/11/19(土) 17:57:36.10
相加相乗平均使うだけだろ?
751132人目の素数さん:2011/11/19(土) 20:14:46.04
あほと, ちゃうか?
どうやって, 相加相乗使えるねん?!
752132人目の素数さん:2011/11/19(土) 22:17:02.33
a+b+c+d≧4(abcd)^(1/4)
1/a+1/b+1/c+1/d≧4(1/abcd)^(1/4)
753132人目の素数さん:2011/11/19(土) 22:59:59.78
>>752
それでは証明できない。
754132人目の素数さん:2011/11/20(日) 12:57:28.57
2ちゃんの数学板の中でもここだけは本物の鬼修羅羅刹が生息する場所だなって思うわ。
自作の不等式問題投げたときも30分で解かれたし。
755Y:2011/11/20(日) 18:16:34.40
数学版楽しい。学校よりも楽しい
756132人目の素数さん:2011/11/20(日) 19:29:31.45
>>701が未だに分からん。

↓こうやって、微分を1つ減らした問題なら解けるんだけどなあ。

f :R → Rは二回微分可能な関数で,全てのxについて次の条件@,Aが成り立っている
@f(x)>0,f'(x)>0,f''(x)>0
Af''(x)≦f(x)
このとき全てのxについて2f(x)>f'(x)が成立することを示せ
757132人目の素数さん:2011/11/20(日) 22:21:04.68
>>756
kwsk!
758132人目の素数さん:2011/11/21(月) 00:28:55.43
>>757

 f(x) は下に有界かつ単調増加だから、lim[x→ -∞] f(x) は収束する。
 f(-∞) = lim[x→ -∞] f(x) = a ≧ 0.
 f '(-∞) = lim[x→ -∞] f '(x) = 0.

f '(x) >0 と (2)から
 (d/dx){f(x)^2 - f '(x)^2} = 2f '(x){f(x) - f "(x)} ≧ 0,
∴ f(x)^2 - f '(x)^2 は単調増加。
∴ f(x)^2 - f '(x)^2 ≧ f(-∞)^2 - f '(-∞)^2 = a^2 - 0^2 ≧ 0,
f(x) + f '(x) >0 で割れば
 f(x) - f '(x) ≧ 0,
759132人目の素数さん:2011/11/22(火) 06:33:29.44
>>758
 (d/dx){f(x)exp(-x)} ≦ 0,
∴ f(x) は単調増加だが、f(x)exp(-x) は(広義)単調減少。
760132人目の素数さん:2011/11/22(火) 07:26:54.80
>>730 >>732

 p' = a^2 -b^2 +bc,
 q' = b^2 -c^2 +ca,
 r' = c^2 -a^2 +ab,
とおくと、
 (左辺) - (右辺) = (1/2)(p'+q')^2 + (1/2)(q'+r')^2 + (1/2)(r'+p')^2 ≧ 0

 casphy - 高校数学 - 不等式、718
761132人目の素数さん:2011/11/22(火) 13:57:08.76
Nice Solution!
Exactly same as mine.
762132人目の素数さん:2011/11/22(火) 21:17:03.30
日本語でおk
763132人目の素数さん:2011/11/24(木) 22:29:54.05
〔問題〕
a,b,c が実数で =(a-b)(b-c)(c-a) のとき、

(1)
納n=1,2] {a^(2n) +b^(2n) +c^(2n) -(ab)^n -(bc)^n -(ca)^n}
  ≧ (3/2)|處,

(2)
納n=1,4] {a^(2n) +b^(2n) +c^(2n) -(ab)^n -(bc)^n -(ca)^n}
  ≧ 3(1 + a+b+c +a^2 +b^2 +c^2)|處,

を、示して下さい。

 casphy - 高校数学 - 不等式、719,722,725
764132人目の素数さん:2011/11/26(土) 07:13:35.37
765132人目の素数さん:2011/11/26(土) 09:46:58.28
>>764


1.Let a be an arbitrary real number and b a positive real number. Evaluate the integral
  ∫[0,∞) cos(ax)/cosh(bx) dx.
  (Recall that cosh(x) = (1/2)(e^x + e^-x) is the hyperbolic cosine.)
  {Wed., 2010/Jan/20 (Day 2)}


2.Let f be a holomorphic function on a domain containing the closed disc {z : |z|≦3}, and suppose that
  f(1) = f(i) = f(-1) = f(-i) = 0.
Show that
  |f(0)| ≦ (1/80)・max{|f(z)| : |z|=3},
and find all such functions for which equality holds in this inequality.
  {Thu., 2010/Jan/21 (Day 3)}
766132人目の素数さん:2011/11/26(土) 10:33:54.24
>>765 (1)

 cos(ax) = (1/2){e^(iax) + e^(-iax)} から、
 ∫[0,∞) cos(ax)e^(-cx) dx
  = (1/2)∫[0,∞) {e^(-(c-ia)x) + e^(-(c+ia)x)} dx
  = (1/2){1/(c-ia) + 1/(c+ia)}
  = c/(a^2+c^2),  (c>0)

 1/cosh(bx) = 2e^(-bx)/{1+e^(-2bx)} = 2Σ[k=0,∞) e^(-(2k+1)bx),
を使っても出せぬぅ.....

答: π/{2b・cosh(πa/2b)},

〔参考書〕
森口・宇田川・一松:「数学公式I」岩波全書221, p.256 (1956)
767132人目の素数さん:2011/11/26(土) 11:48:54.54
>>765 訂正

 1/cosh(bx) = 2e^(-bx)/{1+e^(-2bx)} = 2Σ[k=0,∞) (-1)^k・e^(-(2k+1)bx),
768132人目の素数さん:2011/11/26(土) 20:36:13.48
証文の出し遅れのような気がしますが、> 570 の解答については
http://www.emis.de/journals/JIPAM/images/105_09_JIPAM/105_09.pdf
に目を通しておいて下さい。ついでに、
http://www.math.s.chiba-u.ac.jp/~ando/ineq.pdf
も読んで頂けると幸いです。
769132人目の素数さん:2011/11/26(土) 20:43:16.45
ついでにこういう定理をご存知ですか。
定理1. 4次斉次多項式f(a,b,c)について、
任意の実数a,b,cに対しf(a,b,c)≧0が成り立つための必要十分条件は、
f(1,0,0)≧0かつf(x,1,1)≧0 (∀x∈R)である。

定理2. 3〜5次斉次多項式f(a,b,c)について、
任意のa,b,c≧0に対しf(a,b,c)≧0が成り立つための必要十分条件は、
f(x,1,0)≧0かつf(x,1,1)≧0 (∀x≧0)である。
770132人目の素数さん:2011/11/26(土) 20:45:04.43
すいません。直前の訂正です。

定理1. 4次斉次対称多項式f(a,b,c)について、
任意の実数a,b,cに対しf(a,b,c)≧0が成り立つための必要十分条件は、
f(1,0,0)≧0かつf(x,1,1)≧0 (∀x∈R)である。

定理2. 3〜5次斉次対称多項式f(a,b,c)について、
任意のa,b,c≧0に対しf(a,b,c)≧0が成り立つための必要十分条件は、
f(x,1,0)≧0かつf(x,1,1)≧0 (∀x≧0)である。
771132人目の素数さん:2011/11/26(土) 21:30:54.77
>>765
b> 0
Pi/2b sech(a Pi/2b)
772132人目の素数さん:2011/11/26(土) 22:13:07.75
>>768

>>570
 N→w, I→-p, J→-q, K→r, L→(p+q-r-w)
とおけば (1.6) になりますね。
もっとも、これらの文献では w=1 としているようですが....
773132人目の素数さん:2011/11/27(日) 22:11:38.11
>>769-770

基本対称式を x+y+z=s, xy+yz+zx=t, xyz=u とおくと
 f(x,y,z) = f(1,0,0)(s^4 -6sst +8tt +3su) + f(0,1,1)(8tt-sst-15su)/4 + f(1,1,1)(8tt-2sst-5su)/3 + f(2,1,1)(sst-4tt+3su)/4
と書けるが…
774132人目の素数さん:2011/11/30(水) 13:27:41.48
775132人目の素数さん:2011/11/30(水) 23:51:09.00
>>774

(64_1)
(1) sgn(a),   x = |a|・tanθ とおく。
(2) 部分分数に分けて
 f(x)f(t-x) = (1/2π)f(t/2){(1/2 + x/t)f(x) + (1/2 + (t-x)/t)f(t-x)}
 xf(x) は奇関数だから、積分すれば0.
 (t-x)f(t-x) も同様。
 ∴ (1/2π)f(t/2)∫(-∞,∞) {f(x) + f(t-x)}/2 dx = (1/2π)f(t/2),

66-5
問題1.
 (左辺) - (右辺) = (4/5)(x-y)^2 + (4/5)(x+y)(z-x-y) + (z-x-y)^2 ≧0,
 z = x+y+Z (Z≧0) を与式に代入する。

問題2.
 (与式) > ∫[0,1] (x^2)e^(-x) dx
   = [ -(x^2 +2x +2)e^(-x) ](x=0,1)
   = 2 - (5/e) = 0.160603

 (与式) < ∫[0,1] (x^2)・e^(-x^3) dx
   = [ -(1/3)e^(-x^3) ](x=0,1)
   = (1/3)(1 - 1/e) = 0.210707
(真値は (1/4)(√π)erf(1) - 1/(2e) = 0.189472345820492...)

67-2
(1) f(x) = (x+1/x)^2 は下に凸だから
 (a + 1/a)^2 + (b + 1/b)^2 + (c + 1/c)^2
  = f(a) + f(b) + f(c)
  ≧ 3f((a+b+c)/3)   (← 下に凸)
  = 3f(1/3) = 3(10/3)^2 = 100/3,
776132人目の素数さん:2011/12/02(金) 01:41:46.25
>>763・・・
777132人目の素数さん:2011/12/02(金) 01:43:06.65
67-3

m は {m(m-1)+2}/2 項目に初めて現れれる。これをnとすると、mは
 a_n = [ (1 + √(8n-7))/2 ]

>>774

66-5
 I_n = ∫[0,1] x^2・exp(-x^n) dx
は nについて単調増加で、 1/3 に収束する。

 I_n ≒ 1/3 - 1/(1.2553312n + 4.22642)  (n>>1)
778132人目の素数さん:2011/12/02(金) 14:24:30.43
>>775
67-2 (1)

〔補題〕
ある区間で f(x) >0 とする。
(1) f が下に凸, a>1  ⇒ f^a も下に凸。
(2) f が上に凸, 0<a<1 ⇒ f^a も上に凸。
(3) f が上に凸, a<0  ⇒ f^a は下に凸。

(略証)
 f が上に凸 ⇔ f " >0,
 f が下に凸 ⇔ f " <0,

 f(x)^a = g(x) とおくと
 g ' = a・f^(a-1)f ',
 g " = a(a-1)f^(a-2){f '}^2 + a・f^(a-1)・f ",
779132人目の素数さん:2011/12/02(金) 14:51:44.63
>>778 の訂正

(略証)
 f が下に凸 ⇔ f " >0,
 f が上に凸 ⇔ f " <0,

780132人目の素数さん:2011/12/02(金) 21:23:27.67
>>763 (1) >>776

・三角不等式 |b+c| + |a+b| ≧ |c-a| を使って

(左辺) = {a^2 +b^2 +c^2 -ab-bc-ca} + {a^4 +b^4 +c^4 -(ab)^2 -(bc)^2 -(ca)^2}
  = (1/2){(a-b)^2 +(b-c)^2 +(c-a)^2} + (1/2){(a^2 -b^2)^2 +(b^2 -c^2)^2 +(c^2 -a^2)^2}
  = (1/4){(a-b)^2 +(b^2 -c^2)^2} + (1/4){(b-c)^2 +(a^2 -b^2)^2} + cyclic
  ≧(1/2){|a-b||b^2 -c^2| + |b-c||a^2 -b^2|} + cyclic (← 相加・相乗平均)
  = (1/2)|a-b||b-c|(|b+c|+|a+b|) + cyclic
  ≧(1/2)|a-b||b-c||c-a| + cyclic  (← △不等式)
  = (3/2)|處,

・あるいは
  = (a-b)(b-c)(c-a) = (1/3){(c-a)+(c-b)}(a^2 -b^2) + cyclic,
から
 (左辺) - (3/2)|處 = (1/4)(c-a)^2 + (1/4)(c-b)^2 + (1/2)(a^2 -b^2)^2 ±(1/2){(c-a)+(c-b)}(a^2 -b^2) + cyclic
  = (1/4){(c-a)±(a^2 -b^2)}^2 + (1/4){(c-b)±(a^2 -b^2)}^2 + cyclic (複号同順)
  = (1/8){(2c-a-b)±2(a^2 -b^2)}^2 + (1/8)(a-b)^2 + cyclic
  ≧ 0,
781Y:2011/12/07(水) 18:11:04.99
すげえ
782132人目の素数さん:2011/12/08(木) 23:37:47.56
>>701は誰も解けないの?

783132人目の素数さん:2011/12/09(金) 00:09:56.40
ヒントやろうか?
784132人目の素数さん:2011/12/09(金) 00:50:16.48
いやいらない
785132人目の素数さん:2011/12/09(金) 06:09:30.37
以下の真ん中あたり
http://www.sugakukobo.com/

問題
http://www.sugakukobo.com/pdf/SuuSemi_1.pdf

解説
http://www.sugakukobo.com/pdf/SuuSemi_2.pdf

( ゚∀゚)プケラッチョ!
786132人目の素数さん:2011/12/09(金) 06:12:53.86
>>783
さっさとよこせ!でございます
787132人目の素数さん:2011/12/09(金) 14:19:18.49
Putnam Competition, 1999 B-4
788132人目の素数さん:2011/12/09(金) 23:36:44.27
獲得金メダル! 国際数学オリンピック第1章「不等式」,小林一章,朝倉書店,2011年
   http://www.asakura.co.jp/books/isbn/978-4-254-1113...

嫁! ( ゚∀゚)プケラッチョ!
789132人目の素数さん:2011/12/10(土) 05:07:17.81
x、y、z≧0に対して、
x^4・y + y^4・z + z^4・x ≧ x^2・y^2・z + y^2・z^2・x + z^2・x^2・y

( ゚∀゚)プケラッチョ!
790132人目の素数さん:2011/12/10(土) 17:07:44.95
>>788
http://www.asakura.co.jp/books/isbn/978-4-254-11132-3/
A5/192ページ/2011年11月25日
定価2730円
数学オリンピック(JMO・IMO)出場者自身による,類例のない数学オリンピック問題の解説書。
単なる「問題と解答」にとどまらず,知っておきたい知識や実際の試験での考え方,答案の組み立て方などにも踏み込んで高い実践力を養成する。


>>789
相加・相乗平均より
 (6x^4・y + 5y^4・z + 2z^4・x)/(6+5+2) ≧ x^2・y^2・z,
循環的にたす。

( ゚∀゚)プケラッチョ!
791132人目の素数さん:2011/12/10(土) 17:31:36.86
>>788
> 獲得金メダル! 国際数学オリンピック第1章「不等式」,小林一章,朝倉書店,2011年
>    http://www.asakura.co.jp/books/isbn/978-4-254-11132-3/

■Muirheadの不等式■
x、y、z >0 とする
 p_1 ≧ q_1
 p_1 + p_2 ≧ q_1 + q_2
 p_1 + p_2 + p_3 = q_1 + q_2 + q_3
のとき、(i、j、k) は (1、2、3) の並び替えとして
 Σx^(p_i)・y^(p_j)・z^(p_k) ≧ Σx^(q_i)・y^(q_j)・z^(q_k)

(P.10より)
Muirheadの不等式を用いて不等式を照明することを Bunching といいます
(日本選手の間でも2003年頃から普及しはじめ、「バンチ」と呼ぶ人が多いです)

    ___   なんで Bunching なのか小一時間問い詰めたい
  ./  ≧ \  ああ問い詰めたいね
  |::::  \ ./ |   別にムッハァ-でもいいじゃんかと!
  |::::: (● (● |
  ヽ::::... .∀....ノ /  チン ☆
 _(  ⊃  ⊃  チン ☆
 |\ ̄ ̄ ̄ ̄旦 ̄\
 | | ̄ ̄ ̄ ̄ ̄ ̄ ̄|
 \|  愛媛みかん |
    ̄ ̄ ̄ ̄ ̄ ̄ ̄
792132人目の素数さん:2011/12/10(土) 19:40:28.18
>>791

Bunchin さんは63歳になられました....

http://ja.wikipedia.org/wiki/%E6%A1%82%E6%96%87%E7%8F%8D
793132人目の素数さん:2011/12/10(土) 22:21:58.20
>>792

「ねこやなぎ」の由来は?
http://www.youtube.com/watch?v=PYHozLh3QyA
794132人目の素数さん:2011/12/11(日) 06:20:05.34
Bunchin師匠の独演会

http://www.youtube.com/watch?v=aW5DJHMrrcA
795132人目の素数さん:2011/12/11(日) 07:08:35.73
( ゚∀゚) 荒らすなYO!
796132人目の素数さん:2011/12/11(日) 21:06:29.75
>>791

 p_1 ≧ p_2 ≧ p_3,
 q_1 ≧ q_2 ≧ q_3,
とするんでつか? 

 (p)ゝ(q)
と書き、pはqの優数列である(p majorizes q)という。

参考文献[3] p.125 (1987.10)
797132人目の素数さん:2011/12/12(月) 17:25:05.25
a, b, c>0 with abc=1.
For f(a, b, c)=a+b^{20}+c^{11},

f(a, b, c)+f(b, c, a)+f(c, a, b)≦1
798132人目の素数さん:2011/12/12(月) 21:23:27.28
>>782
問題自体に不備がある悪寒
799132人目の素数さん:2011/12/12(月) 21:51:50.20
反例を探したほうがいいかもな
800132人目の素数さん:2011/12/12(月) 23:44:50.54
>>789

両辺を xyz で割ると
 x^3 /z + y^3 /x + z^3 /y ≧ xy + yz + zx,
となる。これはコーシー
 (x^3 /z + y^3 /x + z^3 /y)(xz+yx+zy) ≧ (x^2 + y^2 + z^2)^2 ≧ (xy+yz+zx)^2,
より明らか。(冬)

 casphy - 高校数学 - 不等式 - 735
801132人目の素数さん:2011/12/13(火) 09:19:04.81
>>789
チェビシェフによる。

 Σ(乱順序積) ≧ Σ(逆順序積) から
 x^3 /z + y^3 /x + z^3 /y ≧ x^3 /x + y^3 /y + z^3 /z
   = x^2 + y^2 + z^2 ≧ xy + yz + zx,
または
 xy=Z, yz=X, zx=Y とおいて
 Σ(同順序積) ≧ Σ(乱順序積) より
 x^3 /z + y^3 /x + z^3 /y = YZZ/XX + ZXX/YY + XYY/ZZ
  ≧ YZZ/ZZ + ZXX/XX + XYY/YY = Y + Z + X = zx + xy + yz,
802132人目の素数さん:2011/12/15(木) 15:04:08.16
モローの不等式age!

  _  ∩
( ゚∀゚)彡 モロー! モロー!
 ⊂彡
803132人目の素数さん:2011/12/15(木) 15:12:21.25
           おっぱい!
       おっぱい! おっぱい!
    おっぱい おっぱい! おっぱい!
  おっぱい! ∩   ∩ ノ)   おっぱい!
 おっぱい!  川 ∩ 川彡'三つ  おっぱい!
おっぱい! ⊂ミ∩、⊂ミ∩彡⊃    おっぱい!
おっぱい!⊂三ミ( ゚∀゚)彡三彡三⊃ おっぱい!
おっぱい! ⊂彡川⊂彡川ミ⊃    おっぱい!
おっぱい!⊂彡川∪⊃ U川彡⊃   おっぱい!
 おっぱい! (ノ ∪  川 ∪ミ)  おっぱい!
  おっぱい!      ∪     おっぱい!
    おっぱい! おっぱい! おっぱい!
        おっぱい! おっぱい!
            おっぱい!
804132人目の素数さん:2011/12/15(木) 21:42:40.44
a_n = (1 + 1/n)^n
b_n = (1 + 1/n)^(n+1)

e/(2n+2) < e - a_n < e/(2n+1) < b_n - e < e/(2n)

  _  ∩
( ゚∀゚)彡 モロー! モロー!
 ⊂彡
805132人目の素数さん:2011/12/15(木) 23:57:48.12
数検スレより

676 返信:132人目の素数さん[] 投稿日:2011/12/15(木) 22:38:11.63
>>675

要は
a_n=(1 +1/n)^n
b_n=(1 +1/n)^(n+1)
から、
2=a_1<a_2<…<a_n<…<b_n<…<b_2<b_1=4
の相加相乗を使った証明。

数検1級(H23/4)の問題
『(1 +1/n)^(n +1/2) > e を証明せよ』
とテイラー展開を使った模範解答。

別解として、積分(中点公式)を使った解法

さらに、出題として、
 台形公式を適用した場合、eに対するどのような関係式となるか

発展課題として、
モローの不等式の証明
『e/(2n+2)<e - a_n < e/(2n+1) < b_n - e < e/(2n)』

  _  ∩
( ゚∀゚)彡 モロー! モロー!
 ⊂彡
806132人目の素数さん:2011/12/16(金) 09:22:44.68
797>>Sorry, the correct versio is here.

a, b, c>0 with abc=1.

For f(a, b, c)=a+b^{20}+c^{11},

1/f(a, b, c)+1/f(b, c, a)+1/f(c, a, b)≦1.
807132人目の素数さん:2011/12/16(金) 14:16:29.16
>>701 はPutnam Competitionの1999年の問題の条件の一部が抜け落ちたもの
元の問題ではfはC^3級になってる
808132人目の素数さん:2011/12/16(金) 15:47:23.94
>>807
模範解答はないのですか?
809132人目の素数さん:2011/12/16(金) 18:57:34.40
聞く前に探せ!
810132人目の素数さん:2011/12/16(金) 22:38:36.36
811132人目の素数さん:2011/12/17(土) 01:07:55.84
>>805
 (1 +1/n)^(n +1/2) > e も二項展開でOK

http://kamome.2ch.net/test/read.cgi/math/1295154182/68-69

812132人目の素数さん:2011/12/17(土) 17:02:28.96
>>805
うちの田舎町では売っていないんだけど、相加相乗の証明を教えてちょ
813132人目の素数さん:2011/12/17(土) 17:14:52.36
>>812
a[n-1]/a[n]
=(n^2/(n^2-1))^(n-1)*n/(n+1)
<(((n-1)*(n^2/(n^2-1))+n/(n+1))/n)^n
=1
∴a[n-1]<a[n]
b[n]も同様
814132人目の素数さん:2011/12/17(土) 21:11:12.61
にゃるほど、さんくす
815132人目の素数さん:2011/12/18(日) 03:57:07.52
>>813
=(n^2/(n^2-1))^(n-1)*n/(n+1)
<(((n-1)*(n^2/(n^2-1))+n/(n+1))/n)^n

これがよくわかりません
816132人目の素数さん:2011/12/18(日) 10:35:34.65
>>815
相加相乗
817132人目の素数さん:2011/12/18(日) 10:46:04.97
そうか!そうじょうか!
818132人目の素数さん:2011/12/18(日) 20:53:38.40
>>812 >>815

{n/(n-1), n/(n-1), ……, n/(n-1), 1}
     (n-1)個         1個
の相乗・相加平均で
 {n/(n-1)}^(n-1) < {(n+1)/n}^n,
∴ a[n-1] < a[n],
819132人目の素数さん:2011/12/19(月) 06:39:23.97
ちょうど1になるのか。相加相乗を使ってくださいといわんばかりだな。
820132人目の素数さん:2011/12/19(月) 11:54:03.82
>>805
> さらに、出題として、
>  台形公式を適用した場合、eに対するどのような関係式となるか

∫[a,b] f(x)dx < (b-a)(f(a)+f(b))/2 からeに関する何が得られるか謎でござるよ、ニンニン
821132人目の素数さん:2011/12/20(火) 15:36:42.61
>>2の[4]を本屋で見かけたけどページ数の割に高かったから買うのを躊躇してしまった・・・
こういう感じの基本的な不等式をしっかりと扱った本って他にある?
洋書でもいいんで教えてください
822132人目の素数さん:2011/12/20(火) 20:17:44.59
>>821
本題をケチるなど言語道断!
823132人目の素数さん:2011/12/20(火) 20:18:35.64
Problem372にハァハァ…
http://www.math.ust.hk/excalibur/v16_n2.pdf

( ゚∀゚)プケラッチョ!
824132人目の素数さん:2011/12/20(火) 20:46:00.62
eに関する不等式が出てきた今なら出せる

( ゚∀゚)つ lim[n→∞] { (n+1)^(n+1) / n^n - n^n / (n-1)^(n-1) } =
825132人目の素数さん:2011/12/20(火) 22:34:11.62
>>823

Problem 2.
 Given real numbers x,y,z such that x+y+z=0, show that
 x(x+2)/(2x^2 +1) + y(y+1)/(2y^2 +1) + z(z+2)/(2z^2 +1) ≧ 0,
 When does equality hold ?

Problem 372.
For all a,b,c>0 and abc=1, prove that
 1/{a(a+1)+ab(ab+1)} + 1/{b(b+1)+bc(bc+1)} + 1/{c(c+1)+ca(ca+1)} ≧ 3/4.
826132人目の素数さん:2011/12/20(火) 22:55:55.48
Problem 5

YOSHIO > TETSUYA
827猫は共著のみ ◆MuKUnGPXAY :2011/12/20(火) 22:57:00.38
You need a proof.

--neko--
828132人目の素数さん:2011/12/20(火) 22:58:03.53
前科を比べたら哲也の方が上や
829132人目の素数さん:2011/12/21(水) 00:33:42.48
そうか?
830132人目の素数さん:2011/12/21(水) 03:02:29.47
>>825

Problem 2.
通分して
 (左辺)・(2x^2 +1)(2y^2 +1)(2z^2 +1)
  = (2xyz +2xy +x+y)^2 + (2xyz +2yz +y+z)^2 + (2xyz +2zx +z+x)^2 -(x+y+z)(8xyz+x+y+z-2)
  = (2xyz +2xy +x+y)^2 + (2xyz +2yz +y+z)^2 + (2xyz +2zx +z+x)^2  (← 題意)
  ≧ 0,
 等号成立は (0,0,0) (-1/2,-1/2,1) etc. のとき。
831132人目の素数さん:2011/12/21(水) 03:58:42.26
>>824
通分して、(1 + 1/(n-1))^(n-1)を括り出したまではいいが、その後が進マンボー

                  /^i
             /:::::|
          __/::::::::|
        ,. ‐' ´::::::::::::::::::::::::::ヽ:.、
   , ‐'´:::::::::::::::::::::::::::::::::::::::::::ヽ:\
.  (:::(o):::::::/i:::::::::::::::::::::::::::::::::i::::::i
   ヽ     ̄ ::::::::::::::::::::::::::::::|::::::l 進マンボー
    \        ::::::::::::::::::i::::::i
     `‐ 、        ::::::/::/
          ` ー-- 、.......::/ '´
               i:::::::|
                  i:::::::!
              ヽ:_|
832132人目の素数さん:2011/12/21(水) 05:37:53.01
かわゆす
833猫の育て方 ◆MuKUnGPXAY :2011/12/21(水) 10:04:54.39
その魚は実は寿司ネタとして喰えるのや。


834132人目の素数さん:2011/12/21(水) 11:50:38.85
A(4)は
|2q 1|
|1 3q|
じゃないかと想像がつくやろ
ほしたらA(5)=4q A(4)-2q がどうなるかはピンと来てもおかしくないやろ
それにA(n)はちゃあんとn-2次の行列式で表せるわい
835132人目の素数さん:2011/12/21(水) 11:52:23.12
あかん誤爆したわ
836132人目の素数さん:2011/12/21(水) 15:26:06.45
>>387
> (3) (a^2 +2)(b^2 +2)(c^2 +2)
>  を 3つの対称式の平方和で表わせ。

>>389
> (a^2 + p^2)(b^2 + q^2)(c^2 +r^2) = (abc-aqr-pbr-pqc)^2 + (pbc+aqc+abr-pqr)^2,
> だと2つになるし・・・・・

>>390
>  p=q=r=√2 を入れて
>  {abc-2(a+b+c)}^2 + (bc+ca+ab-2)^2 + (bc+ca+ab-2)^2,

390が分かりませぬ ('A`)
p=q=r=√2 を入れたら、 {abc-2(a+b+c)}^2 + (bc+ca+ab-2√2)^2 で止まって進マンボー!


837132人目の素数さん:2011/12/21(水) 15:28:32.65
ごめん、分かった
838132人目の素数さん:2011/12/21(水) 23:19:52.06
>>804-805

 {2n/(2n+1)}e < a_n < {(2n+1)/(2n+2)}e,
 {(2n+2)/(2n+1)}e < b_n < {(2n+1)/2n}e,
は同値

左側
 >>811 と相加相乗平均より
 {2√(n(n+1))/(2n+1)}e < e < g_n,
右側
 (1 - 1/k^2)^(k+1) > 1 -(k+1)/k^2 = (k^2 -k-1)/k^2, (下に凸)
 a[k]/a[k-1] = (k+1)^k・(k-1)^(k-1)/k^(2k-1) > {2k/(2k-1)}・{(2k+1)/(2k+2)},
k = n+1〜∞ について掛けて
 e / a[n] > (2n+2)/(2n+1),
 a[n] < {(2n+1)/(2n+2)}e,
839132人目の素数さん:2011/12/22(木) 00:35:36.85
>>804-805
 g_n = √(a_n・b_n) とおくと >>838 より

 e < g_n < {(2n+1)/2√(n(n+1))}e < {1 + 1/(8n^2)}e,

>>824

 (与式) = √(n(n+1))・g_n - √((n-1)n)・g_(n-1)
    = {√(n(n+1)) - √((n-1)n)}・e + O(1/n)
    = 2n/{√(n(n+1)) + √((n-1)n)}・e + O(1/n)
    = e + O(1/n)
    → e, (n→∞)
840132人目の素数さん:2011/12/22(木) 23:16:14.96
>>838 より

 e < g_n < {(2n+1)/2√(n(n+1))}e < {1 + 1/(8n(n+1))}e,

でござるよ。
もっとも、マクローリンを使えば一発だが...

 log(g_n) = (n + 1/2)log(1 + 1/n)
   = (n + 1/2){1/n - 1/(2n^2) +1/(3n^3) - …}
   = 1 + 1/(12n^2) -1/(12n^3) + 3/(40n^4) - …
   < 1 + 1/[12n(n+1)] - 1/[288(n^2)(n+1)^2] + …

∴ e < g_n < {1 + 1/[12n(n+1)]}e,
841132人目の素数さん:2011/12/23(金) 12:32:28.53
>>840

 g_n = {1 + δ - (7/10)δ^2 + 1.0237δ^3 - …}e,
ここに、δ=1/[12n(n+1)],
842132人目の素数さん:2011/12/23(金) 13:34:13.89
>>824
lim[n→∞] { (n+1)^(n+1) / n^n - n^n / (n-1)^(n-1) }
= lim[n→∞] (1 + 1/(n-1))^(n-1) * { (2 + 2/n)*(1 - 1/(n^2))^(n-1) - 1/n Σ[k=1 to n] (1 - 1/(n^2))^(k-1) }
= e(2*1-1)
= e

(1 - 1/(n^2))^(n-1) = 1/{ (1 + 1/(n^2-1))^(n^2-1) }^(1/(n+1)) → 1/e^0 =1

1/n Σ[k=1 to n] (1 - 1/(n^2))^(k-1) → 1 になるのは、はさみうちナリよキテレツ!

      //
    / /__
    /  / lim \   パカッ!
   /.∩|:::: \ ./ |
   / | ||::::(● (●.|_  呼んだ?
  //| |ヽ::::....ワ....ノ/
  " ̄ ̄ ̄ ̄ ̄ ̄"
843132人目の素数さん:2011/12/23(金) 17:31:57.89
>>824
F(x)=(x+1)^(x+1)/x^x と置くと (n+1)^(n+1)/n^n-n^n/(n-1)^(n-1)=F(n)-F(n-1)
f(x)=F'(x)={(x+1)^(x+1)/x^x}*log(1+1/x) とすれば F(n)-F(n-1)=∫[n-1,n]f(x)dx
a(x)=(1+1/x)^x ,b(x)=(1+1/x)^(x+1) と置くと f(x)=a(x)*log(b(x))=b(x)*log(a(x))
x>0のときa(x)は単調増加、b(x)は単調減少で、ともにx→∞でeに収束するので
a(x)<e<b(x)より、a(x)*log(e)<f(x)<b(x)*log(e) すなわち a(x)<f(x)<b(x)
1<n-1≦x≦nのとき a(n-1)<f(x)<b(n-1) なので
∫[n-1,n]a(n-1)dx<∫[n-1,n]f(x)dx<∫[n-1,n]b(n-1)dx
{1+1/(n-1)}^(n-1)<(n+1)^(n+1)/n^n-n^n/(n-1)^(n-1)<{1+1/(n-1)}^n
844132人目の素数さん:2011/12/23(金) 19:39:19.09
(a^2 + b^2 + c^2 + d^2)(p^2 + q^2 + r^2 + s^2) を四平方の和で表せ

    ___
  ./  ≧  \   上のほうで平方和に変形するのがあったような希ガス!
  |::::  \ ./ | ハァハァ
  |::::: (● (● |         プケラッチョ!
  ヽ::::... .∀....ノ /  チン ☆
 _(  ⊃  ⊃  チン ☆
 |\ ̄ ̄ ̄ ̄旦 ̄\
 | | ̄ ̄ ̄ ̄ ̄ ̄ ̄|
 \|  愛媛みかん |
    ̄ ̄ ̄ ̄ ̄ ̄ ̄
845132人目の素数さん:2011/12/23(金) 20:01:06.57
>>841
 g_n = (1 +1/n)^(n +1/2) より

 log(g_n) = (n +1/2)・log(1 +1/n)
   = 1 + Σ[k=1,∞) 1/{(2k+1)(2n+1)^k}
   = 1 + 1/{12(n +1/2)^2} + 1/{80(n +1/2)^4} + 1/{448(n +1/2)^6} + …
   = 1 +δ -(6/5)δ^2 +(72/35)δ^3 -(144/35)δ^4 + …
よって
 g_n = {1 + δ -(7/10)δ^2 +(43/42)δ^3 -(7961/4200)δ^4 + …}e,
ここに、δ=1/[12n(n+1)],
846132人目の素数さん:2011/12/23(金) 20:25:17.98
847132人目の素数さん:2011/12/23(金) 20:47:22.19
自演自重汁
848132人目の素数さん:2011/12/23(金) 20:52:43.00
(a^2 + b^2 + c^2 + d^2)(p^2 + q^2 + r^2 + s^2) を七平方の和で表せ

>>842
呼ばない...
849132人目の素数さん:2011/12/24(土) 01:26:35.21
>>846
(;´д`) ハァハァ、ハァハァ、ハァハァ…
850132人目の素数さん:2011/12/24(土) 01:50:06.06
>>848
どうやって7つに?

>>844
行列式で証明するのが線形代数の問題集に載っていたような
851( ゚∀゚)プケラッチョ!:2011/12/24(土) 02:49:48.95
852132人目の素数さん:2011/12/24(土) 03:11:41.99
>>824 別解(?)

 (n+1)^(n+1)/(n^n) - (n^n)/(n-1)^(n-1)
 = {1 + 1/(n-1)}^n * {(n+1)(1 - 1/n^2)^n - (n-1)}
 = b[n-1] * {(n+1)(1 - 1/n^2)^n -(n-1)},
ここで↓を使う。

〔補題〕
 (n-1)/n < (1 - 1/n^2)^n < n/(n+1),
(略証)
(1+x)^n > 1 + nx (下に凸)より
 (1 - 1/n^2)^n > 1 - 1/n = (n-1)/n,
 {1 + 1/(n^2 -1)}^n > 1 + n/(n^2 -1) > (n+1)/n,
853132人目の素数さん:2011/12/24(土) 04:10:10.46
>>850

コーシーの証明に出て来る「ラグランジュの恒等式」

>>1 まとめWiki → まとめページ → よく使う不等式 → コーシーの不等式 → 証明

http://mathworld.wolfram.com/LagrangesIdentity.html
854132人目の素数さん:2011/12/24(土) 07:32:59.66
Problem A.

A.534.
 三角形の3辺が a,b and c で、対応する中線(medians)の長さはそれぞれ sa, sb and sc とする。
 このとき次を示せ。 sa・sb/(a^2+b^2) + sb・sc/(b^2+c^2) + sc・sa/(c^2+a^2) ≧ 9/8.

A.536.
 正の実数 a,b,c,d が a+b+c+d = abc+abd+acd+bcd を満たす。次を証せ。 >>477 >>507
  (a+b)(c+d) + (a+d)(b+c) ≧ 4√{(1+ac)(1+bd)}.

A.545.
 Prove that whenever a>b>1 are integers such that a+b divides ab+1 and a+b and a-b divides ab-1,
then a < b√3.

A.546.
 次を示せ。
 1/{sin[π/(4k+2)]}^2 + 1/{sin[3π/(4k+2)]}^2 + …… + 1/{sin[(2k-1)π/(4k+2)]}^2 = 2k(k+1),
 (k=3: B.4371.を参照。)


A.548.
 Prove that
  Π[i=1,n] {1 + 1/(x1+…+xi)} + Π[j=1,n] {1 + 1/(xi+…+xn)} ≦ n+1,
holds for arbitrary real numbers x1,……,xn ≧1.
855132人目の素数さん:2011/12/24(土) 07:34:53.93
B.4291.
 すべての正数 a,b,c について、a^b・b^c・c^a ≦ a^a・b^b・c^c,

B.4296.
 m_a、m_b は 三角形の辺a、辺b から見た高さを表わす。
 a>b ならば a^2010 + (m_a)^2010 ≧ b^2010 + (m_b)^2010 を示せ。

B.4297.
 すべての実数x,yに対して -1/2 ≦ (x+y)(1-xy)/{(1+x^2)(1+y^2)} ≦ 1/2 を証せ。

B.4303.
 長方形(正方形でない)をその対角線に沿って2つに折る。
 結果として生じる5角形の周長は、元の長方形の周長より短いことを証せ。

B.4306.
 方程式 16^(x^2 +y) + 16^(y^2 +x) = 1 を解け。

B.4310.
 a0,a1,…,an は正の数で、a(k+1) - ak ≧ 1 (k=0,1,…,n-1)とする。次を示せ。
 1 + (1/a0){1 +1/(a1-a0)}…{1 +1/(an-a0)} ≦ (1+1/a0)(1+1/a1)…(1+1/an),

B.4321.
 どんな三角形でも、次の不等式が成り立つことを証せ。
  b/sin(γ + α/3) + c/sin(β + α/3) > (2/3)a/sin(α/3),
856132人目の素数さん:2011/12/24(土) 07:36:05.65
B.4340.
 すべての正数 a1,a2,…,an に対して次の不等式が成り立つことを証せ。
 {a1/(a2+…+an)}^2 + {a2/(a3+・・・+a1)}^2 + …… + {an/(a1+…+a(n-1))}^2 ≧ n/(n-1)^2,

B.4343.
 a,b は正の数を表わし、a^3 + b^3 =1 とする。a^2 +ab +b^2 -a-b >0 を示せ。

B.4355.
 正数 x,y,z の積が1ならば、次式を証せ。
  (z^3 +y^3)/(x^2 +xy +y^2) + (x^3 +y^3)/(y^2 +yz +z^2) + (y^3 +z^3)/(z^2 +zx +z^2) ≧ 2,

B.4370.
 頂点A,B,C,の対辺の長さを a,b,c とする。BC=a, CA=b, AB=c,
内心をIとおき、AI=u, BI=v, CI=w とおく。このとき次を示せ。
 (a+b+c)(1/u+1/v+1/w) ≦ 3(a/u + b/v + c/w),    >>477 >>480

B.4371.
 1/{sin(π/14)}^2 + 1/{sin(3π/14)}^2 + 1/{sin(5π/14)}^2 = 24,
を示せ。(A.536を参照。)                 >>492

B.4376.
 x,y は負でない数ならば、次式を証せ。
 x^4 + y^3 + x^2 + y + 1 > (9/2)xy,

B.4378.
 pは正の素数とする。
 方程式 x^3・y^3 + x^3・y^2 - x^2・y^3 + x^2・y^2 -x +y = p+2 を解け。
857132人目の素数さん:2011/12/24(土) 08:31:18.57
B.4297.
 (1+x^2)(1+y^2) = (x+y)^2 + (1-xy)^2 ≧ 2|(x+y)(1-xy)|,

B.4306.
 x = y = -1/2,

B.4343.
 a^2 +ab +b^2 -a -b = (a^3 + b^3)/(a+b) +2ab -a -b
   = {1/(a+b) +(a+b) -2} + 2(1-a)(1-b) > 0,

B.4376.
 相加・相乗平均で。
 x^4 + x^2 + 1 ≧ 3x^2,
 y^3 + y ≧ 2y^2,
 (左辺) ≧ 3x^2 + 2y^2 ≧ (2√6)xy > (9/2)xy,
858132人目の素数さん:2011/12/24(土) 08:57:05.85
>>853
成程そのまんまでしたね (恥…

 |
 8 <サンクス
 '`
  ̄
859132人目の素数さん:2011/12/24(土) 16:06:27.63
B4303
三角不等式より。
長方形をABCDとし
BCで折るとする。AD,BCの交点をEとする
AB+BC+CD+DA
=AB+CD+AE+CE+(BE+DE)
>AB+CD+AE+CE+BC
860132人目の素数さん:2011/12/24(土) 16:52:29.17
B.4355.
 チェビシェフより
 (左辺) ≧ (x^3 +y^3)/(x^2 +xy+y^2) + (y^3 +z^3)/(y^2 +yz+z^2) + (z^3 +x^3)/(z^2 +zx+z^2)
   ≧ (x+y)/3 + (y+z)/3 + (z+x)/3   (← *)
   = 2{(x+y+z)/3},
以下、相加・相乗平均で簡単。

*) x^2 +xy +y^2 = 3(x^2 -xy +y^2) -2(x-y)^2 ≦ 3(x^2 -xy +y^2),
861132人目の素数さん:2011/12/24(土) 22:36:08.92
B.4310.
nについての帰納法による。

n=0 のときは等号成立。

n≧1 のとき
 a_k - a_0 ≧ k,  (← 題意)
左辺を A_n とおくと、
 A_n = 1 + (1/a0)Π[k=1,n] {1 + 1/(a_k - a_0)}
   ≦ 1 + (1/a0)Π[k=1,n] (1 + 1/k)
   = 1 + (n+1)/a0,
よって
 A_{n+1} = 1 + (A_n - 1){1 + 1/(a(n+1) - a0)}
   = A_n・(1 + 1/a{n+1}) - a0{a_(n+1)/a0 - A_n}/{a(n+1)・(a(n+1) - a0)}
   ≦ A_n・(1 + 1/a{n+1}) - a0{1 + (n+1)/a0 - A_n}/{a(n+1)・(a(n+1) - a0)}
   ≦ A_n・(1 + 1/a{n+1}),
862859:2011/12/25(日) 11:06:38.95
BCで折るんじゃなくてBDで折るんだった;;;

863132人目の素数さん:2011/12/26(月) 16:31:30.91
B.4297
↑a=(1,x),↑b=(1,-y)とし
↑a,↑bのなす角度をθとすると
(x+y)(1-xy)/{(1+x^2)(1+y^2)}=sinθ・cosθ
864132人目の素数さん:2011/12/26(月) 16:45:47.03
>>863

\vec{a}・\vec{b} / |\vec{a}|・|\vec{b}| = cosθで、あとの奴らは何でござる蟹?
865132人目の素数さん:2011/12/26(月) 21:43:19.42
Letx, y, z>0 with xyz=1.
Prove that x^3+y^3+z^3+6≧(x+y+z)^2
866132人目の素数さん:2011/12/26(月) 21:56:48.42
>>864
 x = tanξ, y = tanη とおくと、
 θ = ξ + η,
 tanθ = tan(ξ+η) = (x+y)/(1-xy),  (加法公式)
そこで
 cosθ = cosξ・cosη - sinξ・sinη
    = (1-xy)・cosξ・cosη,
 sinθ = sinξ・cosη + cosξ・sinη
    = (x+y)・cosξ・cosη,
を辺々掛けて右辺に
 (cosξ)^2 = 1/{1 + (tanξ)^2} = 1/(1+x^2),
 (cosη)^2 = 1/{1 + (tanη)^2} = 1/(1+y^2),
を使ったでご猿。
867132人目の素数さん:2011/12/26(月) 22:16:26.36
>>866
なんと!そんなやり方が…
  ___ 
./  ≧ \ 
|::::  \ ./ | 
|::::: (● (● | < ナルホドナー!
ヽ::::... .ワ.....ノ
  ( つ旦O    ,.-、    ,.-、   ,.-、
  と_)_)    (,,■)  (,,■)  (,,■)

868132人目の素数さん:2011/12/27(火) 00:09:48.53
>865
僕の考えてたこととは違ってたりして勉強になりますた。
ありがとうございます。m(__)m
863の者ですが、
A(1,x),B(1,-y)とすると
x+y=2△OAB (Oは原点)
となります。
2△OAB=|↑a||↑b|sinθ
ってことを考えてました。
869132人目の素数さん:2011/12/27(火) 04:40:53.32
蟹、猿、おにぎり…、なるほどな
870132人目の素数さん:2011/12/28(水) 23:56:03.00
>>498

0 < a,b ≦ c,d としても一般性を失わない。
題意により ab ≦ 1 ≦ cd,
また、a+b+c+d ≧ 2√(ab) +c+d ≧ 2√(ab) + 2/√(ab) ≡ t とおくと t ≧ 4,

左辺を g(a,b,c,d) とおくと
 g(a,b,c,d) - g(√(ab), √(ab),c,d)
 = (√a - √b)^2・{1/ab - 9/[(a+b+c+d)(2√ab +c +d)]}
 ≧ (√a - √b)^2・{1/ab - 9/(t^2)}
 ≧ (√a - √b)^2・(1 - 9/16) ≧ 0,    >>508
∴ a=b の場合を考えれば十分。以下、数セミ(2012/01)と同様。

 s ≡ a+b+c+d ≧ 2a + 2√(cd) = 2a + 2/a ≡ t とおくと t≧4,
(左辺) = 2/a + (c+d)/cd + 9/s
   = 2/a + (a^2)(s-2a) + 9/s
   = 2/a -2a^3 + (a^2・s + 9/s) ≡ f(s),
sの変域は [t,∞) で、極小となるのは s = 3/a のとき。

(i) a ≧ 1/√2 のとき、t≧3/a,
 f(s) ≦ f(t) = t + 9/t
   = 25/4 + (t-4)(t - 9/4)/t
   ≧ 25/4,

(ii) a ≦ 1/√2 のとき、t≦3/a,
 f(s) ≧ f(3/a) = 2/a +6a -2a^3  (aについて単調減少)
   = 9/√2 + (1/√2 - a) + (2/a)(1/√2 - a)^2・{2 -(√2)a -a^2},
   ≧ 9/√2 > 25/4,
871132人目の素数さん:2011/12/29(木) 18:30:34.49
> 789 (790, 800, 801,
次の命題の(2)のn=2の場合と, (1)でm=n=1の場合を合わせると得られます.

命題. m, n は非負整数, r は2以上の整数,
x,y,z≧0 とするとき, 以下の不等式が成立する.
(1) x^(m+n) + y^(m+n) + z^(m+n) ≧ x^m y^n + y^m z^n + z^m x^n
(2) x^(n+2) y + y^(n+2) z + z^(n+2) x ≧ xyz(x^n + y^n + z^n)
(3) x^(n+r) y^r + y^(n+r) z^r + z^(n+r) x^r
≧ xyz(x^(n+r-2) y^(r-1) + y^(n+r-2) z^(r-1) + z^(n+r2) x^(r-1))

証明は, 並べ替え不等式や, 重み付きAM-GM不等式を使うだけです.

斉次交代不等式は, 斉次対称不等式と異なり, Muirheadの不等式等が
使えなくて, 時々, 不等式の形が弱くなります.
> 770 に3変数3〜5次斉次対称不等式の話を書いたけど,
3変数6〜8次斉次対称不等式についても似たような定理(もっとステートメントが
長い)があって, 大体それで解決できてしまいますが,
5次以上の3変数斉次交代不等式のほうは, あまり強力な一般論がありません.
872132人目の素数さん:2011/12/29(木) 19:47:35.43
キャスフィ高校数学板
不等式スレより
743 じゅー [2011/12/29(木) 16:01:06]

出題です??

実数 a,b,c,d,e,f が??
ae + bf + cd + af + bd + ce =0??
ad + be + cf + de + ef + fd = 0??
a^2 + b^2 + c^2 = 1??
d^2 + e^2 + f^2 + 2ab + 2bc + 2ca = 2??
を満たすとき??
|ae + bf + cd - af - bd - ce|??
の最大値を求めて下さい。??
考え方もよければお願いします。??
873132人目の素数さん:2011/12/29(木) 19:51:21.65
スマホの2ch mateとかゆーので書き込むと
文字化け(?)するのか...
874132人目の素数さん:2011/12/30(金) 04:48:34.62
>>871 (3)

・並べ替え(チェビシェフ):
Σ(乱順序積) ≧ Σ(逆順序積) より
 x{x^(n+r-1)・y^r} + y{y^(n+r-1)・z^r} + z{z^(n+r-1)・x^r} ≧ {x^(n+r-1)・y^r}z + {y^(n+r-1)・z^r}x + {z^(n+r-1)・x^r}y,

・ x^(n+r)・y^r を{(n^2+nr+r^2) -r-n}個、y^(n+r)・z^r をr個、z^(n+r)・x^r をn個でAM-GM。
875132人目の素数さん:2011/12/30(金) 05:00:25.30
>>823 >>825

Problem 372. の Solution を貼っておこう....

 abc=1 ゆえ、例によって a=z/y, b=x/z, c=y/x とおく。

 (左辺) = y^2/[z(z+y) +x(x+y)] + z^2/[x(x+z) +y(y+z)] + x^2/[y(y+x) +z(z+x)]
  ≧ (x^2 +y^2 +z^2)^2/{y^2・[z(z+y) +x(x+y)] + cyclic} (← コーシー または 重み付きAM-HM)
  = (x^2 +y^2 +z^2)^2/{xy(x+y)^2 +yz(y+z)^2 +zx(z+x)^2}
  = {(1/2)(x^2 -y^2)^2 +3(xy)^2 + cyclic}/{xy(x+y)^2 +cyclic}
  = 3/4 + {(5/16)(x^2 -y^2)^2 + (3/16)(x-y)^4 + cyclic}/{xy(x+y)^2 +cyclic},
  ≧ 3/4,

*)
 (x^2 -y^2)^2 = (x+y)^2・(x-y)^2 = 4xy(x-y)^2 + (x-y)^4,
 (xy)^2 = (1/4)xy(x+y)^2 -(1/4)xy(x-y)^2,
876132人目の素数さん:2011/12/30(金) 05:24:30.98
877132人目の素数さん:2011/12/30(金) 06:22:02.22
>>823

Problem 377.
 nは正の整数とする。
 i=1,2,・・・,n に対して z_i および w_i は複素数で、次を満たす:
 ε1, ε2,・・・・,εn = ±1 のすべて(2^n とおり)の組合せについて、
 | Σ[i=1,n] εi・z_i | ≦ | Σ[j=1,n] εj・w_j |,
が成り立つ。次式を証せ。
 Σ[i=1,n] |z_i|^2 ≦ Σ[j=1,n] |w_j|^2,
878132人目の素数さん:2011/12/31(土) 06:13:08.99
>>877

 条件式は (両辺) ≧0 だから、2乗しても成り立つ。すなわち、
 {Σ[i=1,n] εi・zi}{Σ[j=1,n] εj・(zj)~} ≦ {Σ[i=1,n] εi・wi}{Σ[j=1,n] εj・(w_j)~},
Σ[j=1,n] εj・(wj)~},
 Σ[i=1,n][j=1,n] εi・εj・zi・(zj)~ ≦ Σ[i=1,n][j=1,n] εi・εj・wi・(wj)~,
εのすべて(2^nとおり)の組合せについて加えると、
 εi・εj → 2^n (i=j)
     → 0  (i≠j)  (±1が同数あるから)
となり、i=j だけが残る。よって
 Σ[i=1,n] zi・zi~ ≦ Σ[j=1,n] wj・wj~,
879132人目の素数さん:2012/01/01(日) 04:29:42.89

C.1043. (201009)
 f(x) = (x+a)^2 /{(a-b)(a-c)} + (x+b)^2 /{(b-c)(b-a)} + (x+c)^2 /{(c-a)(c-b)},
の値を求めよ。ここに a,b,c は相異なる実数である。

K.266. (201011)
 bd > 0 のとき、(a+c)/(b+d) は a/b と c/d の中間にある。
 bd < 0 のとき、(a+c)/(b+d) は a/b と c/d の外側にある。
880132人目の素数さん:2012/01/01(日) 04:39:55.40
年明け早々 簡単すぎた .... orz

(略解)
C.1043.
 f(x) ={(c-b)(x+a)^2 + (a-c)(x+b)^2 + (b-a)(x+c)^2} /{(a-b)(b-c)(c-a)} = 1,
 または f(-a) = (b-a)/(b-c) + (c-a)/(c-b) = 1, f(-b) = 1, f(-c) = 1, から。

K.266.
 {(a/b) - (a+c)/(b+d)}{(a+c)/(b+d) - (c/d)} = (ad-bc)^2/{bd(b+d)^2},

B.4306.
相加・相乗平均で
 (左辺) = 16^(x^2 + y) + 16^(y^2 + x)
  ≧ 16^{(x^2 + y + y^2 + x)/2 + 1/4}
  = 16^{(1/2)(x + 1/2)^2 + (1/2)(y + 1/2)^2}
  ≧ 16^0
  = 1,
等号条件から x = y = -1/2,

B.4340.
 a1 + a2 + ・・・ + an = s とおく。
 φ(x) = {x/(s-x)}^2 = {s/(s-x) - 1}^2 は x<s で下に凸。
881132人目の素数さん:2012/01/03(火) 08:06:41.12
>>865

 (x+y+z)/3 = A, (xyz)^(1/3) = G とおく。 A-G ≧ 0,

(左辺) - (右辺) = 9{A^3 + (x-A)(y-A)(z-A)} - 9AAG
  = 9AA(A-G) + 9(x-A)(y-A)(z-A)
  = 9AA(A-G) + 9{2A^3 -(xy+yz+zx)A +xyz}
  = 9AA(A-G)/4 + (9/4)(2A+G)(A-G)^2 + (3/4)F1(x,y,z)
  ≧ 0,
ここに
 F1(x,y,z) = x(x-y)(x-z) + y(y-z)(y-x) + z(z-x)(z-y)
      = (x+y+z)^3 -4(x+y+z)(xy+yz+zx) + 9xyz ≧ 0, (Schur)
ぬるぽ
882132人目の素数さん:2012/01/03(火) 20:12:43.06
02-01-0014 安 藤 哲 哉 (千 葉 大 理) ] 3 変数斉次巡回不等式と代数曲面
http://mathsoc.jp/meeting/shinshu11sept/talklist/talkList_02.pdf#search=

これ気にならん? ('A`)プケラ
883132人目の素数さん:2012/01/04(水) 00:09:32.16
>>882

■射影幾何学における2つの定理
http://www.geocities.jp/ikuro_kotaro/koramu/666_p3.htm


「代数曲線・代数曲面入門」新装版 −複素代数幾何の源流−
安藤哲哉(著)
出版社:(有)数学書房 (2011/01)
判型: A5判、496頁、
定価: 7350円
ISBN-10: 490334262X
ISBN-13: 978-4903342627
http://www.sugakushobo.co.jp/903342_62_mae.html
日本人初のフィールズ賞受賞者小平邦彦先生をはじめ多くの日本人数学者が貢献した複素代数幾何学への入門書。
定義・命題・定理・証明などの修正、および誤植の訂正をして新装版として出版。


「代数曲線・代数曲面入門」−複素代数幾何の源流−
安藤哲哉(著)
出版社: 白揚社 (2007/02)
判型:A5判、478頁、22cm
定価:7350円
ISBN-10: 4826931077
ISBN-13: 978-4826931076
安藤 哲哉
1959年愛知県瀬戸市生まれ。岐阜県(旧)明智町出身。1982年東京大学理学部数学科卒業。同大学院を経て、1986年千葉大学講師。千葉大学理学部情報・数理学科助教授。理学博士(東京大学)、専門は代数幾何学。(BOOK)
884132人目の素数さん:2012/01/06(金) 19:22:20.17
> 882
その話の内容の2/3は下に書いてあります。
http://www.math.s.chiba-u.ac.jp/~ando/ineq.pdf
残りの1/3の内容は、暇を見つけてタイプします。
日本語のメモ程度のものはタイプしてありますが、UPするにはどうも。
885あのこうちやんは始皇帝だった:2012/01/06(金) 19:29:55.16
>>884

 お前は、定職に就くのが、先決だろが!!!!!!!!!!
886132人目の素数さん:2012/01/08(日) 05:57:33.72
難しい…、ゴクリ
887132人目の素数さん:2012/01/10(火) 23:35:14.84
〔補題〕
a,b,c が実数のとき
 |(a-b)(b-c)(c-a)| ≦ {1/(3√6)}{(a-b)^2 + (b-c)^2 + (c-a)^2}^(3/2),
888132人目の素数さん:2012/01/11(水) 00:36:08.04
>>887

(略証)
bはaとcの中間にある、としてもよい。
 (a-b)(b-c)(c-a) = とおくと
 |處 ≦ (1/4)(|a-b|+|b-c|)^2 |c-a| = (1/4)|c-a|^3,
ところで、
 (c-a)^2 = (1/3){2(a-b)^2 + 2(b-c)^2 - (a-2b+c)^2} + (2/3)(c-a)^2
  ≦ (2/3){(a-b)^2 + (b-c)^2 + (c-a)^2}
  = (4/3)(s^2 -3t),

なお、a,b,c ≧ 0 のときは
 |處 ≦ 0.227083346211・s(s^2 -3t),

http://www.casphy.com/bbs/test/read.cgi/highmath/1169210077/744-745 , 527
 高校数学 - 不等式
889132人目の素数さん:2012/01/11(水) 22:07:10.38
>>887

 竸2 = {(a-b)(b-c)(c-a)}^2
    = (1/54){(a-b)^2 + (b-c)^2 + (c-a)^2}^3 - (1/27){(2a-b-c)(2b-c-a)(2c-a-b)}^2,
    = (4/27)(s^2 -3t)^3 - (1/27){(3a-s)(3b-s)(3c-s)}^2,
890132人目の素数さん:2012/01/11(水) 22:28:53.84
889すげっ。
メモメモ...φ(..)
891132人目の素数さん:2012/01/15(日) 04:08:40.30
>>887-889

 (2a-b-c)/3 = a - s/3 = a ',
 (2b-c-a)/3 = b - s/3 = b ',
 (2c-a-b)/3 = c - s/3 = c ',
と置くのがいいらしいヨ

casphy - 高校数学 - 不等式 - 749
892132人目の素数さん:2012/01/15(日) 23:37:42.81
おもしろいね
893132人目の素数さん:2012/01/15(日) 23:49:34.37
〔補題〕
 a,b,c ≧ 0
  = (a-b)(b-c)(c-a),
のとき
 |處 ≦ {(a+b+c)^3 -27abc}/(6√3),


casphy - 高校数学 - 不等式 - 748-750
894132人目の素数さん:2012/01/20(金) 14:45:56.52
最近知った不等式と言えば 「小澤の不等式」 ( ゚∀゚)プケラッチョ!
http://mainichi.jp/select/science/news/20120116k0000m040090000c.html
895132人目の素数さん:2012/01/21(土) 03:12:10.06
896132人目の素数さん:2012/01/22(日) 17:12:30.47
落ちたのかと思った ( ゚∀゚)プケラッチョ!
897132人目の素数さん:2012/01/22(日) 22:36:25.33
いや落ちてたでしょ
898132人目の素数さん:2012/01/22(日) 22:56:54.51
a<b<c
rr2r=2r^3
3^3b^3-27(b^2-r^2)b=27r^2b/6*3^.5=4.5r^2b/3^.5
899132人目の素数さん:2012/01/23(月) 23:53:25.33
ふたばから
x^y+y^x>1
x,y>1を示せ

対数とか取らずに解いて欲しいですね
900132人目の素数さん:2012/01/23(月) 23:54:14.95
既出
901132人目の素数さん:2012/01/24(火) 01:34:42.06
>>899
過去ログを見たまえ
902132人目の素数さん:2012/01/24(火) 01:45:26.56
x=y=1/2
903132人目の素数さん:2012/01/28(土) 00:35:32.84
a、b、c >0 のとき、a^3/(a+b)^2 + b^3/(b+c)^2 + c^3/(c+a)^2 ≧ (a+b+c)/4

さいきん立読み中に見かけた問題だが、既出な伊予柑 ( ゚∀゚)プケラッチョ!
904132人目の素数さん:2012/01/28(土) 00:42:37.07
>>899
むしろ対数を取って証明する方法を知りたい

さあ、改造手術の時間です!
a、b >0に対して、 a^a + b^b ≧ a^b + b^a > 1

( ゚∀゚)プケラッチョ!
905132人目の素数さん:2012/01/28(土) 00:54:02.62
>>903
a^3/(a+b)^2≧(2a-b)/4から示す
906132人目の素数さん:2012/01/28(土) 01:40:14.79
>>905
どこから出てくるん、その発想
907名無しさん:2012/01/30(月) 21:15:10.90 ID:AiSkvLuw
>>903
ではでは、次はどんな方法で?
a、b、c >0 のとき、a^2/(a+b) + b^2/(b+c) + c^2/(c+a) ≧ (a+b+c)/2

一般化はできますか? ( ゚∀゚)プケラッチョ!
a、b、c >0 のとき、a^(n+1)/(a+b)^n + b^(n+1)/(b+c)^n + c^(n+1)/(c+a)^n ≧ (a+b+c)/?
908名無しさん:2012/01/30(月) 21:17:38.49 ID:AiSkvLuw
専ブラから書き込めなかったので、落ちたのかと思ったぜ…
IDが出てるし、何が起こったのだ ('A`)ヴォエァ!
909名無しさん:2012/01/30(月) 21:30:41.60 ID:xfpFxcpO
まじだ
910名無しさん:2012/01/30(月) 21:34:49.65 ID:qQ0NhdK4
>>907
a、b、c >0 のとき、a^(n+1)/(a+b)^n + b^(n+1)/(b+c)^n + c^(n+1)/(c+a)^n ≧ (a+b+c)/2^n
911名無しさん:2012/01/30(月) 21:44:24.85 ID:???
>>907
a^2/(a+b)≧(3a-b)/4から示す
912名無しさん:2012/01/30(月) 21:58:49.60 ID:???
>>911
どこから捻り出すのか教えて栗々ポンポン ( ゚∀゚)!
913名無しさん:2012/01/30(月) 22:01:11.43 ID:???
そのコツが分かれば、a^4/(a+b)^3 + b^4/(b+c)^3 + c^4/(c+a)^3 でも作れる鴨
914名無しさん:2012/01/30(月) 22:32:37.59 ID:???
a、b、c >0 のとき、a^(n+1)/(a+b)^n + b^(n+1)/(b+c)^n + c^(n+1)/(c+a)^n ≧ (a+b+c)/2^n

( ゚∀゚) しゅっび どぅっび〜
915名無しさん:2012/01/30(月) 22:41:55.32 ID:???
>>912
a^2/(a+b)≧(xa+(1-x)b)/2
から上手くなってくれるように調整
2a^2≧xa^2+ab+(1-x)b^2
(a-b)((2-x)a+(1-x)b)≧0
からx=3/2だとうまくいくなーと
916名無しさん:2012/01/30(月) 22:52:48.88 ID:???
>>915
ぐぬぬ…、なるほどな〜
そうやって理詰めで作り出すんですね〜 ヽ('A`)ノ
917132人目の素数さん:2012/02/04(土) 01:56:44.26
 f(t) = 1/(1+t)^n は n>0 と n<-1 で下に凸。よって Jensen により
 (左辺) = a・f(b/a) + b・f(c/b) + c・f(a/c) ≧ (a+b+c)・f(1) = (a+b+c)/(2^n),

>>912
 t=1 で接線を曳く。
 f(t) ≧ f(1) + f '(1)(t-1) = (n+2 -nt)/(2^(n+1)),
918132人目の素数さん:2012/02/04(土) 05:27:35.88
>>917
神光臨キタ━━━┌(_Д_┌ )┐━━━!!
919132人目の素数さん:2012/02/18(土) 22:45:48.88
〔B.C.Carlsonの不等式〕
{sinθ, sinθ, tanθ} の相加平均A、相乗平均G、調和平均Hについて
 H ≦ θ ≦ G ≦ A,


http://uni.2ch.net/test/read.cgi/math/1328045950/22-24
大関:参考書[3] p.45-46 x=cos(2θ) とおく。
920132人目の素数さん:2012/02/19(日) 17:04:34.06
はぁはぁ…
921132人目の素数さん:2012/02/21(火) 19:34:44.87
プケラッチョ!
( ゚∀゚)つ http://arxiv.org/pdf/1201.5534v1.pdf
922132人目の素数さん:2012/02/22(水) 02:14:02.62
>>921

大関:参考書[3] の p.71-72 方法4.
923132人目の素数さん:2012/02/22(水) 11:49:02.40
          r〜〜〜〜〜
   __    _ノ うっうっうっ・・・
  /__  `ヽ_ ⌒ヽ〜〜〜〜〜
  |〈___ノf レ1(
 ,L| しL.し'゙"
 "`  "′
924132人目の素数さん:2012/02/22(水) 19:17:21.73
x、y >0、 z≧x+yのとき、x^2+y^2+z^2 ≧(6/5)(xy+yz+zx)
( ゚∀゚)ブリッ!
925132人目の素数さん:2012/02/22(水) 21:46:39.74
シャピロ(Shapiro)の巡回不等式の証明です。
大関[3]でも取り上げられています。
http://www.math.s.chiba-u.ac.jp/~ando/Shapiro.pdf
926132人目の素数さん:2012/02/22(水) 22:41:53.90
>>924

z = x + y + Z' を入れて
 x^2 + y^2 + z^2 - (6/5)(xy+yz+zx)
 = x^2 + y^2 + (x+y+Z')^2 - (6/5){xy + (x+y)(x+y+Z')}
 = (4/5)(x-y)^2 + (4/5)(x+y)Z' + (Z')^2,
ここで題意より x+y>0, Z' ≧0.
927132人目の素数さん:2012/02/23(木) 00:15:12.40
          __ノ)-'´ ̄ ̄`ー- 、_
        , '´  _. -‐'''"二ニニ=-`ヽ、
      /   /:::::; -‐''"        `ーノ
     /   /:::::/           \
     /    /::::::/          | | |  |
     |   |:::::/ /     |  | | | |  |
      |   |::/ / / |  | ||  | | ,ハ .| ,ハ|
      |   |/ / / /| ,ハノ| /|ノレ,ニ|ル' 
     |   |  | / / レ',二、レ′ ,ィイ|゙/   私は只の数ヲタなんかとは付き合わないわ。
.     |   \ ∠イ  ,イイ|    ,`-' |      頭が良くて数学が出来てかっこいい人。それが必要条件よ。
     |     l^,人|  ` `-'     ゝ  |        さらに Ann.of Math に論文書けば十分条件にもなるわよ。
      |      ` -'\       ー'  人          一番嫌いなのは論文数を増やすためにくだらない論文を書いて
    |        /(l     __/  ヽ、           良い論文の出版を遅らせるお馬鹿な人。
     |       (:::::`‐-、__  |::::`、     ヒニニヽ、         あなたの論文が Ann of Math に accept される確率は?
    |      / `‐-、::::::::::`‐-、::::\   /,ニニ、\            それとも最近は Inv. Math. の方が上かしら?
   |      |::::::::::::::::::|` -、:::::::,ヘ ̄|'、  ヒニ二、 \
.   |      /::::::::::::::::::|::::::::\/:::O`、::\   | '、   \
   |      /:::::::::::::::::::/:::::::::::::::::::::::::::::'、::::\ノ  ヽ、  |
  |      |:::::/:::::::::/:::::::::::::::::::::::::::::::::::'、',::::'、  /:\__/‐、
  |      |/:::::::::::/::::::::::::::::::::::::::::::::::O::| '、::| く::::::::::::: ̄|
   |     /_..-'´ ̄`ー-、:::::::::::::::::::::::::::::::::::|/:/`‐'::\;;;;;;;_|
   |    |/::::::::::::::::::::::\:::::::::::::::::::::::::::::|::/::::|::::/:::::::::::/
    |   /:::::::::::::::::::::::::::::::::|:::::::::::::::::::::O::|::|::::::|:::::::::::::::/
928132人目の素数さん:2012/02/23(木) 08:30:24.39
ペレルマンのような数学者もいる一方で、有名論文誌にアクセプトされることだけを
夢見ているような人もいるわけだから、世の中いろいろだね。
929132人目の素数さん:2012/02/23(木) 20:00:02.25
Page 3、Problem 384
http://www.math.ust.hk/excalibur/v16_n3.pdf

  ∧_∧
  ( ;´∀`) < おまいらも不等式を見てボッキするだろ?
  人 Y /
 ( ヽ し
 (_)_)
930132人目の素数さん:2012/02/24(金) 00:13:04.88
>>929

〔Problem 384.〕
For all positive real numbers a,b,c satisfying a+b+c=3, prove that
 (aa+3bb)/{abb(4-ab)} + (bb+3cc)/{bcc(4-bc)} + (cc+3aa)/{caa(4-ca)} ≧ 4.
931132人目の素数さん:2012/02/27(月) 21:51:47.13
>>930
お手上げ侍 ヽ(・∀・)/
932132人目の素数さん:2012/02/28(火) 13:03:59.60
933132人目の素数さん:2012/02/28(火) 16:38:34.82
>>932
うむ、大関の書いた本だからな。ぶりっ
934132人目の素数さん:2012/02/28(火) 18:41:17.41
>>932
先月予約済みだわい
935132人目の素数さん:2012/02/29(水) 11:43:48.53
よだれが出てきた…、うひょひょひょひょひょっ!
( ゚ρ゚)つhttp://www.kyoritsu-pub.co.jp/bookdetail/9784320019898
936132人目の素数さん:2012/02/29(水) 19:34:28.87
>>935
マシューの不等式って、何ぞや?
937132人目の素数さん:2012/03/01(木) 03:56:10.95
ウヒョッ!
( ゚∀゚)つ △ABCの面積Sについて、S ≦ (a+b+c)^2 / (12√3)
938132人目の素数さん:2012/03/03(土) 15:26:50.71
>>936

〔マシュー(Mathieu)の不等式〕
K > 0 のとき
 1/(K + 1/2) < Σ(n=1,∞) 2n/(n^2 + K)^2 < 1/K,

casphy - 高校数学 - 不等式 - 783〜784
http://www.casphy.com/bbs/test/read.cgi/highmath/1169210077/783-784


>>937

例によって (a+b+c)/2 = s とおく。
相乗・相加平均から、
 (s-a)(s-b)(s-c) ≦ (s/3)^3,
これをヘロンの公式に入れて
 S = √{s(s-a)(s-b)(s-c)} ≦ √{s(s/3)^3} = (s^2)/(3√3).
939132人目の素数さん:2012/03/03(土) 17:47:32.94
>>938

-1 < K < 1 では
 2{ζ(3) - 2Kζ(5)} ≦ Σ(n=1,∞) 2n/(n^2 + K)^2 ≦ 2ζ(3),
の方がいい鴨。

(略証)
 (n^2 -2K)/n^6 < {(n^2 - K)/n^4}^2 < 1/(n^2 + K)^2 < 1/n^4,
から。
940132人目の素数さん:2012/03/08(木) 18:02:50.09
( ゚∀゚)つ 答えを持っていないので申し訳ないが…

鋭角三角形の辺の長さをa、b、c、内接円の半径r、外接円の半径Rに対して、
r/R ≦ sqrt{2*(2a^2-(b-c)^2)*(2b^2-(c-a)^2)*(2c^2-(a-b)^2)} / {(a+b)(b+c)(c+a)}
941132人目の素数さん:2012/03/08(木) 18:22:22.06
もう一つあった ('A`)ヴォエァ!

R/2r ≧ exp{(a-b)^2/2c^2 + (b-c)^2/2a^2 + (c-a)^2/2b^2}
942132人目の素数さん:2012/03/08(木) 18:23:06.92
>>941は任意の三角形で。
943132人目の素数さん:2012/03/08(木) 23:01:47.16
|{exp(tx)}-{exp(ty)}|≦(t/2)*|x-y|*({exp(tx)}+{exp(ty)})
t,x,yは任意の実数
この不等式の証明が判りません。
教えていただければ幸いです。
お願いします。
944132人目の素数さん:2012/03/09(金) 04:01:50.89
>>943

u>0 とする。
f(u) = exp(u) とおくと
 ∫[-u,u] f(u ') du ' = ∫[-u,u] exp(u ') du ' = exp(u) - exp(-u),
一方、台形近似では
 u*{f(u)+f(-u)} = u*{exp(u)+exp(-u)},
f(u) は下に凸だから、
 |exp(u) - exp(-u)| ≦ |u|{exp(u) + exp(-u)},

これに u = |(t/2)(x-y)| を入れて exp{(t/2)(x+y)} を掛ける。
945132人目の素数さん:2012/03/09(金) 10:49:34.81
>>944
すごい!ありがとうございます、お陰ですっきりしました!
946132人目の素数さん:2012/03/09(金) 16:17:45.67
>>943
t<0なら成立しない
947132人目の素数さん:2012/03/09(金) 22:39:54.45
確かによくみたらtは非負って書いてました。
ご指摘ありがとう。
948132人目の素数さん:2012/03/10(土) 03:09:30.30
>>944

u ' = 0 での接線で近似すると、面積は 2u

以上により 
 2|u| ≦ |exp(u) - exp(-u)| ≦ |u|{exp(u)+exp(-u)},
949132人目の素数さん:2012/03/11(日) 23:26:05.21
なるほどなー
950132人目の素数さん:2012/03/12(月) 14:11:25.99
正の実数a, b, c が、a*b*c=1 を満たすとき、
 a^3 + b^3 + c^3 ≧ a^2 + b^2 + c^2
が成り立つことを証明せよ。
951132人目の素数さん:2012/03/12(月) 16:14:45.47
>>950
一目で5通りくらいの解法が浮かぶほどの既出問題。
ぶりぶりっと改造せずにはいられない。( ゚∀゚)ブリッ!

abc=1をみたす正の数a、b、cと任意の自然数nに対して、
a^n + b^n + c^n ≧ a^{n-1} + b^{n-1} + c^{n-1}
952132人目の素数さん:2012/03/12(月) 18:02:52.91
>>951
5通りもあるのか・・・1日考えたが解けん
既出問題なら答えを教えてくれませんか?
ちなみに出展は大数2002年4月号の宿題
953132人目の素数さん:2012/03/12(月) 18:47:43.70
ヒラメキなしで普通に解くなら、
f(x) = a^x + b^x + c^x が単調増加することを示す
( ゚∀゚)ブリブリブリッ!
954132人目の素数さん:2012/03/12(月) 20:00:00.25
p>q≧0。
x≧0。
y≧0。
z≧0。
r=(p−q)/3p。
s=(p+2q)/3p。
rx+ry+sz≧x^ry^rz^s。
x+y+z≧(xyz)^r(x^(s−r)+y^(s−r)+z^(s−r))。
x=a^p。
y=b^p。
z=c^p。
abc=1。
a^p+b^p+c^p≧a^q+b^q+c^q。
955132人目の素数さん:2012/03/15(木) 07:24:30.66
          __ノ)-'´ ̄ ̄`ー- 、_
        , '´  _. -‐'''"二ニニ=-`ヽ、
      /   /:::::; -‐''"        `ーノ
     /   /:::::/           \
     /    /::::::/          | | |  |
     |   |:::::/ /     |  | | | |  |
      |   |::/ / / |  | ||  | | ,ハ .| ,ハ|
      |   |/ / / /| ,ハノ| /|ノレ,ニ|ル' 
     |   |  | / / レ',二、レ′ ,ィイ|゙/   私は只の数ヲタなんかとは付き合わないわ。
.     |   \ ∠イ  ,イイ|    ,`-' |      頭が良くて数学が出来てかっこいい人。それが必要条件よ。
     |     l^,人|  ` `-'     ゝ  |        さらに Ann.of Math に論文書けば十分条件にもなるわよ。
      |      ` -'\       ー'  人          一番嫌いなのは論文数を増やすためにくだらない論文を書いて
    |        /(l     __/  ヽ、           良い論文の出版を遅らせるお馬鹿な人。
     |       (:::::`‐-、__  |::::`、     ヒニニヽ、         あなたの論文が Ann of Math に accept される確率は?
    |      / `‐-、::::::::::`‐-、::::\   /,ニニ、\            それとも最近は Inv. Math. の方が上かしら?
   |      |::::::::::::::::::|` -、:::::::,ヘ ̄|'、  ヒニ二、 \
.   |      /::::::::::::::::::|::::::::\/:::O`、::\   | '、   \
   |      /:::::::::::::::::::/:::::::::::::::::::::::::::::'、::::\ノ  ヽ、  |
  |      |:::::/:::::::::/:::::::::::::::::::::::::::::::::::'、',::::'、  /:\__/‐、
  |      |/:::::::::::/::::::::::::::::::::::::::::::::::O::| '、::| く::::::::::::: ̄|
   |     /_..-'´ ̄`ー-、:::::::::::::::::::::::::::::::::::|/:/`‐'::\;;;;;;;_|
   |    |/::::::::::::::::::::::\:::::::::::::::::::::::::::::|::/::::|::::/:::::::::::/
    |   /:::::::::::::::::::::::::::::::::|:::::::::::::::::::::O::|::|::::::|:::::::::::::::/
956132人目の素数さん:2012/03/23(金) 20:45:32.87
                  _______
         /      ∩|   |
        /      / /|不等式
           /  / /  |____|
              / /     | |
  .        / / /∧   ./ /
            / / ´_ゝ`)/  おうい!大関の新刊書を手に入れたぞー!
          / |      /
            |    /
            |   /⌒l
             ヽ   | /
           / | ゙ー'| L
        /     |  /(_  ヽ
         / / ノ
       /  / /
     /   ( ヽ  
957132人目の素数さん:2012/03/23(金) 20:47:16.26
                    | |    |    | |
                 |     | | || | | |  |
                    |  |  ∧_∧  |      |
                  || | /⌒( ´_ゝ`) 本気だせよ! タコ助
                   |  |/  ノ    ) ノ ||  |     |
                   | /  ノゝ   / ノ  |   |   |   |
                    (  ヽ   __ヽ ||   |  | | |
                    \ \  |   |  / / |   ||  || |  |
                 | |_/ /ヽ つ|不等式つ  7   | ||  || | |
               | / ____<    |__|   >
                     <       ガシャッ!!!
                       V V V V
958132人目の素数さん:2012/03/23(金) 20:52:36.97

  ∧_∧   / ̄ ̄ ̄ ̄ ̄ ̄ ̄ ̄ ̄ ̄ ̄ ̄ ̄ ̄ ̄ ̄ ̄
 ( ・∀・)< 不等式専門の本が出ただけでも良しとせねば
 ( 建前 )  \_________________
 | | |
__(__)_)___________________
 ( _)_)
 | | |
 ( 本音 )  / ̄ ̄ ̄ ̄ ̄ ̄ ̄ ̄ ̄ ̄ ̄ ̄ ̄ ̄ ̄ ̄ ̄
 ( 。A。)< いくら素人向けシリーズでも、手を抜くなよ!
  ∨ ̄∨   \_________________
959132人目の素数さん:2012/03/24(土) 12:35:17.99
かわいいw
他のシリーズとレベルを合わせたのかもね
960132人目の素数さん:2012/03/24(土) 18:41:11.73
共立のDQNシリーズは、DQNをなめすぎだと思うよ
かつてのワンポイント双書の平成版ということらしいが、どうだか…
961132人目の素数さん:2012/03/24(土) 19:04:10.09
著者には本気で不等式の本を執筆してもらいたいものだな ( ゚∀゚)
962132人目の素数さん:2012/03/24(土) 20:57:23.87
本文よりも寧ろ参考文献として挙げられているヤツが気になった
963132人目の素数さん:2012/03/25(日) 02:31:17.73
他に買った人の意見を聞きたいッ!
964132人目の素数さん:2012/03/25(日) 10:17:10.37
>>962
kwsk
965132人目の素数さん:2012/03/25(日) 10:45:17.90
>>964
http://www.amazon.com/Analytic-Inequalities-D-S-Mitrinovic/dp/0387048375

不等式ヲタに違いない著者の大関が、こう述べている
「一応分類はされているが、不等式がこれでもかというほど並んでいて、いくら眺めても飽きない」
966132人目の素数さん:2012/03/25(日) 11:25:59.51
>>958
他のシリーズを立読みしてみたら、なんとなく分かったような気がした
前作の「不等式への招待」は枠で囲んであって読みやすかったが、
今回の読みづらさ、余白の多さはシリーズ全般に当てはまる
シリーズの構成上、紹介本にせざるを得なかったんじゃないのかなと…

だからこそ、シリーズじゃない不等式の本を出してほしいと思った
967132人目の素数さん:2012/03/26(月) 13:23:55.51
          __ノ)-'´ ̄ ̄`ー- 、_
        , '´  _. -‐'''"二ニニ=-`ヽ、
      /   /:::::; -‐''"        `ーノ
     /   /:::::/           \
     /    /::::::/          | | |  |
     |   |:::::/ /     |  | | | |  |
      |   |::/ / / |  | ||  | | ,ハ .| ,ハ|
      |   |/ / / /| ,ハノ| /|ノレ,ニ|ル' 
     |   |  | / / レ',二、レ′ ,ィイ|゙/   私は只の数ヲタなんかとは付き合わないわ。
.     |   \ ∠イ  ,イイ|    ,`-' |      頭が良くて数学が出来てかっこいい人。それが必要条件よ。
     |     l^,人|  ` `-'     ゝ  |        さらに Ann.of Math に論文書けば十分条件にもなるわよ。
      |      ` -'\       ー'  人          一番嫌いなのは論文数を増やすためにくだらない論文を書いて
    |        /(l     __/  ヽ、           良い論文の出版を遅らせるお馬鹿な人。
     |       (:::::`‐-、__  |::::`、     ヒニニヽ、         あなたの論文が Ann of Math に accept される確率は?
    |      / `‐-、::::::::::`‐-、::::\   /,ニニ、\            それとも最近は Inv. Math. の方が上かしら?
   |      |::::::::::::::::::|` -、:::::::,ヘ ̄|'、  ヒニ二、 \
.   |      /::::::::::::::::::|::::::::\/:::O`、::\   | '、   \
   |      /:::::::::::::::::::/:::::::::::::::::::::::::::::'、::::\ノ  ヽ、  |
  |      |:::::/:::::::::/:::::::::::::::::::::::::::::::::::'、',::::'、  /:\__/‐、
  |      |/:::::::::::/::::::::::::::::::::::::::::::::::O::| '、::| く::::::::::::: ̄|
   |     /_..-'´ ̄`ー-、:::::::::::::::::::::::::::::::::::|/:/`‐'::\;;;;;;;_|
   |    |/::::::::::::::::::::::\:::::::::::::::::::::::::::::|::/::::|::::/:::::::::::/
    |   /:::::::::::::::::::::::::::::::::|:::::::::::::::::::::O::|::|::::::|:::::::::::::::/
968132人目の素数さん:2012/03/26(月) 17:56:17.70
          __ノ)-'´ ̄ ̄`ー- 、_
        , '´  _. -‐'''"二ニニ=-`ヽ、
      /   /:::::; -‐''"        `ーノ
     /   /:::::/           \
     /    /::::::/          | | |  |
     |   |:::::/ /     |  | | | |  |
      |   |::/ / / |  | ||  | | ,ハ .| ,ハ|
      |   |/ / / /| ,ハノ| /|ノレ,ニ|ル' 
     |   |  | / / レ',二、レ′ ,ィイ|゙/   私は只の数ヲタなんかとは付き合わないわ。
.     |   \ ∠イ  ,イイ|    ,`-' |      頭が良くて数学が出来てかっこいい人。それが必要条件よ。
     |     l^,人|  ` `-'     ゝ  |        さらに Ann.of Math に論文書けば十分条件にもなるわよ。
      |      ` -'\       ー'  人          一番嫌いなのは論文数を増やすためにくだらない論文を書いて
    |        /(l     __/  ヽ、           良い論文の出版を遅らせるお馬鹿な人。
     |       (:::::`‐-、__  |::::`、     ヒニニヽ、         あなたの論文が Ann of Math に accept される確率は?
    |      / `‐-、::::::::::`‐-、::::\   /,ニニ、\            それとも最近は Inv. Math. の方が上かしら?
   |      |::::::::::::::::::|` -、:::::::,ヘ ̄|'、  ヒニ二、 \
.   |      /::::::::::::::::::|::::::::\/:::O`、::\   | '、   \
   |      /:::::::::::::::::::/:::::::::::::::::::::::::::::'、::::\ノ  ヽ、  |
  |      |:::::/:::::::::/:::::::::::::::::::::::::::::::::::'、',::::'、  /:\__/‐、
  |      |/:::::::::::/::::::::::::::::::::::::::::::::::O::| '、::| く::::::::::::: ̄|
   |     /_..-'´ ̄`ー-、:::::::::::::::::::::::::::::::::::|/:/`‐'::\;;;;;;;_|
   |    |/::::::::::::::::::::::\:::::::::::::::::::::::::::::|::/::::|::::/:::::::::::/
    |   /:::::::::::::::::::::::::::::::::|:::::::::::::::::::::O::|::|::::::|:::::::::::::::/
969132人目の素数さん:2012/03/26(月) 18:53:37.98
このスレまで荒らし始めたか、キチガイだな
970132人目の素数さん:2012/03/27(火) 00:40:55.78
          __ノ)-'´ ̄ ̄`ー- 、_
        , '´  _. -‐'''"二ニニ=-`ヽ、
      /   /:::::; -‐''"        `ーノ
     /   /:::::/           \
     /    /::::::/          | | |  |
     |   |:::::/ /     |  | | | |  |
      |   |::/ / / |  | ||  | | ,ハ .| ,ハ|
      |   |/ / / /| ,ハノ| /|ノレ,ニ|ル' 
     |   |  | / / レ',二、レ′ ,ィイ|゙/   私は只の数ヲタなんかとは付き合わないわ。
.     |   \ ∠イ  ,イイ|    ,`-' |      頭が良くて数学が出来てかっこいい人。それが必要条件よ。
     |     l^,人|  ` `-'     ゝ  |        さらに Ann.of Math に論文書けば十分条件にもなるわよ。
      |      ` -'\       ー'  人          一番嫌いなのは論文数を増やすためにくだらない論文を書いて
    |        /(l     __/  ヽ、           良い論文の出版を遅らせるお馬鹿な人。
     |       (:::::`‐-、__  |::::`、     ヒニニヽ、         あなたの論文が Ann of Math に accept される確率は?
    |      / `‐-、::::::::::`‐-、::::\   /,ニニ、\            それとも最近は Inv. Math. の方が上かしら?
   |      |::::::::::::::::::|` -、:::::::,ヘ ̄|'、  ヒニ二、 \
.   |      /::::::::::::::::::|::::::::\/:::O`、::\   | '、   \
   |      /:::::::::::::::::::/:::::::::::::::::::::::::::::'、::::\ノ  ヽ、  |
  |      |:::::/:::::::::/:::::::::::::::::::::::::::::::::::'、',::::'、  /:\__/‐、
  |      |/:::::::::::/::::::::::::::::::::::::::::::::::O::| '、::| く::::::::::::: ̄|
   |     /_..-'´ ̄`ー-、:::::::::::::::::::::::::::::::::::|/:/`‐'::\;;;;;;;_|
   |    |/::::::::::::::::::::::\:::::::::::::::::::::::::::::|::/::::|::::/:::::::::::/
    |   /:::::::::::::::::::::::::::::::::|:::::::::::::::::::::O::|::|::::::|:::::::::::::::/
971132人目の素数さん:2012/03/27(火) 04:14:07.44
新刊書に面白い不等式は載ってないん?
972132人目の素数さん:2012/03/27(火) 05:06:09.35
パラパラッと見たけど、難しい不等式に限って「証明略」なんだよな…
略すんなら、掲載されている論文とか書いとけよと… ('A`)ヴォエァ!

973132人目の素数さん:2012/03/27(火) 06:50:32.28
>>971
( ゚∀゚)つ 自然数m、nに対して、1/(m+n+1) - 1/{(m+1)(n+1)} ≦ 4/45
974132人目の素数さん:2012/03/27(火) 12:35:26.75
ハアハア・・・( ゚∀゚)
975132人目の素数さん:2012/03/27(火) 12:54:03.92
          __ノ)-'´ ̄ ̄`ー- 、_
        , '´  _. -‐'''"二ニニ=-`ヽ、
      /   /:::::; -‐''"        `ーノ
     /   /:::::/           \
     /    /::::::/          | | |  |
     |   |:::::/ /     |  | | | |  |
      |   |::/ / / |  | ||  | | ,ハ .| ,ハ|
      |   |/ / / /| ,ハノ| /|ノレ,ニ|ル' 
     |   |  | / / レ',二、レ′ ,ィイ|゙/   私は只の数ヲタなんかとは付き合わないわ。
.     |   \ ∠イ  ,イイ|    ,`-' |      頭が良くて数学が出来てかっこいい人。それが必要条件よ。
     |     l^,人|  ` `-'     ゝ  |        さらに Ann.of Math に論文書けば十分条件にもなるわよ。
      |      ` -'\       ー'  人          一番嫌いなのは論文数を増やすためにくだらない論文を書いて
    |        /(l     __/  ヽ、           良い論文の出版を遅らせるお馬鹿な人。
     |       (:::::`‐-、__  |::::`、     ヒニニヽ、         あなたの論文が Ann of Math に accept される確率は?
    |      / `‐-、::::::::::`‐-、::::\   /,ニニ、\            それとも最近は Inv. Math. の方が上かしら?
   |      |::::::::::::::::::|` -、:::::::,ヘ ̄|'、  ヒニ二、 \
.   |      /::::::::::::::::::|::::::::\/:::O`、::\   | '、   \
   |      /:::::::::::::::::::/:::::::::::::::::::::::::::::'、::::\ノ  ヽ、  |
  |      |:::::/:::::::::/:::::::::::::::::::::::::::::::::::'、',::::'、  /:\__/‐、
  |      |/:::::::::::/::::::::::::::::::::::::::::::::::O::| '、::| く::::::::::::: ̄|
   |     /_..-'´ ̄`ー-、:::::::::::::::::::::::::::::::::::|/:/`‐'::\;;;;;;;_|
   |    |/::::::::::::::::::::::\:::::::::::::::::::::::::::::|::/::::|::::/:::::::::::/
    |   /:::::::::::::::::::::::::::::::::|:::::::::::::::::::::O::|::|::::::|:::::::::::::::/
976132人目の素数さん:2012/03/27(火) 13:15:59.00
次の不等式は,ある定理から簡単に導かれるのですが,基礎知識なしで解くと,どの程度の難易度と判定されるのでしょうか.

a, b, c を任意の実数とするとき,
(3+\sqrt{3})(a^6+b^6+c^6) + (45-3\sqrt{3}) a^2 b^2 c^2 ≧ 18 a b c (a^3+b^3+c^3)
等号成立条件は a=b=c の他,(a:b:c) = (\sqrt{3}-1 : 1 : 1) とその置換.

類似の不等式が10個ほどあります.

私も不等式の本を1冊書きますので,よろしくお願いします.
977132人目の素数さん
          __ノ)-'´ ̄ ̄`ー- 、_
        , '´  _. -‐'''"二ニニ=-`ヽ、
      /   /:::::; -‐''"        `ーノ
     /   /:::::/           \
     /    /::::::/          | | |  |
     |   |:::::/ /     |  | | | |  |
      |   |::/ / / |  | ||  | | ,ハ .| ,ハ|
      |   |/ / / /| ,ハノ| /|ノレ,ニ|ル' 
     |   |  | / / レ',二、レ′ ,ィイ|゙/   私は只の数ヲタなんかとは付き合わないわ。
.     |   \ ∠イ  ,イイ|    ,`-' |      頭が良くて数学が出来てかっこいい人。それが必要条件よ。
     |     l^,人|  ` `-'     ゝ  |        さらに Ann.of Math に論文書けば十分条件にもなるわよ。
      |      ` -'\       ー'  人          一番嫌いなのは論文数を増やすためにくだらない論文を書いて
    |        /(l     __/  ヽ、           良い論文の出版を遅らせるお馬鹿な人。
     |       (:::::`‐-、__  |::::`、     ヒニニヽ、         あなたの論文が Ann of Math に accept される確率は?
    |      / `‐-、::::::::::`‐-、::::\   /,ニニ、\            それとも最近は Inv. Math. の方が上かしら?
   |      |::::::::::::::::::|` -、:::::::,ヘ ̄|'、  ヒニ二、 \
.   |      /::::::::::::::::::|::::::::\/:::O`、::\   | '、   \
   |      /:::::::::::::::::::/:::::::::::::::::::::::::::::'、::::\ノ  ヽ、  |
  |      |:::::/:::::::::/:::::::::::::::::::::::::::::::::::'、',::::'、  /:\__/‐、
  |      |/:::::::::::/::::::::::::::::::::::::::::::::::O::| '、::| く::::::::::::: ̄|
   |     /_..-'´ ̄`ー-、:::::::::::::::::::::::::::::::::::|/:/`‐'::\;;;;;;;_|
   |    |/::::::::::::::::::::::\:::::::::::::::::::::::::::::|::/::::|::::/:::::::::::/
    |   /:::::::::::::::::::::::::::::::::|:::::::::::::::::::::O::|::|::::::|:::::::::::::::/